Chapter 1-6 Exam Questions_Robins & Cotran

अब Quizwiz के साथ अपने होमवर्क और परीक्षाओं को एस करें!

8 (E) One outcome of acute inflammation with ulceration is chronic inflammation. This is particularly true when the inflammatory process continues for weeks to months. Chronic inflammation is characterized by tissue destruction, mononuclear cell infiltration, and repair. In acute inflammation, the healing process of fibrosis and angiogenesis has not begun. Serous inflammation is an inflammatory process involving a mesothelial surface (e.g., lining of the pericardial cavity), with an outpouring of fluid having little protein or cellular content. Granulomatous inflammation is a form of chronic inflammation in which epithelioid macrophages form aggregates. In fibrinous inflammation, typically involving a mesothelial surface, there is an outpouring of protein-rich fluid that results in precipitation of fibrin.

8 A 36-year-old man has had midepigastric abdominal pain for the past 3 months. An upper gastrointestinal endoscopy shows a 2-cm, sharply demarcated, shallow ulceration of the gastric antrum. A biopsy specimen of the ulcer base shows angiogenesis, fibrosis, and mononuclear cell infiltrates with lymphocytes, macrophages, and plasma cells. Which of the following terms best describes this pathologic process? □ (A) Acute inflammation □ (B) Serous inflammation □ (C) Granulomatous inflammation □ (D) Fibrinous inflammation □ (E) Chronic inflammation

8 (C) This spectrum of findings correlates best with trisomy 18 (Edwards syndrome), in which survival is shortened Robbins & Cotran Review of Pathology Pg. 65 significantly. Turner syndrome (45,X) is associated with the presence of cystic hygroma and hydrops fetalis. The severe anomalies described in this case make it unlikely that a normal 46,XX karyotype is present. Down syndrome (47,XX,+21) is associated with longer survival than described in this case, and the external features can be quite subtle at birth. The "superfemale" karyotype (XXX) leads to mild mental retardation. Generally, abnormal numbers of sex chromosomes are tolerated better than abnormalities of autosomes.

8 A 37-year-old woman gives birth at 35 weeks' gestation to a female infant. Physical examination of the infant soon after delivery shows rocker-bottom feet, a small face and mouth, and low-set ears. On auscultation of the chest, a heart murmur is detected. The appearance of the infant's hands is shown in the figure. The infant dies at 4 months of age. Which of the following karyotypes was most likely present in this infant? □ (A) 45,X □ (B) 46,XX □ (C) 47,XX,+18 □ (D) 47,XX,+21 □ (E) 48,XXX

8 (B) Mutations in the AAT gene give rise to AAT molecules that cannot fold properly. In the PiZZ genotype, both alleles Robbins & Cotran Review of Pathology Pg. 11 have the mutation. The partially folded molecules accumulate in the endoplasmic reticulum and cannot be secreted. Impaired dissociation of the CFTR protein from chaperones causes many cases of cystic fibrosis. There is no abnormality in the synthesis, catabolism, or metabolism of AAT in patients with AAT deficiency.

8 A 47-year-old woman has had worsening dyspnea for the past 5 years. A chest CT scan shows panlobular emphysema. Laboratory studies show the PiZZ genotype of α1-antitrypsin (AAT) deficiency. A liver biopsy specimen examined microscopically shows abundant PAS-positive globules within periportal hepatocytes. Which of the following molecular mechanisms is most likely responsible for this finding in the hepatocytes? □ (A) Excessive hepatic synthesis of AAT □ (B) Retention of poorly folded AAT in the endoplasmic reticulum □ (C) Decreased catabolism of AAT in lysosomes □ (D) Inability to metabolize AAT □ (E) Impaired dissociation of AAT from chaperones

8 (B) This man has a selective (isolated) IgA deficiency. Such individuals are bothered by minor recurrent sinopulmonary infections and by diarrhea. Pneumocystis infections are seen in patients with more severe acquired or inherited immunodeficiency disorders, particularly patients with AIDS, which affect cell-mediated immunity. Hepatitis infections are not directly related to immunodeficiency states, although AIDS patients with a history of injection drug use are often infected with hepatitis B or C. Resistance against fungal and viral infection is mediated by T cells.

8 A 48-year-old man has been healthy all of his life, bothered only by an occasional mild diarrheal illness. On physical examination, his temperature is 37.1°C, and blood pressure is 125/85 mm Hg. Laboratory studies show a total WBC count of 6900/mm3 with 72% segmented neutrophils, 3% bands, 18% lymphocytes, and 7% monocytes. Serum immunoglobulin levels are IgG, 1.9 g/dL; IgM, 0.3 g/dL; and IgA, 0.01 g/dL. The ANA test result is negative. The skin test result for mumps and Candida antigens is positive. This patient is at greatest risk of infection from which of the following agents? □ (A) Pneumocystis carinii □ (B) Streptococcus pneumoniae □ (C) Hepatitis B virus □ (D) Aspergillus flavus □ (E) Herpes simplex virus

9 (C) Serous inflammation is the mildest form of acute inflammation. A blister is a good example of serous inflammation. It is associated primarily with exudation of fluid into the subcorneal or subepidermal space. Because the injury is mild, the fluid is relatively protein-poor. A protein-rich exudate results in fibrin accumulation. Acute inflammatory cells, mainly neutrophils, exuded into a body cavity or space form a purulent (suppurative) exudate, typically associated with liquefactive necrosis. Loss of the epithelium leads to ulceration. Granulomatous inflammation is characterized by collections of transformed macrophages called epithelioid cells.

9 A 5-year-old child reaches up to the stove and touches a pot of boiling soup. Within several hours, there is marked erythema of the skin of the fingers on the child's right hand, and small blisters appear on the finger pads. Which of the following terms best describes this process? □ (A) Fibrinous inflammation □ (B) Purulent inflammation □ (C) Serous inflammation □ (D) Ulceration □ (E) Granulomatous inflammation

1 (B) This young woman has a classic picture of systemic lupus erythematosus (SLE)—facial skin rash that is worsened by sunlight and renal failure with proteinuria and hematuria from immune complex deposition in the glomeruli. Defective clearance and hence increased burden of nuclear apoptotic bodies is considered a fundamental mechanism that underlies SLE. This along with loss of self-tolerance to nuclear antigens gives rise to the pathogenic DNA-anti DNA immune complexes. Activation of TH17 cells occurs in many other immunologic disorders such as inflammatory bowel disease. IFN- γ is a product of CD4+ T cells and NK cells. There is no evidence of delayed hypersensitivity or NK cell dysfunction in SLE. Molecular mimicry occurs when a microbial antigen cross-reacts with a normal tissue as in rheumatic fever. Widespread and non-specific activation of T cells by superantigens occurs in toxic shock syndrome.

1 A 25-year-old woman has had increasing malaise, a skin rash of her face exacerbated by sunlight exposure, and arthralgias and myalgias for the past month. On physical examination she has mild pedal edema. On auscultation, a friction rub is audible over the chest. Laboratory findings include pancytopenia and serum creatinine 3 mg/dL. Urinalysis shows hematuria and proteinuria. A serologic test for syphilis yields a false-positive result. A renal biopsy shows a slight increase in mesangial cells and granular deposits of IgG and complement in the mesangium and along the basement membrane. Which of the following mechanisms is most likely involved in the pathogenesis of her disease? □ (A) Activation of TH17 cells □ (B) Defective clearance of apoptotic nuclei □ (C) Increased production of IFN-γ □ (D) Molecular mimicry □ (E) Superantigen activation of T cells

1 (C) Familial hypercholesterolemia results from mutations in the LDL receptor gene, causing LDL cholesterol to be increased in the blood because it is not catabolized or taken up by the liver. It is an autosomal dominant disorder with a carrier rate of 1 in 500, so the frequency of homozygosity is 1 in 1 million. Heterozygotes have total serum cholesterol levels two to three times normal; homozygotes have levels even higher, with death from myocardial infarction by age 15. The "statin" drugs inhibit the HMG-CoA reductase and reduce cholesterol levels in heterozygotes. Steroid hormone receptors, such as those for cortisol, are located in the cell nucleus. Insulin receptors play a role in glucose metabolism and glycemic control that may be part of diabetes mellitus with risk for atherosclerosis; the statin drugs have no effect on diabetes mellitus. Abnormal leptin receptors may play a role in some forms of obesity. TGF-α is a growth factor with a role in inflammation, cell proliferation, and repair.

1 A clinical study is undertaken with subjects from families in which complications of atherosclerotic cardiovascular disease and tendinous xanthomas occurred before age 30. Some of the children in these families are observed to have early atheroma formation. It is observed that affected individuals benefit from treatment with pharmacologic agents that inhibit HMG-CoA reductase. These affected individuals are most likely to have a mutation in a gene encoding a cell surface receptor for which of the following? □ (A) Cortisol □ (B) Insulin □ (C) LDL cholesterol □ (D) Leptin □ (E) TGF-α

1 (F) The COX-2 enzyme is inducible with acute inflammatory reactions, particularly in neutrophils, in synovium, and in the central nervous system. The cyclooxygenase pathway of arachidonic acid metabolism generates prostaglandins, which mediate pain, fever, and vasodilation. Ankle swelling is most likely to result from peripheral edema secondary to congestive heart failure. Asthma results from bronchoconstriction mediated by leukotrienes that are generated by the lipoxygenase pathway of arachidonic acid metabolism. Easy bruisability results from prolonged glucocorticoid administration, which also causes leukopenia. Inhibition of histamine released from mast cells helps reduce urticaria. Fever can be mediated by prostaglandin release, not inhibition.

1 In a 6-month randomized trial of a pharmacologic agent, one group of patients receives a cyclooxygenase-2 (COX-2) inhibitor, and a control group does not. Laboratory measurements during the trial show no significant differences between the groups in WBC count, platelet count, hemoglobin, and creatinine. The group receiving the drug reports subjective findings different from those of the control group, however. Which of the following findings was most likely reported by the group receiving the drug? □ (A) Ankle swelling □ (B) Increased bouts of asthma □ (C) Easy bruisability □ (D) Reduced urticaria □ (E) Increased febrile episodes □ (F) Reduced arthritis pain

1 (C) TGF-β stimulates many steps in fibrogenesis, including fibroblast chemotaxis and production of collagen by fibroblasts, while inhibiting degradation of collagen. All of the other steps listed are unaffected by TGF-β.

1 In an experiment, surgical incisions are made in a study group of laboratory rats. Observations about the wounds are recorded over a 2-week period using various chemical mediators. Which of the following steps in the inflammatory-repair response is most likely affected by neutralization of transforming growth factor β (TGF-β)? □ (A) Leukocyte extravasation □ (B) Increase in vascular permeability □ (C) Production of collagen □ (D) Chemotaxis of lymphocytes □ (E) Migration of epithelial cells

1 (B) The initial response to injury is arteriolar vasoconstriction, but this is transient, and the coagulation mechanism must be initiated to maintain hemostasis. Protein C is involved in anticoagulation to counteract clotting. Platelet aggregation occurs with release of factors such as ADP, but this takes several minutes. Neutrophils are not essential to hemostasis. Fibrin polymerization is part of secondary hemostasis after the vascular injury is initially closed.

1 While shaving one morning, a 23-year-old man nicks his lip with a razor. Seconds after the injury, the bleeding stops. Which of the following mechanisms is most likely to reduce blood loss from a small dermal arteriole? □ (A) Protein C activation □ (B) Vasoconstriction □ (C) Platelet aggregation □ (D) Neutrophil chemotaxis □ (E) Fibrin polymerization

1 0 (C) Reduced workload causes shrinkage of cell size because of loss of cell substance, a process called atrophy. Aplasia refers to lack of embryonic development; hypoplasia describes poor or subnormal development. Dystrophy of muscles refers to inherited disorders of skeletal muscles that lead to muscle weakness and wasting. Hyaline change (hyalinosis) refers to a nonspecific, pink, glassy eosinophilic appearance of cells.

10 A 30-year-old man sustains a left femoral fracture in a skiing accident, and his leg is placed in a plaster cast. After the leg has been immobilized for several weeks, the diameter of the left calf has decreased. This change is most likely to result from which of the following alterations in the calf muscles? □ (A) Aplasia □ (B) Hypoplasia □ (C) Atrophy □ (D) Dystrophy □ (E) Hyalinosis

10 (B) Spread of the cancer to the dermal lymphatics produces a peau d'orange appearance of the breast. Because the breast has an extensive venous drainage, cancer or other focal mass lesions are unlikely to cause significant congestion and edema of the breast. Ischemia is rare in the breast because of the abundant arterial supply. Passive congestion does not involve the breast. Chronic inflammation is rare in breast tissue and is not associated with cancer.

10 A 39-year-old woman comes to the physician because she has noticed a lump in her breast. Over the past 2 months, the left breast has become slightly enlarged compared with the right breast. On physical examination, the skin overlying the left breast is thickened, reddish orange, and pitted. Mammography shows a 3-cm underlying density, and a fine-needle aspirate of the density indicates carcinoma. Which of the following mechanisms best explains the gross appearance of the left breast? □ (A) Venous thrombosis □ (B) Lymphatic obstruction □ (C) Ischemia □ (D) Chronic passive congestion □ (E) Chronic inflammation

1 0 (D) Fibronectin is a key component of the extracellular matrix and has a structure that looks like a paper clip. Fibronectin can be synthesized by monocytes, fibroblasts, and endothelium. Heparin that is infused has an anticoagulant function. Dermatan sulfate, a glycosaminoglycan, acts to form a gel that provides resilience and lubrication. Procollagen produced by fibroblasts is formed into ropelike strands of collagen, which provide tensile strength. Hyaluronic acid binds water to form a gelatinous extracellular matrix.

10 An experiment is conducted involving cellular aspects of wound healing. Components of the extracellular matrix are analyzed to determine their sites of production and their binding patterns to other tissue components. Which of the following molecules synthesized by fibroblasts can best bind to cellular integrins and extracellular collagen and attach epidermal basal cells to basement membrane? □ (A) Heparin □ (B) Dermatan sulfate □ (C) Procollagen □ (D) Fibronectin □ (E) Hyaluronic acid

10 (G) Fever is produced by various inflammatory mediators, but the major cytokines that produce fever are interleukin-1 (IL-1) and tumor necrosis factor (TNF), which are produced by macrophages and other cell types. IL-1 and TNF can have autocrine, paracrine, and endocrine effects. They mediate the acute phase responses, such as fever, nausea, and neutrophil release from marrow. Bradykinin, generated from the kinin system on surface contact of Hageman factor with collagen and basement membrane from vascular injury, promotes vascular permeability, smooth muscle contraction, and pain. Leukotriene B4, generated in the lipoxygenase pathway of arachidonic acid metabolism, is a potent neutrophil chemotactic factor. Histamine released from mast cells is a potent vasodilator, increasing vascular permeability. Myeloperoxidase is contained within the azurophilic granules of neutrophils and in the presence of halide converts hydrogen peroxide to HOCl−, which destroys phagocytized organisms by halogenation. Nitric oxide generated by macrophages aids in destruction of microorganisms; nitric oxide released from endothelium mediates vasodilation and inhibits platelet activation. Phospholipase C, which catalyzes the release of arachidonic acid, is generated from platelet activation.

10 For the past 2 days, a 41-year-old man has had a severe headache, and he now has a temperature of 39.2°C. A lumbar puncture is performed, and the cerebrospinal fluid obtained has a WBC count of 910/mm3 with 94% neutrophils and 6% lymphocytes. Which of the following substances is the most likely mediator for the fever observed in this patient? □ (A) Bradykinin □ (B) Leukotriene B4 □ (C) Histamine □ (D) Myeloperoxidase □ (E) Nitric oxide □ (F) Phospholipase C □ (G) Tumor necrosis factor

1 0 (A) CD4+ cells of the TH2 type are essential to the induction of type I hypersensitivity because they can secrete cytokines, such as interleukin (IL)-4, IL-5, IL-3, and granulocyte-macrophage colony-stimulating factor, which are required for the growth, recruitment, and activation of mast cells and eosinophils. Natural killer cells can lyse other cells, such as virus-infected cells, without prior sensitization. Macrophages can secrete various cytokines, but they are not essential to type I hypersensitivity. Dendritic cells trap antigen and aid in antigen presentation. Neutrophils are recruited by cytokines to participate in acute inflammatory reactions.

10 In an experiment, antigen is used to induce an immediate (type I) hypersensitivity response. Cytokines are secreted that are observed to stimulate IgE production by B cells, promote mast cell growth, and recruit and activate eosinophils in this response. Which of the following cells is most likely to be the source of these cytokines? □ (A) CD4+ lymphocytes □ (B) Natural killer cells □ (C) Macrophages □ (D) Dendritic cells □ (E) Neutrophils

10 (B) The figure shows a single palmar flexion crease and a single flexion crease on the fifth digit, both features of trisomy 21. Although there is an increased risk of Down syndrome with increasing maternal age, most infants with Down syndrome are born to younger women because there are far more pregnancies at younger maternal ages. Monosomy X may be marked by a short fourth metacarpal. With trisomy 18, the fingers are often clenched, with digits 2 and 5 overlapping digits 3 and 4. Triploidy may be marked by syndactyly of digits 3 and 4. There are no characteristic hand features in males with Klinefelter syndrome.

10 The left hand of an infant born at 38 weeks' gestation to a 25-year-old woman, G2, P1, has the appearance shown in the figure. The infant is small for gestational age. Which of the following chromosomal abnormalities is most likely to be present? □ (A) 45,X □ (B) 47,XX,+21 □ (C) 47,XY,+18 □ (D) 69,XXY □ (E) 47,XXY

11 (D) This is a classic pattern of maternal inheritance resulting from a mutation in mitochondrial DNA. Males and females are affected, but affected males cannot transmit the disease to their offspring. Because mitochondrial DNA encodes many enzymes involved in oxidative phosphorylation, mutations in mitochondrial genes exert their most deleterious effects on organs most dependent on oxidative phosphorylation, including the central nervous system and muscles. The other listed options do not exhibit strict maternal inheritance.

11 A 19-year-old man is referred to a neurologist because of failing eyesight and progressive muscle weakness. The neurologist takes a history and finds that several of the patient's male and female relatives have similar symptoms. His mother, her brother and sister, and two of the aunt's children are affected, but the uncle's children are not. Which of the following types of genetic disorders is most likely to be present in this patient? □ (A) Trinucleotide repeat expansion □ (B) Genetic imprinting □ (C) X-linked inheritance pattern □ (D) Mitochondrial mutation □ (E) Uniparental disomy

11 (B) Von Willebrand's factor acts as a "glue" between platelets and the exposed extracellular matrix of the vessel wall after vascular injury. None of the other steps listed depends on von Willebrand's factor. Because the patient's prothrombin time is normal, a lack of prothrombin or the presence of an inhibitor is unlikely.

11 A 29-year-old woman has a history of frequent nosebleeds and increased menstrual flow. On physical examination, petechiae and purpura are present on the skin of her extremities. Laboratory studies show normal partial thromboplastin time, prothrombin time, and platelet count, and decreased von Willebrand factor activity. This patient most likely has a derangement in which of the following steps in hemostasis? □ (A) Vasoconstriction □ (B) Platelet adhesion □ (C) Platelet aggregation □ (D) Prothrombin generation □ (E) Prothrombin inhibition □ (F) Fibrin polymerization

11 (A) The patient has a defect in leukocyte rolling, the first step in transmigration of neutrophils from the vasculature to the tissues. Rolling depends on interaction between selectins (P-selectin and E-selectin on endothelial cells, and LRobbins & Cotran Review of Pathology Pg. 26 selectin on neutrophils) and their sialylated ligand molecules (e.g., sialylated Lewis X). Integrins are involved in the next step of transmigration, during which there is firm adhesion between neutrophils and endothelial cells. Leukotriene B4 is a chemotactic agent, complement C3b facilitates phagocytosis, and NADPH oxidase is involved in microbicidal activity.

11 A 6-year-old child has a history of recurrent infections with pyogenic bacteria, including Staphylococcus aureus and Streptococcus pneumoniae. The infections are accompanied by a neutrophilic leukocytosis. Microscopic examination of a biopsy specimen obtained from an area of soft tissue necrosis shows microbial organisms, but very few neutrophils. An analysis of neutrophil function shows a defect in rolling. This child's increased susceptibility to infection is most likely caused by a defect in which of the following molecules? □ (A) Selectins □ (B) Integrins □ (C) Leukotriene B4 □ (D) Complement C3b □ (E) NADPH oxidase

1 1 (D) The healing process sometimes results in an exuberant production of collagen, giving rise to a keloid. This tendency may run in families. Organization occurs as granulation tissue is replaced by fibrous tissue. Dehiscence occurs when a wound pulls apart. If normal tissue architecture is restored, resolution of inflammation has occurred. Secondary union describes the process by which large wounds fill in and contract.

11 An 18-year-old man lacerated his left hand and required sutures. The sutures were removed 1 week later. Wound healing continued, but the site became disfigured by a prominent raised, nodular scar that developed over the next 2 months. Which of the following terms best describes the process that occurred during this 2-month period? □ (A) Organization □ (B) Dehiscence □ (C) Resolution □ (D) Keloid formation □ (E) Secondary union

11 (B) Germ cells have the highest telomerase activity, and the telomere length can be stabilized in these cells. This allows testicular germ cells to retain the ability to divide throughout life. Normal somatic cells have no telomerase activity, and telomeres progressively shorten with each cell division until growth arrest occurs.

11 An experiment analyzes cells for enzyme activity associated with sustained cellular proliferation. Which of the following cells is most likely to have the highest telomerase activity? □ (A) Endothelial cells □ (B) Germ cells □ (C) Neurons □ (D) Neutrophils □ (E) Erythrocytes

11 (E) This woman has dermatomyositis, a form of inflammatory myopathy in which capillaries are the primary target for antibody and complement-mediated injury. Anti-Jo-1 antibodies, although not present in most cases, are quite specific for inflammatory myopathies. The perivascular and perimysial inflammatory infiltrates result in peripheral muscle fascicular myocyte necrosis. The process is mediated by CD4+ cells and B cells. The heliotrope rash is a characteristic feature of dermatomyositis. Anti-double-stranded DNA is specific for systemic lupus erythematosus (SLE), in which there can be myositis without significant inflammation or necrosis. Rheumatoid factor is present in most patients with rheumatoid arthritis, which is accompanied by inflammatory destruction of joints, not muscle, although muscle may atrophy secondary to diminished movement. The anti-U1-ribonucleoprotein antibodies suggest a diagnosis of mixed connective tissue disease, a condition that can overlap with polymyositis. Antihistone antibodies are associated with drug-induced SLE.

11 For the past 6 weeks, a 52-year-old woman has had bilateral diffuse pain in her thighs and shoulders. She has difficulty rising from a chair and climbing steps. She has a rash with a violaceous color around the orbits and on the skin of her knuckles. On physical examination, she is afebrile. Muscle strength is 4/5 in all extremities. Laboratory studies show serum creatine kinase of 753 U/L, and the ANA test result is positive with a titer of 1 : 160. Which of the following tests is most specific for the diagnosis of this patient's underlying condition? □ (A) Anti-double-stranded DNA antibodies □ (B) Rheumatoid factor □ (C) Anti-U1-ribonucleoprotein antibodies □ (D) Antihistone antibodies □ (E) Anti-Jo-1 antibodies

12 (B) The figure shows the so-called wire loop glomerular capillary lesions of lupus nephritis. Anti-Sm and anti-doublestranded DNA are specific for systemic lupus erythematosus. Anti-Sm is present in only 25% of cases, however. Scl-70 is a marker for diffuse systemic sclerosis. Jo-1 is most specific for polymyositis. HLA-B27 is seen in ankylosing spondylitis. Anticentromere antibody is seen most often with limited scleroderma.

12 A 31-year-old woman has had increasing edema, chest pain, and an erythematous rash for the past 6 months. Laboratory studies show increasing serum creatinine, and urinalysis shows proteinuria with RBC casts. A renal biopsy is performed, and the light microscopic appearance of the PAS-stained specimen is shown in the figure. If present, which of the following antibodies is most helpful in diagnosing this patient's condition? □ (A) Scl-70 □ (B) Anti-Sm □ (C) Jo-1 □ (D) Anti-HLA-B27 □ (E) Anticentromere

12 (C) Inflammation from reflux of gastric acid has resulted in replacement of normal esophageal squamous epithelium by intestinal-type columnar epithelium with goblet cells. Such conversion of one adult cell type to another cell type is called metaplasia. The cells are not significantly increased in size (hypertrophic). The lamina propria has some inflammatory cells, but is not atrophic. Goblet cells are not normal constituents of the esophageal mucosa.

12 A 32-year-old man experiences "heartburn" and gastric reflux after eating a large meal. After many months of symptoms, he undergoes upper gastrointestinal endoscopy, and a biopsy specimen of the esophageal epithelium is obtained. Which of the following pathologic changes, seen in the figure, has most likely occurred? □ (A) Squamous metaplasia □ (B) Mucosal hypertrophy □ (C) Columnar epithelial metaplasia □ (D) Atrophy of lamina propria □ (E) Goblet cell hyperplasia

12 (A) The figure shows a large pulmonary thromboembolus. The most common risk factor is immobilization leading to venous stasis. These thrombi form in the large deep leg or pelvic veins, not in the pulmonary arteries. Coagulopathies from acquired or inherited disorders, such as those from lupus anticoagulant (antiphospholipid antibodies) or factor V (Leiden) mutation, are possible causes of thrombosis, but they usually manifest at a younger age. These causes also are far less common risks for pulmonary thromboembolism than venous stasis. Local inflammation from pneumonia may result in thrombosis of small vessels in affected areas.

12 A 70-year-old man who was hospitalized 3 weeks ago for a cerebral infarction is ambulating (walking) for the first time. Within minutes of returning to his hospital room, he has sudden onset of dyspnea with diaphoresis. He cannot be resuscitated. The gross appearance of the hilum of the left lung at autopsy is shown in the figure. Which of the following risk factors most likely contributed to this finding? □ (A) Venous stasis □ (B) Pulmonary arterial atherosclerosis □ (C) Lupus anticoagulant □ (D) Bronchopneumonia □ (E) Factor V mutation

12 (A) Almost all of the normal genetic material is present in the case of a robertsonian translocation because only a small amount of the p arm from each chromosome is lost. Statistically, one of six fetuses in a mother who carries a robertsonian translocation will also be a carrier. In balanced reciprocal translocation, the same possibility of inheriting the defect exists. The other listed structural abnormalities are likely to result in loss of significant genetic material, reducing survivability, or to interfere with meiosis.

12 A healthy 20-year-old woman, G3, P2, Ab1, comes to the physician for a prenatal visit. She has previously given birth to a liveborn infant and a stillborn infant, both with the same karyotypic abnormality. On physical examination, she is at the 50th percentile for height and weight. She has no physical abnormalities noted. Which of the following karyotypic abnormalities is most likely to be present in this patient? □ (A) Robertsonian translocation □ (B) Ring chromosome □ (C) Isochromosome □ (D) Paracentric inversion □ (E) Deletion of q arm

12 (C) Basic fibroblast growth factor is a potent inducer of angiogenesis. It can participate in all steps of angiogenesis. Epidermal growth factor and interleukin-1 have no angiogenic activity. Platelet-derived growth factor plays a role in vascular remodeling. Endostatin is an inhibitor of angiogenesis.

12 An experiment involves factors controlling wound healing. Skin ulcerations are observed, and the factors involved in the healing process are analyzed. Which of the following factors is most likely to be effective in promoting angiogenesis? □ (A) Platelet-derived growth factor □ (B) Epidermal growth factor □ (C) Basic fibroblast growth factor □ (D) Endostatin □ (E) Interleukin-1

12 (C) The polarizable material is the suture, and a multinucleated giant cell reaction, typically with foreign body giant cells, is characteristic of a granulomatous reaction to foreign material. Chronic inflammation alone is unlikely to produce a localized nodule with giant cells. An abscess, typically from a wound infection, would have liquefactive necrosis and numerous neutrophils. An ulceration involves loss of epidermis or other epithelial layer. Edema refers to accumulation of fluid in the interstitial space. It does not produce a cellular nodule.

12 One month after an appendectomy, a 25-year-old woman palpates a small nodule beneath the skin at the site of the healed right lower quadrant incision. The nodule is excised, and microscopic examination shows macrophages, collagen, a few small lymphocytes, and multinucleated giant cells. Polarizable, refractile material is seen in the nodule. Which of the following complications of the surgery best accounts for these findings? □ (A) Chronic inflammation □ (B) Abscess formation □ (C) Suture granuloma □ (D) Ulceration □ (E) Edema

1 3 (A) This is an example of antibody directed at a parasitic infection, with Fc receptor-mediated inflammation and phagocytosis. IgG and IgE antibodies bearing Fc receptors coat the parasite. Macrophages, natural killer cells, and neutrophils can recognize the Fc receptor and destroy the antibody-coated target cells. Complement-mediated lysis is most typical of immune destruction of RBCs with hemolysis. Langhans giant cells are seen in granulomatous inflammation, a form of type IV hypersensitivity. Acute inflammatory reactions with abscess formation have little effect against tissue parasites. Leukotriene C4 is a potent agent that promotes vascular permeability and bronchial smooth muscle contraction in type I hypersensitivity reactions.

13 A 23-year-old man has had myalgias and a fever for the past week. On physical examination, his temperature is 38.6°C. He has diffuse muscle tenderness, but no rashes or joint pain on movement. Laboratory studies show elevated serum creatine kinase and peripheral blood eosinophilia. Larvae of Trichinella spiralis are present within the skeletal muscle fibers of a gastrocnemius biopsy specimen. Two years later, a chest radiograph shows only a few small calcifications in the diaphragm. Which of the following immunological mechanisms most likely contributed to the destruction of the larvae? □ (A) Antibody Fc receptor-mediated inflammation □ (B) Complement-mediated cellular lysis □ (C) Formation of Langhans giant cells □ (D) Abscess formation with neutrophils □ (E) Synthesis of leukotriene C4 in mast cells

13 (E) The figure shows dense collagen with some remaining dilated blood vessels, typical of the final phase of wound healing, which is extensive by the end of the first month. On day 1, the wound is filled only with fibrin and inflammatory cells. Macrophages and granulation tissue are seen 2 to 3 days postoperatively. Neovascularization is most prominent by days 4 and 5. By week 2, collagen is prominent, and fewer vessels and inflammatory cells are seen.

13 A 24-year-old man with acute appendicitis undergoes surgical removal of the inflamed appendix. The incision site is sutured. A trichrome-stained section of the site is shown in the figure. How long after the surgery would this appearance most likely be seen? □ (A) 1 day □ (B) 2 to 3 days □ (C) 4 to 5 days □ (D) 2 weeks □ (E) 1 month

13 (A) Recurrent thrombotic episodes at such a young age strongly suggest an inherited coagulopathy. The factor V (Leiden) mutation affects 2% to 15% of the population, and more than half of all individuals with a history of recurrent deep venous thrombosis have such a defect. Inherited deficiencies of the anticoagulant proteins antithrombin III and protein C can cause hypercoagulable states, but these are much less common than factor V mutation. Although some cancers elaborate factors that promote thrombosis, this patient is unlikely to have cancer at such a young age; a 10-year history of thrombosis is unlikely to occur in a patient with cancer. Hyperhomocysteinemia is a less common cause of inherited risk of thrombosis than is factor V mutation. It also is a risk factor for atherosclerosis that predisposes to arterial thrombosis. Smoking promotes atherosclerosis with arterial thrombosis.

13 A 25-year-old woman has had multiple episodes of deep venous thrombosis during the past 10 years and one episode of pulmonary thromboembolism during the past year. Prothrombin time, partial thromboplastin time, platelet count, and platelet function studies all are normal. Which of the following risk factors has most likely contributed to the patient's condition? □ (A) Factor V mutation □ (B) Antithrombin III deficiency □ (C) Mutation in protein C □ (D) Hyperhomocysteinemia □ (E) Smoking cigarettes

1 3 (B) Chronic granulomatous disease is characterized by reduced killing of ingested microbes because of inherited defects in the NADPH oxidase system. Two thirds of cases are X-linked, and one third are autosomal recessive. This system generates superoxide anions (O2 −), essential for the subsequent production of microbicidal products such as H2O2, OH, and HOCl−. Macrophage activation by interferon-γ is a key feature of granulomatous inflammation, which is typical of mycobacterial infections. Firm adhesions between leukocytes and endothelium are impaired in leukocyte adhesion deficiency type 1, in which there is a mutation in the β chain of integrins. Lysozyme contained in neutrophil granules is responsible for oxygen-independent killing of bacteria. Impaired opsonization can lead to infections in states of immunoglobulin deficiency.

13 A 3-year-old boy and other male relatives have a history of multiple recurrent infections, including Aspergillus, Staphylococcus, Serratia, Nocardia, and Pseudomonas species. Physical examination shows generalized tender lymphadenopathy. Laboratory findings show normal numbers of morphologically normal circulating WBCs. This child's increased susceptibility to infection is most likely caused by a defect in which of the following steps of the inflammatory response? □ (A) Activation of macrophages by interferon-γ □ (B) Oxygen-dependent killing of bacteria by neutrophils □ (C) Firm adhesion between leukocytes and endothelial cells □ (D) Synthesis of lysozyme in neutrophil granules □ (E) Opsonization of bacteria by immunoglobulins

13 (D) Cleft lip and palate, along with microcephaly and polydactyly, are features of trisomy 13 (Patau syndrome). These infants also commonly have severe heart defects. For monosomy X (45,X) to be considered, the infant must be female. The severe anomalies described in this case may occur with a normal karyotype (46,XY), but the spectrum of findings, particularly the polydactyly, suggests trisomy 13. Klinefelter syndrome (47,XXY) results in phenotypic males who are hard to distinguish from males with a 46,XY karyotype. Infants with trisomy 18 lack polydactyly and are more likely to have micrognathia than are infants with trisomy 13. Triploidy with 69 chromosomes leads to stillbirth in virtually all cases.

13 A 36-year-old woman gives birth at 34 weeks' gestation to a male infant who lives for only 1 hour after delivery. On physical examination, the infant is at the 30th percentile for height and weight. Anomalies include microcephaly, a cleft lip and palate, and postaxial polydactyly, with six digits on each hand and foot. Which of the following karyotypes is most likely to be present in this infant? □ (A) 45,X □ (B) 46,XY □ (C) 47,XXY □ (D) 47,XY,+13 □ (E) 47,XY,+18 □ (F) 69,XXY

13 (A) The onset of menstruation is an example of orderly, programmed cell death (apoptosis) through hormonal stimuli. The endometrium breaks down, sloughs off, and then regenerates. Caseous necrosis is typical of granulomatous inflammation, resulting most commonly from mycobacterial infection. Heterophagocytosis is typified by the clearing of an area of necrosis through macrophage ingestion of the necrotic cells. With cellular atrophy, there is often no visible necrosis, but the tissues shrink, something that occurs in the endometrium after menopause. Liquefactive necrosis can occur in any tissue after acute bacterial infection or in the brain after ischemia.

13 On day 28 of her menstrual cycle, a 23-year-old woman experiences onset of menstrual bleeding that lasts for 6 days. She has had regular cycles for many years. Which of the following processes is most likely occurring in the endometrium just before the onset of bleeding? □ (A) Apoptosis □ (B) Caseous necrosis □ (C) Heterophagocytosis □ (D) Atrophy □ (E) Liquefactive necrosis

1 4 (B) These findings represent chronic rejection. The progressive renal failure results from ischemic changes with vascular narrowing. Cell lysis with macrophages is typical of antibody-dependent cell-mediated cytotoxicity, which does not play a key role in chronic rejection. Granulomatous inflammation is not typical of transplant rejection. Release of leukotriene C4 from mast cells is a feature of type I hypersensitivity. Complement-mediated cell lysis can occur when antidonor antibodies are preformed in the host, as occurs in hyperacute rejection.

14 A 45-year-old man with chronic renal failure received a kidney transplant from his brother 36 months ago. For the next 30 months, he had only minor episodes of rejection that were controlled with immunosuppressive therapy. In the past 6 months, he has had increasing serum creatinine and urea nitrogen levels. On physical examination, he is afebrile. Microscopic examination of a urinalysis specimen shows no WBCs. CT scan of the pelvis shows that the allograft is reduced in size. Which of the following immunological processes most likely accounts for these findings? □ (A) Macrophage-mediated cell lysis □ (B) Vascular intimal fibrosis □ (C) Granulomatous vasculitis □ (D) Release of leukotriene C4 from mast cells □ (E) Complement-mediated cell lysis

14 (A) Hepatocytes are quiescent (stable) cells that can reenter the cell cycle and proliferate in response to hepatic injury, enabling the liver to regenerate partially. Acute hepatitis results in hepatocyte necrosis, marked by elevations in AST and ALT. After the acute process has ended, cells return to the G0 phase, and the liver becomes quiescent again.

14 A 50-year-old woman tests positive for hepatitis A antibody. The serum AST level is 275 U/L, and ALT is 310 U/L. One month later, these enzyme levels have returned to normal. Which phase of the cell cycle best describes the hepatocytes 1 month after infection? □ (A) G0 □ (B) G1 □ (C) S □ (D) G2 □ (E) M

14 (E) This infant has Pompe disease, a form of glycogen storage disease that results from a deficiency in lysosomal glucosidase (acid maltase). The glycogen stored in the myocardium results in massive cardiomegaly and heart failure within 2 years. Glucocerebrosidase deficiency occurs in Gaucher disease. In the most common form of the disease, there is no neurologic impairment, and patients have splenomegaly and skeletal disease as a consequence of increased lysosomal glucocerebrosides in cells of the mononuclear phagocyte system. Glucose-6-phosphatase deficiency leads to von Gierke disease, characterized by hepatomegaly, renomegaly, and impaired gluconeogenesis leading to hypoglycemia and hyperlipidemia. Hexosaminidase A deficiency occurs in Tay-Sachs disease and is associated with severe neurologic impairment, poor motor development, and blindness beginning in infancy. Homogentisic oxidase deficiency leads to alkaptonuria with ochronosis or to deposition of a blue-black pigment in joints, resulting in arthropathy. Myophosphorylase Robbins & Cotran Review of Pathology Pg. 66 deficiency leads to McArdle disease, characterized by muscle cramping after strenuous exercise. Sphingomyelinase deficiency occurs in Niemann-Pick disease; affected individuals with type A have hepatosplenomegaly, lymphadenopathy, and severe motor and mental impairment.

14 A 6-month-old male infant is brought to the physician because of failure to thrive and abdominal enlargement. His parents are concerned that he has shown minimal movement since birth. On physical examination, the infant has marked muscle weakness and hepatosplenomegaly. A chest radiograph shows marked cardiomegaly. He dies of congestive heart failure at age 19 months. The microscopic appearance of myocardial fibers at autopsy is shown in the figure. A deficiency of which of the following enzymes is most likely to be present in this infant? □ (A) Glucocerebrosidase □ (B) Glucose-6-phosphatase □ (C) Hexosaminidase A □ (D) Homogentisic oxidase □ (E) Lysosomal glucosidase □ (F) Myophosphorylase □ (G) Sphingomyelinase

14 (D) The patient has deep and superficial venous thrombosis as a consequence of venous stasis from immobilization. The large, deep thrombi can embolize to the lungs, leading to death. Gangrene occurs from arterial, not venous, occlusion in the leg. Vessels with thrombi typically stay intact; if a hematoma had developed as a consequence of the trauma from the fall, it would be organizing and decreasing in size after 2 weeks. Disseminated intravascular coagulation is not a common complication in patients with thrombosis of the extremities or in patients recuperating from an injury. Fat embolism can occur with fractures, but pulmonary problems typically appear 1 to 3 days after the traumatic event.

14 A 76-year-old woman is hospitalized after falling and fracturing her left femoral trochanter. Two weeks later, the left leg is swollen, particularly below the knee. She experiences pain on movement of the leg; on palpation, there is tenderness. Which of the following complications is most likely to occur after these events? □ (A) Gangrenous necrosis of the foot □ (B) Hematoma of the thigh □ (C) Disseminated intravascular coagulation □ (D) Pulmonary thromboembolism □ (E) Fat embolism

14 (B) When DNA damage is induced by chemotherapeutic drugs (or other agents), normal p53 genes trigger the cells to undergo apoptosis. When p53 is inactivated, this pathway of cell death can be blocked, rendering the chemotherapy less effective. BCL-2 and NF-κB activity favor cell survival. Cytochrome P-450 does not affect apoptosis. Granzyme B can trigger apoptosis, but it is found in cytotoxic T cells and not in tumor cells.

14 In a clinical trial, a chemotherapeutic agent is given to patients with breast cancer metastases. Samples of the cancer cells are obtained and assessed for the presence of death of tumor cells by apoptosis. Mutational inactivation of which of the following products is most likely to render tumor cells resistant to the effects of such an agent? □ (A) BCL-2 □ (B) p53 □ (C) NF-κB □ (D) Cytochrome P-450 □ (E) Granzyme B

14 (E) The respiratory, or oxidative, burst generates reactive oxygen species (i.e., superoxide anion) that are important in destruction of engulfed bacteria. Myelopoiesis does not depend on generation of superoxide. Endothelial attachment of neutrophils is aided by adhesion molecules on the endothelium and the neutrophil surface. These molecules include selectins and integrins. Bacteria are opsonized by complement C3b and IgG, allowing the bacteria to be more readily phagocytosed.

14 In an experiment, neutrophils collected from peripheral blood are analyzed for a "burst" of oxygen consumption. This respiratory burst is an essential step for which of the following events in an acute inflammatory response? □ (A) Increased production in bone marrow □ (B) Attachment to endothelial cells □ (C) Opsonization of bacteria □ (D) Phagocytosis of bacteria □ (E) Generation of microbicidal activity

15 (A) This child has one of the three forms of Gaucher disease. Type 1, seen in this child, accounts for 99% of cases and does not involve the central nervous system (CNS). It is caused by a deficiency of glucocerebrosidase, and infusion with this enzyme reduces severity and progression. Type 2 involves the CNS and is lethal in infancy. Type 3 also involves the CNS, although not as severely as type 2. A deficiency of acid maltase is a feature of Pompe disease. Von Gierke disease results from deficiency of glucose-6-phosphatase. Sphingomyelinase deficiency leads to Niemann-Pick disease types A and B. Type A, the more common form, is associated with severe neurologic deterioration. Type B, the less common form, may resemble the findings in this case, but the appearance of macrophages is different: they contain many small vacuoles. Tay-Sachs disease involves a deficiency of hexosaminidase A and is associated with severe mental retardation and death before 10 years of age.

15 A 10-year-old child has had recurrent otitis media for the past 8 years. On physical examination, there is hepatosplenomegaly. No external anomalies are present. Laboratory findings include anemia and leukopenia. A bone marrow biopsy is performed, and high magnification of the sample shows the findings depicted in the figure. An inherited deficiency of which of the following enzymes is most likely to produce these findings? □ (A) Glucocerebrosidase □ (B) Acid maltase □ (C) Glucose-6-phosphatase □ (D) Sphingomyelinase □ (E) Hexosaminidase A

1 5 (A) Factor VIII, tissue factor (thromboplastin), and factor V act as cofactors or reaction accelerators in the clotting cascade. Factor VIII acts as a reaction accelerator for the conversion of factor X and factor Xa. The platelet surface provides phospholipid for assembly of coagulation factors. Platelet aggregation is promoted by thromboxane A2 and ADP. Thromboxane A2 is released when platelets are activated during the process of platelet adhesion. Fibrin polymerization is promoted by factor XIII. Antithrombin III inhibits thrombin to prolong the prothrombin time.

15 A 12-year-old boy has a history of multiple soft tissue hemorrhages and acute upper airway obstruction from hematoma formation in the neck. On physical examination, he has decreased range of motion of the large joints, particularly the knees and ankles. He has no petechiae or purpura of the skin. Laboratory studies show normal prothrombin time, elevated partial thromboplastin time, and normal platelet count, but markedly decreased factor VIII activity. Which of the following mechanisms best describes the development of this disease? □ (A) Decrease in a coagulation cascade component □ (B) Decrease in membrane phospholipid □ (C) Failure of platelet aggregation □ (D) Failure of fibrin polymerization □ (E) Inability to neutralize antithrombin III □ (F) Inability of platelets to release thromboxane A2

15 (A) Natural killer (NK) cells have the ability to respond without prior sensitization. They carry receptors for MHC class I molecules, which inhibit their lytic function. When expression of class I MHC molecules is reduced on the cell surface, the inhibitory receptors on NK cells do not receive a negative signal, and the cell is killed. NK cells are often the first line of defense against viral infection. Neutrophils provide a nonspecific immune response, primarily to bacterial infections and not to intracellular viral infections. Macrophages can process antigen and can phagocytize necrotic cells. CD4+ cells are helper T cells that assist other cells, such as NK cells, macrophages, and B cells, in the immune response. Dendritic cells aid in antigen presentation.

15 A 20-year-old man steps into an elevator full of people who are coughing and sneezing, all of whom appear to have colds or the flu. The influenza viral particles that he inhales attach to respiratory epithelium, and viral transformation reduces the MHC class I molecules on these epithelial cells. Which of the following cells is most likely to respond to destroy the infected cells? □ (A) Natural killer cell □ (B) Neutrophil □ (C) Macrophage □ (D) CD4 cell □ (E) Dendritic cell

15 (C) This patient is experiencing an acute inflammatory response, with edema, erythema, and pain of short duration. Neutrophils form an exudate and release various proteases, which can produce liquefactive necrosis, starting at the mucosa and extending through the wall of the tube. This mechanism results in perforation. Fibroblasts are more likely participants in chronic inflammatory responses and in healing responses, generally appearing more than 1 week after the initial event. Langhans giant cells are a feature of granulomatous inflammation. Mononuclear infiltrates are more typical of chronic inflammation of the fallopian tube, in which rupture is less likely. Epithelial metaplasia is most likely to occur in the setting of chronic irritation with inflammation.

15 A 20-year-old, sexually active woman experiences lower abdominal pain of 24 hours' duration. She has no previous history of this type of pain. Her temperature is 37.9°C, and on palpation, the left lower abdomen is markedly tender. Laboratory findings include a total WBC count of 29,000/mm3 with 75% segmented neutrophils, 6% bands, 14% lymphocytes, and 5% monocytes. Laparotomy reveals a distended, fluid-filled, reddened left fallopian tube that is about to rupture. A left salpingectomy is performed. Which of the following is most likely to be seen on microscopic examination of the excised fallopian tube? □ (A) Fibroblastic proliferation □ (B) Langhans giant cells □ (C) Liquefactive necrosis □ (D) Mononuclear infiltrates □ (E) Squamous metaplasia

18 (C) Endothelial cells can release nitric oxide to produce vasodilation. Nitric oxide also can be administered to patients to promote vasodilation in areas of ischemic injury. Thromboxane A2, platelet-activating factor, and leukotriene E4 have vasoconstrictive properties. Bradykinin mainly increases vascular permeability and produces pain.

18 A 78-year-old woman experiences a sudden loss of consciousness, with loss of movement on the right side of the body. Cerebral angiography shows an occlusion of the left middle cerebral artery. To prevent further ischemic injury to the cerebral cortex, which of the following mediators would be most beneficial? □ (A) Thromboxane A2 □ (B) Bradykinin □ (C) Nitric oxide □ (D) Platelet-activating factor □ (E) Leukotriene E4

1 5 (B) Lobules increase under hormonal influence (mainly progesterone) to provide for normal lactation. The breast stroma plays no role in lactation and may increase with pathologic processes. Epithelial dysplasia denotes disordered growth and maturation of epithelial cells that may progress to cancer. Accumulation of fat within cells is a common manifestation of sublethal cell injury or, uncommonly, of inborn errors in fat metabolism. Epithelial metaplasia in the breast is a pathologic process.

15 After the birth of her first child, a 19-year-old woman breastfed the infant for about 1 year. Which of the following processes that occurred in the breast during pregnancy allowed her to breastfeed the infant? □ (A) Stromal hypertrophy □ (B) Lobular hyperplasia □ (C) Epithelial dysplasia □ (D) Intracellular accumulation of fat □ (E) Ductal epithelial metaplasia

15 (B) Embryonic stem (ES) cells are multipotent and can give rise to all cells, including hepatocytes. Gene targeting to produce "knockout" mice is done in cultures of ES cells, which are then injected into mouse blastocysts and implanted into the uterus of a surrogate mother. Mesenchymal stem cells also are multipotential, but they are not useful for gene targeting. Hematopoietic stem cells can give rise to all hematopoietic cells, but not other types of cells. Hepatocytes and oval cells within the liver can give rise only to liver cells.

15 In an experiment, the role of low-density lipoprotein (LDL) receptors in uptake of lipids in the liver is studied. A mouse model is created in which the LDL receptor gene is not expressed in the liver. For creating such a "knockout" mouse, which of the following cells would be most useful? □ (A) Adult bone marrow mesenchymal progenitor cells □ (B) Embryonic stem cells in culture □ (C) Hepatic oval cells □ (D) Hematopoietic stem cells □ (E) Regenerating hepatocytes

16 (E) Each unit of blood contains about 250 mg of iron. The body has no mechanism for getting rid of excess iron. About 10 to 20 mg of iron per day is lost with normal desquamation of epithelia; menstruating women lose slightly more. Any excess iron becomes storage iron, or hemosiderin. Over time, hemosiderosis involves more and more tissues of the body, particularly the liver. Initially, hemosiderin deposits are found in Kupffer cells and other mononuclear phagocytes in the bone marrow, spleen, and lymph nodes. With great excess of iron, liver cells also accumulate iron. Steatosis usually Robbins & Cotran Review of Pathology Pg. 12 occurs with ingestion of hepatotoxins, such as alcohol. Bilirubin, a breakdown product of blood, can be excreted in the bile so that a person does not become jaundiced. Glycogen storage diseases are inherited and present in childhood. Amyloid is an abnormal protein derived from a variety of precursors, such as immunoglobulin light chains.

16 A 22-year-old woman has a congenital anemia that has required multiple transfusions of RBCs for many years. On physical examination, she now has no significant findings; however, liver function tests show reduced serum albumin. Which of the following findings would most likely appear in a liver biopsy specimen? □ (A) Steatosis in hepatocytes □ (B) Bilirubin in canaliculi □ (C) Glycogen in hepatocytes □ (D) Amyloid in portal triads □ (E) Hemosiderin in hepatocytes

16 (E) These findings indicate a failure of terminal differentiation of B cells into IgA-secreting plasma cells. Lack of IgA in mucosal secretions increases the risk of respiratory and gastrointestinal infections. IgA antibodies present in serum can lead to a transfusion reaction with IgA in donor serum. Individuals with severe combined immunodeficiency would not live as long as this patient with such mild infections. HIV infection is marked by failure of cell-mediated immunity. DiGeorge syndrome manifests in infancy with failure of cell-mediated immunity from lack of functional T cells. Wiskott-Aldrich syndrome is associated with eczema and thrombocytopenia.

16 A 35-year-old man has a history of mild infections of the upper respiratory tract. He also has had diarrhea for most of his life, although not severe enough to cause malabsorption and weight loss. After an episode of trauma with blood loss, he receives a blood transfusion and has an anaphylactic reaction. Which of the following underlying conditions best explains these findings? □ (A) Severe combined immunodeficiency □ (B) HIV infection □ (C) DiGeorge syndrome □ (D) Wiskott-Aldrich syndrome □ (E) Selective IgA deficiency

16 (D) Interferon-γ is secreted by activated T cells and is an important mediator of granulomatous inflammation. It causes activation of macrophages and their transformation into epithelioid cells and then giant cells. Tumor necrosis factor can be secreted by activated macrophages and induces activation of lymphocytes and proliferation of fibroblasts, which are other elements of a granuloma. Complement C3b acts as an opsonin in acute inflammatory reactions. Leukotriene B4 induces chemotaxis in acute inflammatory processes. Interleukin-1 can be secreted by macrophages to produce various effects, including fever, leukocyte adherence, fibroblast proliferation, and cytokine secretion.

16 A 9-year-old boy has had a chronic cough and fever for the past month. A chest radiograph shows enlargement of hilar lymph nodes and bilateral pulmonary nodular interstitial infiltrates. A sputum sample contains acid-fast bacilli. A transbronchial biopsy specimen shows granulomatous inflammation within the lung, marked by the presence of Langhans giant cells. Which of the following mediators is most likely to contribute to giant cell formation? □ (A) Tumor necrosis factor □ (B) Complement C3b □ (C) Leukotriene B4 □ (D) Interferon-γ □ (E) Interleukin-1

16 (E) The figure shows a subacute infarction with granulation tissue formation containing numerous capillaries stimulated by vascular endothelial growth factor, representing a healing response. Epidermal growth factor aids in re-epithelialization of a surface wound. Interleukin-2 mediates lymphocyte activation. Leukotriene B4 mediates vasoconstriction and bronchoconstriction. Thromboxane A2 aids vasoconstriction and platelet aggregation. Tumor necrosis factor induces endothelial activation and many responses that occur secondary to inflammation, including fever, loss of appetite, sleep disturbances, hypotension, and increased corticosteroid production.

16 In an experiment, glass beads are embolized into the coronary arteries of rats, resulting in myocardial injury. After 7 days, sections of the myocardium are studied using light microscopy. The microscopic appearance of one of these sections is shown in the figure. Which of the following mediators is most likely being expressed to produce this appearance? □ (A) Epidermal growth factor □ (B) Interleukin-2 □ (C) Leukotriene B4 □ (D) Thromboxane A2 □ (E) Vascular endothelial growth factor □ (F) Tumor necrosis factor

16 (A) These findings are characteristic of fragile X syndrome, a condition in which there are 250 to 4000 tandem repeats of the trinucleotide sequence CGG. Generally, as the number of trinucleotide repeats increases, the manifestations of the associated conditions worsen or have an earlier onset. The trinucleotide mutations are dynamic; because their number increases during oogenesis, subsequent male offspring have more severe disease compared with earlier generations. With a frameshift mutation, one, two, or three nucleotide base pairs are inserted or deleted. As a result, the protein transcribed is abnormal. A missense mutation results from a single nucleotide base substitution, and it leads to elaboration of an abnormal protein. Abnormalities of mitochondrial DNA, typically involving genes associated with oxidative phosphorylation, are transmitted on the maternal side.

16 Mental retardation has affected several generations of a family, and most of the affected individuals have been males. The severity of mental retardation has increased with each passing generation. Genetic testing is performed, and about 20% of the males who have the genetic abnormality are unaffected. Which of the following mechanisms is most likely to produce this genetic condition? □ (A) Trinucleotide repeat mutation □ (B) Frameshift mutation □ (C) Missense mutation □ (D) Point mutation □ (E) Mitochondrial DNA mutation

16 (D) The patient is quickly going into shock after trauma. So-called shock lung, with diffuse alveolar damage, is common in this situation. Infarction of the liver is uncommon because of this organ's dual blood supply. Basal ganglia hemorrhages are more typical of hypertension, not hypotension with shock. Passive congestion is less likely because of the diminished blood volumes and tissue perfusion that occur in shock. Gangrene requires much longer to develop and is not a common complication of shock.

16 Within 1 hour after a gunshot wound to the abdomen, a 19-year-old man exhibits tachycardia. His skin is cool and clammy to the touch, and blood pressure is 80/30 mm Hg. Which of the following organ-specific changes is most likely to occur within 2 days after this injury? □ (A) Acute hepatic infarction □ (B) Cerebral basal ganglia hemorrhage □ (C) Renal passive congestion □ (D) Pulmonary diffuse alveolar damage □ (E) Gangrenous necrosis of the lower legs

17 (C) The patient has neurofibromatosis type 1 (NF-1), characterized by the development of multiple neurofibromas and pigmented skin lesions. Neurofibromas are most numerous in the dermis but also may occur in visceral organs. Patients with NF-1 also may develop a type of sarcomatous neoplasm known as a malignant peripheral nerve sheath tumor. Dermatofibromas are subcutaneous masses that typically are small and solitary. Leiomyomas occur most frequently in the uterus. Lipomas can occur almost anywhere in the body, but do so sporadically. Hemangiomas may occur sporadically on the skin; they typically are red-blue masses.

17 A 15-year-old girl is brought to the physician by her parents, who are concerned because she has developed multiple nodules on her skin. On physical examination, there are 20 scattered, 0.3- to 1-cm, firm nodules on the patient's trunk and extremities. There are 12 light brown macules averaging 2 to 5 cm in diameter on the skin of the trunk. Slit-lamp examination shows pigmented nodules in the iris. These findings are most likely to be associated with which of the following types of neoplasm? □ (A) Dermatofibroma Robbins & Cotran Review of Pathology Pg. 59 □ (B) Leiomyoma □ (C) Neurofibroma □ (D) Lipoma □ (E) Hemangioma

1 7 (C) Myeloperoxidase is present in the azurophilic granules of neutrophils. It converts H2O2 into HOCl−, a powerful oxidant and antimicrobial agent. Degranulation occurs when phagolysosomes are formed with engulfed bacteria in phagocytic vacuoles within the neutrophil cytoplasm. In contrast, prostaglandin production depends on a functioning cyclooxygenase pathway of arachidonic acid metabolism. Oxygen consumption with an oxidative or respiratory burst after phagocytosis is aided by glucose oxidation and activation of neutrophil NADPH oxidase, resulting in generation of superoxide that is converted by spontaneous dismutation to H2O2.

17 A 5-year-old child has a history of recurrent bacterial infections, including pneumonia and otitis media. Analysis of leukocytes collected from the peripheral blood shows a deficiency in myeloperoxidase. Which of the following is the most likely cause of this child's increased susceptibility to infections? □ (A) Defective neutrophil degranulation □ (B) Defective production of prostaglandins □ (C) Failure to produce hydroxy-halide radicals (HOCl−) □ (D) Decreased oxygen consumption after phagocytosis □ (E) Failure to produce hydrogen peroxide

17 (B) The deep eosinophilic staining, loss of nuclei, and loss of cell structure suggest an early ischemic injury, resulting in coagulative necrosis. This finding is typically caused by loss of blood flow. Viral infection could cause necrosis of the myocardium, but this is usually accompanied by an inflammatory infiltrate consisting of lymphocytes and macrophages. Blunt trauma produces hemorrhage. An immunological injury may produce focal cell injury, but not widespread ischemic injury. Lack of protein leads to a catabolic state with gradual decrease in cell size, but it does not cause ischemic changes.

17 A 50-year-old man experienced an episode of chest pain 6 hours before his death. A histologic section of left ventricular myocardium taken at autopsy showed a deeply eosinophilic-staining area with loss of nuclei and cross-striations in myocardial fibers. There was no hemorrhage or inflammation. Which of the following conditions most likely produced these myocardial changes? □ (A) Viral infection □ (B) Coronary artery thrombosis □ (C) Blunt chest trauma □ (D) Antibodies directed against myocardium □ (E) Protein-deficient diet

17 (B) Atherosclerotic damage to vascular endothelium is the most common cause of arterial thrombosis; this damage occurs almost imperceptibly over many years. Inhibitors to coagulation, such as antiphospholipid antibodies, typically prolong the partial thromboplastin time, the prothrombin time, or both. Decreased levels of antithrombin III and mutation in the factor V gene are inherited causes of hypercoagulability; they are far less common than atherosclerosis of coronary vessels. Decreased production of tissue plasminogen activator from intact endothelial cells may occur in anoxia of the endothelial cells in veins with sluggish circulation. Stasis of blood flow is important in the low-pressure venous circulation.

17 A 56-year-old man with a history of diabetes mellitus goes to the emergency department because he has had left-sided chest pain that radiates to the arm for the past 5 hours. Serial measurements of serum creatine kinase-MB levels show an elevated level 24 hours after the onset of pain. Partial thromboplastin time and prothrombin time are normal. Coronary angiography shows occlusion of the left anterior descending artery. Which of the following mechanisms is the most likely cause of thrombosis in this patient? □ (A) Antibody inhibitor to coagulation □ (B) Damage to endothelium □ (C) Decreased antithrombin III level □ (D) Decreased tissue plasminogen activator □ (E) Mutation in factor V gene □ (F) Stasis of blood flow

1 7 (C) This history is typical of the late-phase reaction in type I hypersensitivity. The initial rapid response is largely caused by degranulation of mast cells. The late-phase reaction follows without additional exposure to antigen and is characterized by more intense infiltration by inflammatory cells, such as neutrophils, eosinophils, basophils, monocytes, and CD4+ lymphocytes. There is more tissue destruction in this late phase.

17 A laboratory worker who is "allergic" to fungal spores is accidentally exposed to a culture of the incriminating fungus on a Friday afternoon. Within 1 hour, he develops bouts of sneezing, watery eyes, and nasal discharge. The symptoms seem to subside within a few hours of returning home, but reappear the next morning, although the laboratory fungus is not present in his home environment. The symptoms persist through the weekend, and on Monday morning, he sees the physician. Which of the following cells is most likely to be seen on microscopic examination of the patient's nasal discharge? □ (A) Mast cells and neutrophils □ (B) Lymphocytes and macrophages □ (C) Neutrophils, eosinophils, and CD4+ lymphocytes □ (D) Neutrophils and CD8+ lymphocytes □ (E) Mast cells, lymphocytes, and macrophages

17 (D) Interferon-γ is a cytokine secreted by lymphocytes at the site of inflammation; it has a paracrine effect that causes transformation of monocytes to activated macrophages. Acetylcholine is a neurotransmitter that does not act on inflammatory cells. Cyclic AMP acts as a "second messenger" within a cell. Heparan sulfate is a component of the extracellular matrix that does not have a signaling function. Transforming growth factor β has an inhibitory function on leukocytes.

17 An experiment is conducted in which the time sequence of events in wound healing is analyzed. Histologic sections are produced from samples of the tissue at the site of a small superficial skin incision in laboratory animals. During the first week, the number of macrophages that are activated to phagocytize tissue debris increases. Which of the following signaling molecules is most likely to play a significant role in producing this finding? □ (A) Acetylcholine □ (B) Cyclic AMP □ (C) Heparan sulfate □ (D) Interferon-γ □ (E) Transforming growth factor β

1 8 (B) Macrophages can become infected with HIV and are not destroyed as CD4+ cells are. Instead, macrophages survive to carry the infection to tissues throughout the body, particularly the brain. HIV infection of the brain can result in encephalitis and dementia. Natural killer cells and neutrophils play no significant role in HIV infection. CD8+ lymphocytes cannot be infected with HIV. Langerhans cells in mucosal surfaces may aid in initial HIV infection of CD4+ lymphocytes.

18 A 30-year-old man infected with HIV begins to have difficulty with activities of daily living. He has memory problems and decreased ability to perform functions that require fine motor control, such as writing and painting. His CD4+ lymphocyte count currently is 150/μL. Which of the following cell types is most important for the dissemination of the infection into the central nervous system? □ (A) Natural killer cell □ (B) Macrophage □ (C) Neutrophil □ (D) CD8+ lymphocyte □ (E) Langerhans cell

18 (C) These features are characteristic of trisomy 21, but the child is not severely affected, which suggests mosaicism. In mosaic individuals, greater numbers of potentially normal cells having the proper chromosomal complement are present, which may allow infants with abnormalities of chromosome number to survive to term and beyond. Haploidy is present in gametes. Loss of an autosomal chromosome is devastating; the only monosomy associated with possible survival to term is Turner syndrome (monosomy X). Most aneuploid conditions (trisomies and monosomies) lead to fetal demise; fetuses with trisomy 21 are the most likely to survive to term. Triploid fetuses rarely survive beyond the second trimester and are virtually never live-born. Likewise, tetraploidy accounts for many first-trimester fetal losses and is not survivable.

18 A 39-year-old woman gives birth to a term infant with a right transverse palmar crease, low-set ears, oblique palpebral fissures, and a heart murmur. The infant survives to childhood and exhibits only mild mental retardation. Which of the following chromosomal abnormalities is most likely to be present in the somatic cells of this child? □ (A) Haploidy □ (B) Monosomy □ (C) Mosaicism □ (D) Tetraploidy □ (E) Triploidy

18 (D) Nodular prostatic hyperplasia (also known as benign prostatic hyperplasia [BPH]) is a common condition in older men that results from proliferation of prostatic glands and stroma. The prostate becomes more sensitive to androgenic stimulation with age. This is an example of pathologic hyperplasia. Apoptosis results in a loss of, not an increase in, cells. Dysplasia refers to disordered epithelial cell growth and maturation. Fatty change in hepatocytes may produce hepatomegaly. Although BPH is often called "benign prostatic hypertrophy," this term is technically incorrect; it is the number of glands and stromal cells that is increased, rather than the size of existing cells. A change in the glandular epithelium to squamous epithelium would be an example of metaplasia.

18 A 69-year-old man has had difficulty with urination, including hesitancy and frequency, for the past 5 years. A digital rectal examination reveals that the prostate gland is palpably enlarged to about twice normal size. A transurethral resection of the prostate is performed, and the microscopic appearance of the prostate "chips" obtained is that of nodules of glands with intervening stroma. Which of the following pathologic processes has most likely occurred in the prostate? □ (A) Apoptosis □ (B) Dysplasia □ (C) Fatty change □ (D) Hyperplasia □ (E) Hypertrophy □ (F) Metaplasia

1 8 (A) ADP is released from the platelet dense bodies and is a potent stimulator of platelet aggregation. ADP also stimulates further release of ADP from other platelets. Many other substances involved in hemostasis, such as fibrinogen, von Willebrand factor (vWF), and factor V, are stored in the α granules of platelets. Thromboxane A2, another powerful aggregator of platelets, is synthesized by the cyclooxygenase pathway; it is not stored in dense bodies. Fibronectin forms part of the extracellular matrix between cells that "glues" them together. Plasminogen is activated to inhibit coagulation. Platelet aggregation requires active platelet metabolism; platelet stimulation by agonists such as ADP, thrombin, collagen, or epinephrine; the presence of calcium or magnesium ions and specific plasma proteins such as fibrinogen or vWF; and a platelet receptor, the glycoprotein IIb/IIIa (GPIIb/IIIa) complex. Platelet stimulation results in the generation of intracellular second messengers that transmit the stimulus back to the platelet surface, exposing protein-binding sites on GPIIb/IIIa. Fibrinogen or vWF then binds to GPIIb/IIIa and cross-links adjacent platelets to produce platelet aggregates. The patients in this experiment could have Glanzmann thrombasthenia, in which platelets are deficient or defective in the GPIIb/IIIa complex, do not bind fibrinogen, and cannot form aggregates, although the platelets can be stimulated by ADP, undergo shape change, and are of normal size.

18 An experiment is conducted in which platelet function is analyzed. A substance is obtained from the dense bodies (delta granules) of normal pooled platelets from healthy blood donors. When this substance is added to platelets obtained from patients with a bleeding disorder, no platelet aggregation occurs. Adding the substance to platelets from a normal control group induces platelet aggregation. Which of the following substances is most likely to produce this effect? □ (A) Adenosine diphosphate □ (B) Antithrombin III □ (C) Fibronectin □ (D) Fibrinogen □ (E) Plasminogen □ (F) Thromboxane A2 □ (G) Von Willebrand factor

1 8 (E) The mitogen-activated protein (MAP) kinase cascade is involved in signaling from activation via cell surface receptors for growth factors. This pathway is particularly important for signaling of epidermal growth factor and fibroblast growth factor. Ligand binding, such as occurs with acetylcholine at a nerve-muscle junction, alters the conformation of ion channel receptors to allow flow of specific ions such as calcium into the cell, changing the electric potential across the cell membrane. Cyclic AMP is a "second messenger" that is typically activated via ligand binding to receptors with seven transmembrane segments that associate with GTP-hydrolyzing proteins; chemokine receptors function in this fashion. Cyclin-dependent kinases act within the nucleus. JAK/STAT pathways typically are recruited by cytokine receptors.

18 In an experiment, various soluble mediators are added to a cell culture containing epidermal cells to determine which of the mediators might be useful for promoting epidermal cell growth. When epidermal growth factor (EGF) is added, it binds to epidermal cell surface receptors, inducing RAS protein activation, with subsequent transcription factor translocation and DNA transcription activation. This effect in the epidermal cells is most likely to be mediated through which of the following intracellular pathways? □ (A) Calcium ion channel □ (B) Cyclic AMP □ (C) Cyclin-dependent kinase □ (D) JAK/STAT system □ (E) Mitogen-activated protein kinase

19 (A) Physicians must be cautious in assigning sex to an infant with ambiguous genitalia; changing one's opinion is about as popular as an umpire changing the call. True hermaphroditism, with ovarian and testicular tissue present, is rare. This infant has female pseudohermaphroditism, resulting from exposure of the fetus to excessive androgenic stimulation, which in this case is due to congenital adrenal hyperplasia. The gonadal and the karyotypic sex are female. Male pseudohermaphroditism has various forms, but the most common is testicular feminization, resulting from androgen insensitivity. Affected individuals are phenotypically females, but have testes and a 46,XY karyotype. Nondisjunctional events lead to monosomies or trisomies. Trinucleotide repeats are seen in males with fragile X syndrome. Abnormalities of mitochondrial DNA have a maternal transmission pattern and do not involve sex chromosomes or sexual characteristics.

19 A 23-year-old woman gives birth to a term infant after an uncomplicated pregnancy. On physical examination, the infant has ambiguous external genitalia. The parents want to know the infant's sex, but the physician is hesitant to assign a sex without further information. A chromosomal analysis indicates a karyotype of 46,XX. An abdominal CT scan shows bilaterally enlarged adrenal glands, and the internal genitalia appear to consist of uterus, fallopian tubes, and ovaries. This infant is most likely to have which of the following abnormalities? □ (A) Female pseudohermaphroditism □ (B) Testicular feminization □ (C) Nondisjunctional event with loss of Y chromosome □ (D) Excessive trinucleotide repeats □ (E) Mitochondrial DNA mutation

19 (C) This patient has a localized immune complex reaction (Arthus reaction), which activates and depletes complement C3 and C4. IgE concentration is increased in individuals with atopy and the potential for type I hypersensitivity. Although neutrophils are being recruited locally to the inflammatory reaction in this case, they are not depleted systemically, and they may be increased in the circulation. Skin tests are measures of type IV hypersensitivity when antigens such as tuberculin are used. CD4+ lymphocytes assist in various antibody-mediated and cell-mediated immune reactions, but their numbers in peripheral blood do not change appreciably.

19 A 29-year-old man has developed marked abdominal pain over the past week. On physical examination, there is diffuse abdominal tenderness with decreased bowel sounds, but no masses are noted. The stool is negative for occult blood. Laboratory studies show a serum creatinine level of 4.4 mg/dL and urea nitrogen level of 42 mg/dL. Microscopic examination of a renal biopsy specimen shows focal fibrinoid necrosis of the small arterial and arteriolar vascular media and intravascular microthrombi. Scattered neutrophils are seen in these areas of necrosis. Which of the following laboratory findings would most likely be present in this patient? □ (A) Increased IgE □ (B) Neutropenia □ (C) Decreased complement C3 □ (D) Tuberculin skin test positivity □ (E) CD4+ lymphocytosis

19 (F) Inflammation involving an epithelial surface may cause such extensive necrosis that the surface becomes eroded, forming an ulcer. If the inflammation continues, the ulcer can continue to penetrate downward into submucosa and muscularis. Alternatively, the ulcer may heal or may remain chronically inflamed. An abscess is a localized collection of neutrophils in tissues. A caseating granuloma is a granulomatous inflammation with central necrosis; the necrosis has elements of liquefaction and coagulative necrosis. Chronic inflammation occurs when there is a preponderance of mononuclear cells, such as lymphocytes, macrophages, and plasma cells, in a process that has gone on for more than a few days—more likely weeks or months—or that accompanies repeated bouts of acute inflammation. Pus, or a purulent exudate, appears semiliquid and yellowish because of the large numbers of granulocytes present. A serous effusion is a watery-appearing transudate that resembles an ultrafiltrate of blood plasma, with a low cell and protein content.

19 A 50-year-old man has experienced midabdominal pain for several weeks. He is afebrile. There is mild upper abdominal tenderness on palpation, and bowel sounds are present. Stool is positive for occult blood. An upper gastrointestinal endoscopy is performed, and biopsy specimens are taken. The figure shows microscopic examination of a biopsy specimen of a duodenal lesion. Which of the following terms best describes this lesion? □ (A) Abscess □ (B) Caseating granuloma □ (C) Chronic inflammation □ (D) Purulent exudate □ (E) Serous effusion □ (F) Ulceration

19 (A) If the existing cell damage is not great after myocardial infarction, the restoration of blood flow can help prevent further damage. The reperfusion of damaged cells results in generation of oxygen-derived free radicals, however, causing a reperfusion injury. The elevation in the CK level is indicative of myocardial cell necrosis because this intracellular enzyme does not leak in large quantities from intact cells. Myocardial fibers do not regenerate to a significant degree, and atrophic fibers would have less enzyme to release. t-PA does not produce a chemical injury; it induces thrombolysis to restore blood flow in blocked coronary arteries.

19 A 54-year-old man experienced onset of severe substernal chest pain over 3 hours. An ECG showed changes consistent with an acute myocardial infarction. After thrombolytic therapy with tissue plasminogen activator (t-PA), his serum creatine kinase (CK) level increased. Which of the following events most likely occurred after t-PA therapy? □ (A) Reperfusion injury □ (B) Cellular regeneration □ (C) Chemical injury □ (D) Increased synthesis of CK □ (E) Myofiber atrophy

19 (D) These transformed cells are designated iPS cells because they have been induced to become pleuripotent. This transformation process gets around the problem of using embryonic stem cells derived from manipulation of human embryos, which has ethical and religious concerns. Embryonic stem cells are totipotent, but they become pleuripotent cells that can further divide into many different cell lines, yet maintain themselves in a replicating pool. Thus, pleuripotent cells are the next best thing compared to embryonic cells for deriving human cells that could replace damaged or diseased tissues. Further differentiation of pleuripotent cells gives rise to cells with more restricted developmental capacity, such as mesenchymal stem cells that can give rise to tissues such as muscle and cartilage but not endodermal or ectodermal cells.

19 Dermal fibroblasts are harvested from the skin biopsy specimen of an adult man. These fibroblasts are transduced with genes encoding for transcription factors including Sox2 and c-myc. Under appropriate culture conditions these cells are then able to generate endodermal, mesodermal, and ectodermal cells. Into which of the following kinds of stem cell have these fibroblasts been transformed? □ (A) Embryonic □ (B) Lineage-committed □ (C) Mesenchymal □ (D) Pleuripotent

1 9 (E) Thrombomodulin is present on intact endothelium and binds thrombin, which inhibits coagulation by activating protein C. Calcium is a cofactor that assists clotting in the coagulation cascade (ethylenediaminetetraacetic acid [EDTA] in some blood collection tubes binds calcium to prevent clotting). Fibrin protein forms a meshwork that is essential to thrombus formation. Platelet factor 4 is released from the α granules of platelets and promotes platelet aggregation during the coagulation process. Prothrombin is converted to thrombin in the coagulation cascade. Tumor necrosis factor is not significantly involved in coagulation.

19 In an experiment, thrombus formation is studied in areas of vascular damage. After a thrombus forms in an area of vascular injury, the propagation of the thrombus to normal arteries is prevented. Researchers identify a substance that diminishes thrombus propagation by binding to thrombin, converting it from a procoagulant to an anticoagulant that activates protein C. Which of the following substances is most likely to produce this effect? □ (A) Calcium □ (B) Fibrin □ (C) Platelet factor 4 □ (D) Prothrombin □ (E) Thrombomodulin □ (F) Tumor necrosis factor

2 (E) The liver is one of the few organs in the human body that can partially regenerate. This is a form of compensatory hyperplasia. The stimuli to hepatocyte mitotic activity cease when the liver has attained its normal size. Apoptosis is single cell death and frequently occurs with viral hepatitis. Dysplasia is disordered epithelial cell growth that can be premalignant. Fatty change can lead to hepatomegaly; this is not a regenerative process, but is the result of toxic hepatocyte injury. Hydropic change, or cell swelling, does not produce regeneration. Hepatocytes can reenter the cell cycle and proliferate to regenerate the liver; they do not just increase in size.

2 A 16-year-old boy sustained blunt trauma to the abdomen when the vehicle he was driving struck a bridge abutment at high speed. Peritoneal lavage shows a hemoperitoneum, and at laparotomy, a small portion of the left lobe of the liver is removed because of the injury. Several weeks later, a CT scan of the abdomen shows that the liver has nearly regained its size before the injury. Which of the following processes best explains this CT scan finding? □ (A) Apoptosis □ (B) Dysplasia □ (C) Fatty change □ (D) Hydropic change □ (E) Hyperplasia □ (F) Hypertrophy □ (G) Metaplasia

22 (F) This child has features of X-linked agammaglobulinemia of Bruton. In this condition, B cell maturation stops after the rearrangement of heavy-chain genes, and light chains are not produced. Complete immunoglobulin molecules with heavy and light chains are not assembled and transported to the cell membrane. The lack of immunoglobulins predisposes the child to recurrent bacterial infections. Because T cell function remains intact, viral, fungal, and protozoal infections are uncommon. CD4+ and CD8+ lymphocytes differentiate from precursors in the thymus, which is not affected by the BTK gene mutation that gives rise to Bruton agammaglobulinemia. Follicular dendritic cells are a form of antigen-presenting cell that is not affected by B-cell and T-cell disorders. Monocytes may leave the circulation to become tissue macrophages, a process not dependent on B cell maturation. Natural killer (NK) cells are part of the innate immune system and respond to antibody coating abnormal cells—a process diminished by reduced antibody production—but the NK cells themselves are not directly affected by lack of immunoglobulin. Lack of stem cell differentiation is incompatible with life.

22 A 4-year-old boy has a history of recurrent sinopulmonary infections with Staphylococcus aureus and Streptococcus pneumoniae since age 17 months. He also developed an arthritis that cleared with immunoglobulin therapy. On physical examination, he is at the 30th percentile for height and weight. His temperature is 37.9°C. There is no lymphadenopathy, and lymph nodes are difficult to palpate. There is no hepatosplenomegaly. Laboratory studies show total serum protein of 5.1 g/dL and albumin of 4.6 g/dL. A lymph node biopsy specimen shows lymph nodes with rudimentary germinal centers. Over the next 10 years, the child develops arthralgias and erythematous skin rashes and has a positive ANA test result. Which of the following types of cells has most likely failed to differentiate to produce this patient's disease? □ (A) CD4+ lymphocyte □ (B) CD8+ lymphocyte □ (C) Follicular dendritic cell □ (D) Monocyte □ (E) Natural killer cell □ (F) Pre-B cell □ (G) Stem cell

2 (C) These findings are characteristic of Klinefelter syndrome, a relatively common chromosomal abnormality, which occurs in about 1 of 660 live-born males. The findings can be subtle. The 46,X,i(Xq) karyotype is a variant of Turner syndrome (seen only in females), caused by a defective second X chromosome. The 47,XYY karyotype occurs in about 1 in 1000 live-born males and is associated with taller-than-average stature. A person with a mosaic such as 46,XX/47,XX,+21 has milder features of Down syndrome than a person with the more typical 47,XX,+21 karyotype. The 22q11 deletion syndrome is associated with congenital defects affecting the palate, face, and heart and, in some cases, with T cell immunodeficiency.

2 A 27-year-old man comes to the physician for an infertility work-up. He and his wife have been trying to conceive a child for 6 years. Physical examination shows bilateral gynecomastia, reduced testicular size, reduced body hair, and increased length between the soles of his feet and the pubic bone. A semen analysis indicates oligospermia. Laboratory studies show increased follicle-stimulating hormone level and slightly decreased testosterone level. Which of the following karyotypes is most likely to be present in this man? □ (A) 46,X,i(Xq) □ (B) 47,XYY □ (C) 47,XXY □ (D) 46XX/47XX,+21 □ (E) 46,XY,del(22q11)

2 (B) This patient has clinical and serologic features of systemic lupus erythematosus (SLE). She also has a false-positive test result for syphilis, indicating the presence of anticardiolipin antibodies. These antibodies against phospholipid-protein complexes (antiphospholipid antibodies) also are called lupus anticoagulants because they interfere with in vitro clotting tests. In vivo, they are thrombogenic. Hence, these patients can have recurrent thrombosis. Lupus anticoagulants also can occur in the absence of SLE. The other listed options can occur in SLE, but they are not mediated by antiphospholipid antibodies.

2 A 30-year-old woman has had fever and arthralgia for the past 2 weeks. On physical examination, she has a temperature of 37.6°C and an erythematous malar rash. Initial laboratory studies are positive for ANAs at 1 : 1600 and anti-doublestranded DNA antibodies at 1 : 3200. Serum creatinine is markedly elevated, and serum complement levels are decreased. A VDRL test for syphilis is positive, and in vitro tests of coagulation (prothrombin time and partial thromboplastin time) are prolonged. Which of the following clinical features of her illness is most likely caused by antibodies that interfere with the coagulation test? □ (A) Arthritis □ (B) Recurrent thrombosis □ (C) Rash □ (D) Renal failure □ (E) Fever

2 (E) The elevated creatine kinase level indicates that myocardial necrosis has occurred. The destruction of myocardial fibers precludes complete resolution. The area of myocardial necrosis is gradually replaced by a fibrous scar. Liquefactive necrosis with abscess formation is not a feature of ischemic myocardial injury. Coagulative necrosis is typical of myocardial infarction, but after 1 month, a scar would be present. Nodular regeneration is typical of hepatocyte injury because hepatocytes are stable cells.

2 A 60-year-old woman developed chest pain that persisted for 4 hours. A radiographic imaging procedure showed an apparent myocardial infarction involving a 3 × 4 cm area of the posterior left ventricular free wall. Laboratory findings showed serum creatine kinase of 600 U/L. The patient received anti-arrhythmic and pressor agents to treat the decreased cardiac output while in the hospital. Which of the following pathologic findings would most likely be seen in the left ventricle 1 month later? □ (A) Abscess □ (B) Complete resolution □ (C) Coagulative necrosis □ (D) Nodular regeneration □ (E) Fibrous scar

2 (G) A thrombotic mass that forms on a cardiac valve (or, less commonly, on the cardiac mural endocardium) is known as a vegetation. Such vegetations may produce thromboemboli. Vegetations on the right-sided heart valves may embolize to the lungs; vegetations on the left embolize systemically to organs such as the brain, spleen, and kidney. A so-called paradoxical embolus occurs when a right-sided cardiac thrombus crosses a patent foramen ovale and enters the systemic arterial circulation. Patients with cancer may have a hypercoagulable state (e.g., Trousseau syndrome, with malignant neoplasms) that favors the development of arterial and venous thromboses. An adenocarcinoma is a malignant neoplasm that arises from glandular epithelium, forming a mass lesion; endocardial metastases are quite rare. Atheromas form in arteries and do not typically involve the cardiac valves. Chronic passive congestion refers to capillary, sinusoidal, or venous stasis of blood within an organ such as the lungs or liver. Mural thrombi are thrombi that form on the surfaces of the heart or large arteries. The term typically is reserved for large thrombi in a cardiac chamber or dilated aorta or large aortic branch; it is not used to describe thrombotic lesions on cardiac valves. A petechial hemorrhage is a grossly pinpoint hemorrhage. Phlebothrombosis occurs when stasis in large veins promotes thrombosis formation.

2 A 73-year-old man was diagnosed 1 year ago with pancreatic adenocarcinoma. He now sees his physician because of a transient ischemic attack. On auscultation of the chest, a heart murmur is heard. Echocardiography shows a 1-cm nodular lesion on the superior aspect of the anterior mitral valve leaflet. The valve leaflet appears to be intact. The blood culture is negative. Which of the following terms best describes this mitral valve lesion? □ (A) Adenocarcinoma □ (B) Atheroma □ (C) Chronic passive congestion □ (D) Mural thrombus □ (E) Petechial hemorrhage □ (F) Phlebothrombosis □ (G) Vegetation

2 (B) Chemokines include many molecules that are chemotactic for neutrophils, eosinophils, lymphocytes, monocytes, and basophils. Bradykinin causes pain and increased vascular permeability. Histamine causes vascular leakage, and prostaglandins have multiple actions, but they do not cause chemotaxis. Complement C3a causes increased vascular permeability by releasing histamine from mast cells.

2 An experiment introduces bacteria into a perfused tissue preparation. Leukocytes leave the vasculature and migrate to the site of bacterial inoculation. The movement of these leukocytes is most likely to be mediated by which of the following substances? □ (A) Bradykinin □ (B) Chemokines □ (C) Histamine □ (D) Prostaglandins □ (E) Complement C3a

23 (C) Embolization of the thrombus led to blockage of a renal arterial branch, causing an acute renal infarction in this patient. An ischemic injury to most internal organs produces a pattern of cell death called coagulative necrosis. Liquefactive necrosis occurs after ischemic injury to the brain and is the pattern seen with abscess formation. Caseous necrosis can be seen in various forms of granulomatous inflammation, typified by tuberculosis. Fat necrosis is usually seen in pancreatic and breast tissue. Gangrenous necrosis is a form of coagulative necrosis that usually results from ischemia and affects limbs.

23 A 63-year-old man has a 2-year history of worsening congestive heart failure. An echocardiogram shows mitral stenosis with left atrial dilation. A thrombus is present in the left atrium. One month later, he experiences left flank pain and notes hematuria. Laboratory testing shows elevated serum AST. Which of the following patterns of tissue injury is most likely to be present? □ (A) Liquefactive necrosis □ (B) Caseous necrosis □ (C) Coagulative necrosis □ (D) Fat necrosis □ (E) Gangrenous necrosis

20 (B) The findings listed suggest a severe inherited neurologic disease, and the pattern of inheritance (e.g., normal parents, an affected sibling) is consistent with an autosomal recessive disorder. This inheritance pattern and the cherry red spot in the retina are characteristic of Tay-Sachs disease, caused by mutations in the gene that encodes a lysosomal enzyme hexosaminidase A. Mitochondrial genes have a maternal pattern of transmission. Mutations in genes affecting receptor proteins and structural proteins typically give rise to an autosomal dominant pattern of inheritance. Genomic imprinting is characterized by a parent-of-origin effect.

20 A 1-year-old female infant is brought to the physician because of failure to thrive, poor neurologic development, and poor motor function. Physical examination shows a "cherry red" spot on the macula of the retina. The infant's muscle tone is poor. Both parents and a brother and sister are healthy, with no apparent abnormalities. One brother with a similar condition died at the age of 18 months. This genetic disorder most likely resulted from which of the following underlying abnormalities? □ (A) Mutation in a mitochondrial gene □ (B) Mutation in a gene encoding a lysosomal enzyme □ (C) Mutation in a gene encoding a receptor protein □ (D) Mutation in a gene encoding a structural protein □ (E) Genomic imprinting

20 (D) Loss of the nucleus results in cell death. All other cellular morphologic changes listed represent reversible cellular injury. The plasma membrane and intracellular organelles remain functional unless severe damage causes loss of membrane integrity.

20 A 33-year-old woman has had increasing lethargy and decreased urine output for the past week. Laboratory studies show serum creatinine level of 4.3 mg/dL and urea nitrogen level of 40 mg/dL. A renal biopsy is performed, and the specimen is examined using electron microscopy. Which of the following morphologic changes most likely suggests a diagnosis of acute tubular necrosis? □ (A) Mitochondrial swelling □ (B) Plasma membrane blebs □ (C) Chromatin clumping □ (D) Nuclear fragmentation □ (E) Ribosomal disaggregation

2 0 (C) In chronic inflammatory conditions such as rheumatoid arthritis, the serum amyloid-associated (SAA) precursor protein forms the major amyloid fibril protein AA. Amyloid is deposited in interstitial locations, not intracellularly. The P component is a minor component of the amyloid. All amyloid shows characteristic "apple-green" birefringence under polarized light microscopy after Congo red staining—anything else would not be amyloid. Amyloid derived from light chains in association with multiple myeloma has AL fibrils.

20 A 63-year-old man has had chronic arthritis for the past 15 years. Physical examination shows ulnar deviation with bony ankylosis producing swan-neck deformities of the fingers. Laboratory studies show 4.2 g of protein in a 24-hour urine collection, serum creatinine of 3.1 g/dL, and urea nitrogen of 3 g/dL. A rectal biopsy is performed, which shows deposition of amorphous pink material in the mucosa. The material stains positive with Congo red. Which of the following best describes this material in the mucosa? □ (A) It is within the cytoplasm □ (B) It contains greater than 50% P component □ (C) It is derived from an acute-phase reactant □ (D) It does not show birefringence after Congo red staining □ (E) It is derived from λ light chains

20 (C) The formation of an exudate containing a significant amount of protein and cells depends on the "leakiness" of blood vessels, principally venules. The extravascular colloid osmotic pressure increases when exudation has occurred, and the protein content of the extravascular space increases, causing extracellular fluid accumulation. The lymphatics scavenge exuded fluid with protein and reduce the amount of extravascular and extracellular fluid. Sodium and water retention helps drive transudation of fluid. Leukocytosis alone is insufficient for exudation because the leukocytes must be driven to emigrate from the vessels by chemotactic factors.

20 A 72-year-old man with severe emphysema has had worsening right ventricular failure for the past 5 years. For the past 4 days, he has had fever and increasing dyspnea. A chest radiograph shows an accumulation of fluid in the pleural spaces. Fluid obtained by thoracentesis has a specific gravity of 1.030 and contains degenerating neutrophils. The most likely cause of this fluid accumulation is an increase in which of the following mechanisms? □ (A) Colloid osmotic pressure □ (B) Lymphatic pressure □ (C) Vascular permeability □ (D) Renal retention of sodium and water □ (E) Leukocytic diapedesis

20 (B) The patient has septic shock from infection with gram-negative organisms that have lipopolysaccharide, which binds to cells via Toll-like receptor to initiate release of various cytokines such as tumor necrosis factor and interleukin-1 that produce fever. Macrophages are stimulated to destroy the organisms. Nitric oxide is released, promoting vasodilation and circulatory collapse. Complement C3b generated by bacteria via the alternative pathway acts as an opsonin. Plateletactivating factor mediates many features of acute inflammation and in large quantities can cause vasoconstriction and bronchoconstriction. Toxic shock syndrome toxin-1 is a superantigen released by staphylococcal organisms that is a potent activator of T lymphocytes, inducing cytokine release with septic shock.

20 A 77-year-old woman with dysuria for 1 week now has a fever. On examination, she has a temperature of 37.9°C, pulse of 103/min, and blood pressure of 80/40 mm Hg. There is right flank pain. A urinalysis shows numerous white blood cells. Her plasma lactate is increased. Urine culture and blood culture grow Escherichia coli. Which of the following is most likely mediating her cardiovascular collapse through attachment to Toll-like cell receptors? □ (A) Complement C3b □ (B) Lipopolysaccharide □ (C) Nitric oxide □ (D) Platelet-activating factor □ (E) Toxic shock syndrome toxin-1 □ (F) Tumor necrosis factor

2 2 (B) This child has von Gierke disease. Because of the deficiency of glucose-6-phosphatase, glycogen is not metabolized readily to glucose. Affected individuals have severe hypoglycemia, which leads to convulsions. Intracytoplasmic accumulations of glycogen occur mainly in the liver and kidney. Another form of glycogen storage disease, McArdle syndrome, results from a deficiency of muscle phosphorylase and leads to muscle cramping. Tay-Sachs disease is characterized by a deficiency in hexosaminidase A and results in severe neurologic deterioration. In Hurler syndrome, the enzyme α-l-iduronidase is deficient. Affected children have skeletal deformities and a buildup of mucopolysaccharides in endocardium and coronary arteries, leading to heart failure. Cardiomegaly and heart failure mark Pompe disease, another form of glycogen storage disease.

22 A 2-year-old child is brought to the physician after having convulsions. The child has a history of failure to thrive. Physical examination shows hepatomegaly and ecchymoses of the skin. Laboratory studies show a blood glucose level of 31 mg/dL. A liver biopsy specimen shows cells filled with clear vacuoles that stain positive for glycogen. Which of the following conditions is most likely to produce these findings? □ (A) McArdle syndrome □ (B) Von Gierke disease □ (C) Tay-Sachs disease □ (D) Hurler syndrome □ (E) Pompe disease

2 1 (B) This child has an immunodeficiency characterized by infection, a small thymus, congenital malformations, and hypoparathyroidism. This cluster is characteristic of the 22q11.2 deletion syndrome, readily diagnosed by FISH. HIV infection can lead to AIDS, but no congenital anomalies are associated with this condition. Trinucleotide repeats of the X chromosome are seen in fragile X syndrome, which manifests with mental retardation in males. Adenosine deaminase deficiency can cause immunodeficiency, but it is not associated with congenital malformations. A lymph node biopsy may show a reduction in T cells or B cells associated with various forms of immunodeficiency, but this is not a specific test that can aid in confirming a specific diagnosis.

21 A 10-year-old boy is brought to the physician because of a cough and earache. The child has a history of recurrent infections, including otitis media, diarrhea, and pneumonia. Physical examination shows an erythematous right tympanic membrane, a cleft palate, and murmur suggestive of congenital heart disease. A thoracic CT scan shows a small thymus. Results of laboratory studies suggest mild hypoparathyroidism. Which of the following diagnostic studies is most likely to be helpful in diagnosing this patient's condition? □ (A) Branched DNA assay for HIV-1 RNA level □ (B) FISH analysis with a probe for chromosome 22q11.2 □ (C) PCR analysis for trinucleotide repeats on the X chromosome □ (D) Adenosine deaminase assay in lymphocytes □ (E) Lymph node biopsy

21 (B) This histologic picture is typical of apoptosis produced by chemotherapeutic agents. The release of cytochrome from the mitochondria is a key step in many forms of apoptosis, and it leads to the activation of caspases. BCL-2 is an antiapoptotic protein that prevents cytochrome c release and prevents caspase activation. Lipofuscin is a pigmented residue representing undigested cellular organelles in autophagic vacuoles. Catalase is a scavenger of hydrogen peroxide. Phospholipases are activated during necrosis and cause cell membrane damage.

21 A 40-year-old man had undifferentiated carcinoma of the lung. Despite chemotherapy, the man died of widespread metastases. At autopsy, tumors were found in many organs. Histologic examination showed many foci in which individual tumor cells appeared shrunken and deeply eosinophilic. Their nuclei exhibited condensed aggregates of chromatin under the nuclear membrane. The process affecting these shrunken tumor cells was most likely triggered by the release of which of the following substances into the cytosol? □ (A) Lipofuscin □ (B) Cytochrome c □ (C) Catalase □ (D) Phospholipase □ (E) BCL-2

21 (B) The figure shows a granuloma with many epithelioid cells and prominent large Langhans giant cells. Macrophage stimulation and transformation to epithelioid cells and giant cells are characteristic of granuloma formation. Interferon-γ promotes the formation of epithelioid cells and giant cells. Complement C5a is chemotactic for neutrophils. Although occasional neutrophils are seen in granulomas, neutrophils do not form a major component of granulomatous inflammation. Bradykinin, released in acute inflammatory responses, results in pain. Macrophages can release nitric oxide to destroy other cells, but nitric oxide does not stimulate macrophages to form a granulomatous response. Prostaglandins are mainly involved in the causation of vasodilation and pain in acute inflammatory responses.

21 A 43-year-old man has had a cough and fever for the past 2 months. A chest radiograph shows bilateral nodular densities, some with calcification, located mainly in the upper lobes of the lungs. A transbronchial lung biopsy is performed, yielding a specimen with the microscopic appearance shown in the figure. Which of the following chemical mediators is most important in the pathogenesis of this lesion? □ (A) Complement C5a □ (B) Interferon-γ □ (C) Bradykinin □ (D) Nitric oxide □ (E) Prostaglandin

21 (B) This patient has a drug-induced systemic lupus erythematosus (SLE)-like condition. Drugs such as procainamide, hydralazine, and isoniazid can cause this condition. Test results for ANA are often positive, but test results for anti-doublestranded DNA are negative. Antihistone antibodies are present in many cases. Characteristic signs and symptoms of SLE are often lacking, and renal involvement is uncommon. Remission occurs when the patient stops taking the drug. Anti-Sm antibody shows specificity for SLE. Anti-Jo-1 antibody has specificity for polymyositis/dermatomyositis. Anti-U1- ribonucleotide protein has specificity for mixed connective tissue disease. Anticentromeric antibody is most likely to be present with limited scleroderma (CREST syndrome). Anti-SS-A antibody is most characteristic of Sjögren syndrome.

21 A 60-year-old woman who undergoes a routine health screening examination has a blood pressure of 155/95 mm Hg. She receives antihypertensive therapy that includes hydralazine. Four months later, she develops arthralgias, myalgias, and a malar erythematous rash. Laboratory findings include an ANA titer of 1 : 2560 in a diffuse pattern. Anti-doublestranded DNA antibodies are not present. Which of the following autoantibodies has the greatest specificity for her condition? □ (A) Anti-Sm □ (B) Antihistone □ (C) Anti-Jo-1 □ (D) Anti-U1-ribonucleoprotein □ (E) Anticentromere □ (F) Anti-SS-A

21 (E) The figure shows a mural thrombus filling an atherosclerotic aortic aneurysm below the renal arteries. Diabetes mellitus accelerates and worsens atherosclerosis. One of the complications of mural thrombosis is embolization, which occurs when a small piece of the clot breaks off. The embolus is carried distally and may occlude the popliteal artery. Because the thrombus is in the arterial circulation, an embolus would not travel to the lungs. A venous thrombus produces leg swelling from edema. Although platelets contribute to the formation of thrombi, the platelet count does not decrease appreciably with formation of a localized thrombus, and a generalized process such as disseminated intravascular coagulation is needed to consume enough platelets to cause thrombocytopenia.

21 A 70-year-old man with a history of diabetes mellitus died of an acute myocardial infarction. At autopsy, the aorta, opened longitudinally, appeared as shown in the figure. Which of the following complications associated with aortic disease would most likely have been present during his life? □ (A) Renal infarction □ (B) Pulmonary thromboembolism □ (C) Edema of the left leg □ (D) Thrombocytopenia □ (E) Popliteal arterial occlusion

22 (B) This acute inflammatory process leads to production of acute-phase reactants, such as C-reactive protein (CRP), fibrinogen, and serum amyloid A (SAA) protein. These proteins, particularly fibrinogen, and immunoglobulins increase red blood cell rouleaux formation to increase sedimentation, which is a nonspecific indicator of inflammation. CRP production is up-regulated by interleukin-6, whereas fibrinogen and SAA are up-regulated mainly by tumor necrosis factor and interleukin-1. Interferon-γ is a potent stimulator of macrophages. Nitric oxide can induce vasodilation or can assist in microbial killing within macrophages. Prostaglandins are vasodilators.

22 A 43-year-old man with a ventricular septal defect has had a cough and fever for the past 2 days. On examination, he has a temperature of 37.6°C and a cardiac murmur. A blood culture grows Streptococcus, viridans group. His erythrocyte sedimentation rate (ESR) is increased. Microbial cells are opsonized and cleared. Which of the following chemical mediators is most important in producing these findings? □ (A) Bradykinin □ (B) C-reactive protein □ (C) Interferon-γ □ (D) Nitric oxide □ (E) Prostaglandin □ (F) Tumor necrosis factor

22 (A) The figure shows a hemorrhagic infarct on the pleura, a typical finding when a medium-sized thromboembolus lodges in a pulmonary artery branch. The infarct is hemorrhagic because the bronchial arterial circulation in the lung (derived from the systemic arterial circulation and separate from the pulmonary arterial circulation) continues to supply a small amount of blood to the affected area of infarction. Passive congestion, whether acute or chronic, is a diffuse process, as is edema, which does not impart a red color. Pulmonary venous thrombosis is rare.

22 A 49-year-old man with congestive heart failure develops Streptococcus pneumoniae after a bout of influenza. After recuperating for 2 weeks, he develops pleuritic chest pain. The pain is caused by the development of the lesion shown in the figure. Which of the following events has most likely occurred? □ (A) Pulmonary infarction □ (B) Chronic pulmonary congestion □ (C) Pulmonary edema □ (D) Acute pulmonary congestion □ (E) Pulmonary venous thrombosis

2 2 (B) The microscopic findings suggest metastatic calcification, with deposition of calcium salts in tissues that have physiologic mechanisms for losing acid, creating an internal alkaline environment that favors calcium precipitation. Hypercalcemia can have a variety of causes, including hyperparathyroidism, bone destruction secondary to metastases, paraneoplastic syndromes, and, less commonly, vitamin D toxicity or sarcoidosis. Chronic renal disease reduces phosphate excretion by the kidney, resulting in an increase in serum phosphate. Because the solubility product of calcium and phosphorus must be maintained, the serum calcium is depressed, triggering increased parathyroid hormone output to increase the calcium level, which promotes calcium deposition. Chronic hepatitis leads to hyperbilirubinemia and jaundice. The granulomas of tuberculosis have caseous necrosis with dystrophic calcification. Another form of dystrophic calcification occurs when atherosclerotic lesions calcify. Dystrophic calcification is seen more often in the elderly, but it is the result of a lifetime of pathologic changes, not aging itself. Pulmonary emphysema can lead to respiratory acidosis that is compensated by metabolic alkalosis, with the result that the serum calcium level remains relatively unchanged.

22 A 70-year-old man died suddenly. At autopsy, multiple tissue sites were sampled for microscopic analysis. Examination of the tissues showed noncrystalline amorphous deposits of calcium salts in gastric mucosa, renal interstitium, and alveolar walls of lungs. Which of the following conditions would most likely explain these findings? □ (A) Chronic hepatitis □ (B) Chronic glomerulonephritis □ (C) Disseminated tuberculosis □ (D) Generalized atherosclerosis □ (E) Normal aging process □ (F) Pulmonary emphysema

23 (C) This patient has severe combined immunodeficiency (SCID). Because the T-cell and B-cell arms of the immune system are deficient, there are severe and recurrent infections with bacteria, viruses, and fungi. With the family history of males being affected, the patient most likely has X-linked SCID. This form results from mutations in the common γ chain that is a part of many cytokine receptors, such as interleukin (IL)-2, IL-4, IL-7, and IL-15. These cytokines are needed for normal B-cell and T-cell development. The marked lymphoid hypoplasia is not typical of HIV infection. Loss of chromosome 22q11 is seen in DiGeorge syndrome. BTK gene mutations give rise to Bruton agammaglobulinemia. Mutation in the CD40 ligand is responsible for hyper-IgM syndrome.

23 A 3-month-old boy has had recurrent infections of the respiratory, gastrointestinal, and urinary tracts since birth. The infectious agents have included Candida albicans, Pneumocystis carinii, Pseudomonas aeruginosa, and cytomegalovirus. Despite intensive treatment with antibiotics and antifungal drugs, he dies at age 5 months. At autopsy, lymph nodes are small with very few lymphocytes and no germinal centers. The thymus, Peyer patches, and tonsils are hypoplastic. There is a family history of other males with similar findings. Which of the following immunological alterations best describes the abnormality that caused this patient's illness? □ (A) Maternal HIV infection □ (B) Loss of chromosome 22q11 □ (C) Mutation in the common γ chain of cytokine receptors □ (D) Mutation in the BTK gene □ (E) Mutation in CD40 ligand

2 3 (F) The figure shows an organizing thrombus in a small artery, with several small recanalized channels. Such a peripheral arterial occlusion was insufficient to produce infarction, as evidenced by the lack of enzyme elevation. Thrombi become organized over time if they are not dissolved by fibrinolytic activity. Air emboli are uncommon and usually the result of trauma. Air emboli on the arterial side can cause ischemia through occlusion even when very small, whereas on the venous side, more than 100 mL of air trapped in the heart may reduce cardiac output. Air emboli from decompression form when gases that became dissolved in tissues at high pressure bubble out at lower pressure in blood and tissues. Cholesterol emboli can break off from atheromas in arteries and proceed distally to occlude small arteries; however, because these emboli are usually quite small, they are seldom clinically significant. Chronic passive congestion refers to capillary, sinusoidal, or venous stasis of blood within an organ such as the lungs or liver. Fat emboli are globules of lipid that are most likely to form after traumatic injury, typically to long bones. Mural thrombi are thrombi that form on the surfaces of the heart or large arteries. After a thrombus has formed, it may become organized with ingrowth of capillaries, fibroblast proliferation, and macrophage infiltration, which eventually clears part or most of the clot, forming one or more new lumens (recanalization).

23 A 58-year-old man with hyperlipidemia and severe atherosclerosis has had anginal pain for the past 24 hours. Laboratory findings show no increase in serum troponin I or creatine kinase-MB. The patient is in stable condition 2 weeks later and has no chest pain, but a small artery in the epicardium has undergone the changes seen in the figure. Which of the following terms best describes this finding in the epicardial artery? □ (A) Air embolus □ (B) Cholesterol embolus □ (C) Chronic passive congestion □ (D) Fat embolus □ (E) Mural thrombus □ (F) Organization with recanalization □ (G) Phlebothrombosis

9 (D) The high lipid content of central nervous system tissues results in liquefactive necrosis as a consequence of ischemic injury, as in this case of a "stroke." Fat necrosis is seen in breast and pancreatic tissues. Coagulative necrosis is the typical result of ischemia in most solid organs. Apoptosis affects single cells and typically is not grossly visible. Karyolysis refers to fading away of cell nuclei in dead cells.

9 A 68-year-old woman suddenly lost consciousness; on awakening 1 hour later, she could not speak or move her right arm and leg. Two months later, a head CT scan showed a large cystic area in the left parietal lobe. Which of the following pathologic processes has most likely occurred in the brain? □ (A) Fat necrosis □ (B) Coagulative necrosis □ (C) Apoptosis □ (D) Liquefactive necrosis □ (E) Karyolysis

23 (E) This girl has features of hemophilia A. This X-linked recessive condition is expected to occur in males, however, who inherit the one maternal X chromosome with the genetic mutation, and they do not have another X chromosome with a normal functional allele, as is the case in her brother. Hemophilia in a female can be explained by the Lyon hypothesis, which states that only one X chromosome in a female is active (the "turned off" X chromosome is the Barr body) for most genes, but this inactivation is a random event. Some individuals are out on the tail end of the Poisson distribution of random events and have few active X chromosomes with the normal allele, leading to markedly diminished factor VIII levels.

23 An 8-year-old girl is brought to the emergency department after experiencing sudden severe dyspnea (SOB). On examination, she has upper airway obstruction from soft tissue swelling in her neck. A radiograph shows a hematoma compressing the trachea. Laboratory studies show her prothrombin time is normal, but her partial thromboplastin time is increased. Further testing reveals less than 1% of normal factor VIII level. Both parents and two female siblings are unaffected by this problem, but a male sibling has experienced a similar episode. Which of the following genetic abnormalities is most likely to account for the findings in this girl? □ (A) Autosomal dominant mutation □ (B) Autosomal recessive mutation □ (C) Genomic imprinting □ (D) Germline mosaicism □ (E) Random X inactivation □ (F) Spontaneous new mutation

23 (C) Interferon-γ secreted from lymphocytes stimulates monocytes and macrophages, which secrete their own cytokines that further activate lymphocytes. Interferon-γ also is important in transforming macrophages into epithelioid cells in a granulomatous inflammatory response. Leukotriene B4, generated in the lipoxygenase pathway of arachidonic acid metabolism, is a potent neutrophil chemotactic factor. Histamine released from mast cells is a potent vasodilator, increasing vascular permeability. Interleukin-1 and tumor necrosis factor, both produced by activated macrophages, mediate many systemic effects, including fever, metabolic wasting, and hypotension. Nitric oxide generated by macrophages aids in destruction of microorganisms; nitric oxide released from endothelium mediates vasodilation and inhibits platelet activation. Binding of agonists such as epinephrine, collagen, or thrombin to platelet surface receptors activates phospholipase C, which catalyzes the release of arachidonic acid from two of the major membrane phospholipids, phosphatidylinositol and phosphatidylcholine.

23 In an experiment, peripheral blood T lymphocytes are collected and placed in a medium that preserves their function. The lymphocytes are activated by contact with antigen and incubated for several hours. The supernatant fluid is collected and is found to contain a substance that is a major stimulator of monocytes and macrophages. Which of the following substances is most likely to stimulate these cells? □ (A) Leukotriene B4 □ (B) Histamine □ (C) Interferon-γ □ (D) Interleukin-1 □ (E) Nitric oxide □ (F) Phospholipase C □ (G) Tumor necrosis factor

2 4 (C) This patient has features of myasthenia gravis, a form of type II hypersensitivity reaction in which antibody is directed against cell surface receptors. Antibodies to acetylcholine receptors impair the function of skeletal muscle motor end plates. Antibodies are produced by B cells, and macrophages are not a significant part of this hypersensitivity reaction; there is little or no inflammation of the muscle in myasthenia gravis. Muscle lysis by CD8+ T cells occurs in polymyositis. Immune complex-mediated injury is a feature of dermatomyositis. Delayed hypersensitivity reactions are more likely in parasitic infestations of muscles.

24 A 34-year-old woman has experienced increasing muscular weakness over the past 5 months. This weakness is most pronounced in muscles that are used extensively, such as the levator palpebrae of the eyelids, causing her to have difficulty with vision by the end of the day. After a night's sleep, her symptoms have lessened. On physical examination, she is afebrile. No skin rashes are noted. Muscle strength is 5/5 initially, but diminishes with repetitive movement. Which of the following is the most likely mechanism for muscle weakness in this patient? □ (A) Secretion of cytokines by activated macrophages □ (B) Lysis of muscle cells by CD8+ lymphocytes □ (C) Antibody-mediated dysfunction of neuromuscular junction □ (D) Immune complex deposition in muscle capillaries □ (E) Delayed hypersensitivity reaction against muscle antigens

24 (A) The patient has a subdural hematoma. A hematoma is a collection of blood in a potential space or within tissue. Purpura denotes blotchy hemorrhage on skin, serosal surfaces, or mucous membrane surfaces; areas larger than 1 to 2 cm are often called ecchymoses. Congestion occurs when there is vascular dilation with pooling of blood within an organ. Petechiae are pinpoint areas of hemorrhage.

24 A 78-year-old woman falls in the bathtub and strikes the back of her head. Over the next 24 hours, she becomes increasingly somnolent. A head CT scan shows an accumulation of fluid beneath the dura, compressing the left cerebral hemisphere. Which of the following terms best describes this collection of fluid? □ (A) Hematoma □ (B) Purpura □ (C) Congestion □ (D) Petechia □ (E) Ecchymosis

24 (D) The formation of a fluid-filled cavity after infection with Staphylococcus aureus suggests that liquefactive necrosis has occurred. The cavity is filled with tissue debris and viable and dead neutrophils (pus). Localized, pus-filled cavities are Robbins & Cotran Review of Pathology Pg. 28 called abscesses. Some bacterial organisms, such as S. aureus, are more likely to be pyogenic, or pus-forming. With complete resolution, the structure of the lung remains almost unaltered. Lung tissue, in contrast to liver, is incapable of regeneration. Scarring or fibrosis may follow acute inflammation as the damaged tissue is replaced by fibrous connective tissue. Most bacterial pneumonias resolve, and progression to continued chronic inflammation is uncommon.

24 A 90-year-old woman is diagnosed with Staphylococcus aureus pneumonia and receives a course of antibiotic therapy. Two weeks later, she no longer has a productive cough, but she still has a temperature of 38.1°C. A chest radiograph shows a 3-cm rounded density in the right lower lobe of the lung whose liquefied contents form a central air-fluid level. There are no surrounding infiltrates. Which of the following terms best describes the outcome of the patient's pneumonia? □ (A) Complete resolution □ (B) Regeneration □ (C) Fibrosis □ (D) Abscess formation □ (E) Progression to chronic inflammation

24 (C) The appearance of lipid vacuoles in many of the hepatocytes is characteristic of fatty change (steatosis) of the liver. Abnormalities in lipoprotein metabolism can lead to steatosis. Alcohol is a hepatotoxin that produces hepatic steatosis. Decreased serum albumin levels and increased prothrombin time suggest alcohol-induced hepatocyte damage. Substance abuse with heroin produces few organ-specific pathologic findings. Exercise has little direct effect on hepatic function. Smoking directly damages lung tissue, but has no direct effect on the liver. Aspirin has a significant effect on platelet function, but not on hepatocytes.

24 At autopsy, a 40-year-old man has an enlarged (2200 g) liver with a yellow cut surface. The microscopic appearance of this liver is shown in the figure. Before death, the man's total serum cholesterol and triglyceride levels were normal, but he had a decreased serum albumin concentration and increased prothrombin time. Which of the following activities most likely led to these findings? □ (A) Injecting heroin □ (B) Playing basketball □ (C) Drinking beer □ (D) Smoking cigarettes □ (E) Ingesting aspirin

2 4 (B) The appearance of multiple siblings with a similar condition known to be autosomal dominant, such as osteogenesis imperfecta type I, when both parents are phenotypically normal suggests that one parent has an autosomaldominant mutation confined to gonadal germ cells, so the affected parent is a mosaic, and somatic cells do not carry the mutation. Genomic imprinting in uniparental disomy is a feature of Angelman syndrome. Multifactorial inheritance is a feature of diseases without a defined inheritance pattern, not osteogenesis imperfecta. Random X inactivation may account for females exhibiting features of an X-linked recessive condition, such as hemophilia A, but this is unlikely to occur three times in the same generation. A spontaneous new mutation can account for one child having an autosomaldominant condition that a parent does not, but the rarity of this event makes three such events in a generation highly unlikely.

24 Three female children in a family with the pedigree shown in the figure are noted to have histories of multiple fractures along with dental problems and hearing impairment. On examination, they are of normal height and weight for age, but have steel gray to blue sclerae. Both parents are unaffected by these abnormalities. Which of the following genetic abnormalities is most likely to account for the findings in these children? □ (A) Genomic imprinting □ (B) Germline mosaicism □ (C) Multifactorial inheritance □ (D) Random X inactivation □ (E) Spontaneous new mutation

2 5 (B) The clinical features of this child—neurologic involvement, hepatosplenomegaly, and accumulation of foamy macrophages—suggest a lysosomal storage disorder. One such disorder, with which the clinical history is quite compatible, is Niemann-Pick disease type A. It is characterized by lysosomal accumulation of sphingomyelin owing to a severe deficiency of sphingomyelinase. In familial hypercholesterolemia, there are fewer LDL receptors on hepatocytes, leading to early and accelerated atherosclerosis by young adulthood. Collagen synthesis is impaired in individuals with Ehlers-Danlos syndrome. The glycogen storage disease known as von Gierke disease results from glucose-6- phosphatase deficiency. Globules of α1-antitrypsin are seen in the liver cells of individuals with inherited deficiency of α1- antitrypsin.

25 A 22-year-old woman delivers an apparently healthy female infant after an uncomplicated pregnancy. At 4 years of age, the girl is brought to the physician because her parents have observed progressive, severe neurologic deterioration. Physical examination shows marked hepatosplenomegaly. A bone marrow biopsy specimen shows numerous foamy vacuolated macrophages. Analysis of which of the following factors is most likely to aid in the diagnosis of this condition? □ (A) Number of LDL receptors on hepatocytes □ (B) Level of sphingomyelinase in splenic macrophages □ (C) Rate of synthesis of collagen □ (D) Level of glucose-6-phosphatase in liver cells □ (E) Level of α1-antitrypsin in the liver

25 (A) This cell is shrunken and has been converted into a dense eosinophilic mass. The surrounding cells are normal, and there is no inflammatory reaction. This pattern is typical of apoptosis. Caspase activation is a universal feature of apoptosis, regardless of the initiating cause. Apoptosis induced in recipient cells from donor lymphocytes occurs with graftversus- host disease. Reduced ATP synthesis and increased glycolysis occur when a cell is subjected to anoxia. These changes are reversible. Lipases are activated in enzymatic fat necrosis. Lipid peroxidation occurs when the cell is injured by free radicals.

25 A 22-year-old woman with leukemia undergoes bone marrow transplantation and receives partially mismatched donor marrow. One month later, she has a scaling skin rash. Examination of a skin biopsy specimen reveals the cellular change shown in the figure. This change most likely results from which of the following biochemical reactions? □ (A) Activation of caspases □ (B) Reduction of ATP synthesis □ (C) Increase in glycolysis □ (D) Activation of lipases □ (E) Lipid peroxidation

25 (B) The liver has a dual blood supply, with a hepatic arterial circulation and a portal venous circulation. Infarction of the liver caused by occlusion of hepatic artery is uncommon. Cerebral infarction typically produces liquefactive necrosis. Infarcts of most solid parenchymal organs such as the kidney, heart, and spleen exhibit coagulative necrosis, and emboli from the left heart often go to these organs.

25 A 28-year-old woman with a 15-year history of recurrent thrombosis from a prothrombin gene mutation develops septicemia after a urinary tract infection with Pseudomonas aeruginosa. Despite aggressive therapy, she dies of multiple organ failure. At autopsy, which of the following organs is most likely to be spared from the effects of ischemic injury? □ (A) Brain □ (B) Liver □ (C) Kidney □ (D) Heart □ (E) Spleen

4 (C) Glucocorticoids inhibit wound healing by impairing collagen synthesis. This is a desirable side effect if the amount of scarring is to be reduced, but it results in the delayed healing of surgical wounds. Re-epithelialization, in part driven by epidermal growth factor, is not affected by corticosteroid therapy. Angiogenesis driven by fibroblast growth factor is not significantly affected by corticosteroids. Serine proteinases are important in wound remodeling. Neutrophil infiltration is not prevented by glucocorticoids.

4 A 23-year-old woman receiving corticosteroid therapy for an autoimmune disease has an abscess on her upper outer right arm. She undergoes minor surgery to incise and drain the abscess, but the wound heals poorly over the next month. Which of the following aspects of wound healing is most likely to be deficient in this patient? □ (A) Re-epithelialization □ (B) Fibroblast growth factor elaboration □ (C) Collagen deposition □ (D) Serine proteinase production □ (E) Neutrophil infiltration

2 5 (E) This patient has infective endocarditis with septic embolization, producing a cerebral abscess. The tissue destruction that accompanies abscess formation as part of acute inflammatory processes occurs from lysosomal enzymatic destruction, aided by release of reactive oxygen species. Nitric oxide generated by macrophages aids in destruction of infectious agents. Interferon-γ released from lymphocytes plays a major role in chronic and granulomatous inflammatory responses. Transforming growth factor-β formed by macrophages promotes fibrosis. Prostaglandins produced by endothelium promote vasodilation.

25 A 30-year-old woman with a history of a congenital ventricular septal defect has had a persistent temperature of 38.6°C and headache for the past 3 weeks. A head CT scan shows an enhancing 3-cm, ring-like lesion in the right parietal lobe. Which of the following actions by inflammatory cells most likely produces this CT finding? □ (A) Formation of nitric oxide by macrophages □ (B) Production of interferon-γ by lymphocytes □ (C) Formation of transforming growth factor-β by macrophages □ (D) Generation of prostaglandin by endothelium □ (E) Release of lysosomal enzymes from neutrophils

2 5 (E) This patient has diffuse systemic sclerosis (scleroderma). The small arteries of the kidney are damaged by a hyperplastic arteriolosclerosis that can be complicated by very high blood pressure and renal failure. Meningitis and adrenal failure are not typical features of autoimmune diseases. Glomerulonephritis is a more typical complication of systemic lupus erythematosus. With scleroderma, the gastrointestinal tract undergoes fibrosis, without any tendency to perforation or ulceration.

25 A 45-year-old woman has experienced difficulty in swallowing that has increased in severity over the past year. She also has experienced malabsorption, shown by a 5-kg weight loss in the past 6 months. She reports increasing dyspnea during this time. On physical examination, her temperature is 36.9°C, pulse is 66/min, respirations are 18/min, and blood pressure is 145/90 mm Hg. Echocardiography shows a large pericardial effusion. The ANA test result is positive at 1 : 512 with a nucleolar pattern. Which of the following serious complications of the patient's underlying autoimmune disease is most likely to occur? □ (A) Meningitis □ (B) Glomerulonephritis □ (C) Perforated duodenal ulcer □ (D) Adrenal failure □ (E) Malignant hypertension

26 (E) The findings listed are characteristic of Turner syndrome (monosomy X), which accounts for many first-trimester fetal losses. The hygroma is quite suggestive of this disorder. Fetuses with this finding are rarely live-born. Trisomy 18 can be marked by multiple anomalies, but overlapping fingers and a short neck are more typical features. The presence of additional X chromosomes may not cause serious fetal anomalies because all but one X chromosome is inactive. Down syndrome (47,XX,+21) may be accompanied by a hygroma and hydrops, but ventricular septal defect is more frequent than coarctation, and horseshoe kidney is uncommon. The 47,XXY karyotype (Klinefelter syndrome) does not result in stillbirth, and these males have no major congenital defects. Triploidy with 69 chromosomes typically leads to fetal loss, but hydrops and hygroma are not features of this condition.

26 A 22-year-old primigravida notes absent fetal movement for 2 days. The fetus is delivered stillborn at 19 weeks' gestation. The macerated fetus shows marked hydrops fetalis and a large posterior cystic hygroma of the neck. At autopsy, internal anomalies are seen, including aortic coarctation and a horseshoe kidney. Which of the following karyotypes is most likely to be present in cells obtained from the fetus? □ (A) 47,XX,+18 □ (B) 48,XXXY □ (C) 47,XX,+21 □ (D) 47,XYY □ (E) 45,X □ (F) 69,XXX

26 EXAM***(A) This infant has DiGeorge syndrome, which can involve the thymus, parathyroids, aorta, and heart. T cell function is deficient, resulting in recurrent and multiple fungal, viral, and protozoal infections. Failure of B cell maturation to plasma cells is a mode of development of common variable immunodeficiency. Some cases of severe combined immunodeficiency are caused by lack of adenosine deaminase. HIV infection does not explain the hypocalcemia at birth. Failure of pre-B cell maturation results in Bruton agammaglobulinemia.

26 A 30-year old woman gives birth at term to a normal-appearing infant girl. One hour after birth, the neonate exhibits tetany. On physical examination, she is at the 55th percentile for height and weight. Laboratory studies show serum calcium of 6.3 mg/dL and phosphorus of 3.5 mg/dL. Over the next year, the infant has bouts of pneumonia caused by Pneumocystis carinii and Aspergillus fumigatus and upper respiratory infections with parainfluenza virus and herpes simplex virus. Which of the following mechanisms is most likely to be responsible for the development of the clinical features seen in this infant? □ (A) Malformation of third and fourth pharyngeal pouches □ (B) Failure of maturation of B cells into plasma cells □ (C) Lack of adenosine deaminase □ (D) Acquisition of maternal HIV infection at delivery □ (E) Failure of differentiation of pre-B cells into B cells

26 (C) Exudation of fluid from venules and capillaries is a key component of the acute inflammatory process. Several mechanisms of increased vascular permeability have been proposed, including formation of interendothelial gaps by contraction of endothelium. This is caused by mediators, such as histamine and leukotrienes. The vessels then become more "leaky," and the fluid leaves the intravascular space to accumulate extravascularly, forming effusions in body cavities or edema in tissues. Arteriolar vasoconstriction is a transient response to injury that diminishes blood loss. After neutrophils reach the site of tissue injury outside of the vascular space, they release lysosomal enzymes. Platelets adhere to damaged endothelium and promote hemostasis. Lymphatic obstruction results in the accumulation of protein-rich lymph and lymphocytes, producing a chylous effusion.

26 A 35-year-old man has had increasing dyspnea for the past 24 hours. A chest radiograph shows large, bilateral pleural effusions. Thoracentesis yields 500 mL of slightly cloudy yellow fluid from the right pleural cavity. Cytologic examination of the fluid shows many neutrophils, but no lymphocytes or RBCs. Which of the following mechanisms contributes most to the accumulation of the fluid in the pleural space? □ (A) Arteriolar vasoconstriction □ (B) Neutrophil release of lysosomes □ (C) Endothelial contraction □ (D) Inhibition of platelet adherence □ (E) Lymphatic obstruction

26 (D) The surgery disrupted lymphatic return, resulting in functional lymphatic obstruction and lymphedema of the arm. The lymphatic channels are important in scavenging fluid and protein that have leaked into the tissues from the intravascular space. Although the amount of fluid that is drained through the lymphatics is not great, it can build up gradually. Cellulitis is caused by an infection of the skin and subcutaneous tissue, and displays erythema, warmth, and tenderness. Congestive heart failure can lead to peripheral edema, which is most marked in dependent areas such as the lower extremities and over the sacrum (in bedridden patients). Decreased plasma oncotic pressure from hypoalbuminemia, or sodium and water retention with heart or renal failure, leads to more generalized edema. Phlebothrombosis leads to swelling with pain and tenderness, but it is uncommon in the upper extremities.

26 A 58-year-old woman diagnosed with breast cancer in the left breast underwent a mastectomy with axillary lymph node dissection. Postoperatively, she developed marked swelling of the left arm that has persisted for several months. On physical examination, her temperature is 36.9°C. The left arm is not tender or erythematous, and it is not painful to movement or touch. Which of the following best describes the mechanism for these findings? □ (A) Cellulitis □ (B) Congestive heart failure □ (C) Decreased plasma oncotic pressure □ (D) Lymphedema □ (E) Sodium and water retention □ (F) Phlebothrombosis

2 6 (D) Lipofuscin is a "wear-and-tear" pigment that increases with aging, particularly in liver and myocardium. The pigment has minimal effect on cellular function in most cases. Rarely, there is marked lipofuscin deposition in a small heart, a so-called brown atrophy. Melanin pigment is responsible for skin tone: the more melanin, the darker the skin. Hemosiderin is the breakdown product of hemoglobin that contains the iron. Hearts with excessive iron deposition tend to be large. Glycogen is increased in some inherited enzyme disorders, and when the heart is involved, heart size increases. Bilirubin, another breakdown product of hemoglobin, imparts a yellow appearance (icterus) to tissues.

26 At autopsy, the heart of a 63-year-old man weighs only 250 g and has small right and left ventricles. The myocardium is firm, with a dark chocolate-brown color throughout. The coronary arteries show minimal atherosclerotic changes. An excessive amount of which of the following substances would most likely be found in the myocardial fibers of this heart? □ (A) Melanin □ (B) Hemosiderin □ (C) Glycogen □ (D) Lipofuscin □ (E) Bilirubin

27 (C) The patient has immunoglobulin deficiency, which prevents opsonization and phagocytosis of microbes. Deficiency of integrins and selectins, or a defect in microtubules, would prevent adhesion and locomotion of neutrophils. H2O2 production is part of the oxygen-dependent killing mechanism. This mechanism is intact in this patient because the neutrophils are able to kill bacteria when immunoglobulins in normal serum allow phagocytosis.

27 A 12-month-old boy with a 6-month history of repeated infections has had a fever and cough for the past 3 days. A Gram stain of sputum shows many gram-positive cocci in chains. CBC shows neutrophilia. Laboratory studies show that the patient's neutrophils phagocytose and kill organisms normally in the presence of normal human serum, but not in his own serum. The neutrophils migrate normally in a chemotaxis assay. Which of the following is the most likely cause of the child's increased susceptibility to infection? □ (A) Deficiency of integrins □ (B) Neutrophil microtubular protein defect □ (C) Immunoglobulin deficiency □ (D) Defective neutrophil generation of hydrogen peroxide □ (E) Deficiency of selectins

27 (B) Type 1 diabetes mellitus has an increased frequency in some families, but the exact mechanism of inheritance is unknown. The risk is about 6% for offspring when first-order relatives are affected. HLA-linked genes and other genetic loci and environmental factors are considered important. This pattern of inheritance is multifactorial. The other listed inheritance patterns are not seen with diabetes mellitus.

27 A 13-year-old boy has been drinking large quantities of fluids and has an insatiable appetite. He is losing weight and has become more tired and listless for the past month. Laboratory findings include normal CBC and fasting serum glucose of 175 mg/dL. Which of the following is the probable inheritance pattern of this disease? □ (A) Autosomal dominant □ (B) Multifactorial □ (C) X-linked recessive □ (D) Mitochondrial DNA □ (E) Autosomal recessive

27 (A) The CD4+ lymphocytes are thought to respond to some unknown antigenic stimulation, releasing cytokines that activate further macrophages and mast cells. The result is extensive dermal fibrosis that produces the clinical appearance of sclerodactyly in scleroderma. Neutrophils and natural killer cells do not participate in this process. Despite scleroderma being an autoimmune disease, inflammation is minimal. The major finding is progressive fibrosis of skin, lung, and gastrointestinal tract.

27 A 39-year-old woman has fingers that are tapered and clawlike, with decreased motion at the small joints. The microscopic appearance of the skin is shown in the figure. The patient also has diffuse interstitial fibrosis of the lungs, with pulmonary hypertension and cor pulmonale. Which of the following dermal inflammatory cells is the most likely initiator of the process that is the cause of her skin disease? □ (A) CD4+ lymphocyte □ (B) Macrophage □ (C) Mast cell □ (D) Neutrophil □ (E) Natural killer cell

27 (C) The patient has septic shock with poor tissue perfusion, evidenced by the high lactate level. Vasodilation is a feature of septic shock, typically as a result of gram-negative endotoxemia. Elevated creatine kinase suggests an acute myocardial infarction, which produces cardiogenic shock. Decreased Po2 suggests a problem with lung ventilation or perfusion. Increased blood urea nitrogen concentration is a feature of renal failure, not the cause of renal failure. Decreased hematocrit suggests hypovolemic shock from blood loss.

27 A 61-year-old woman has had a fever and felt faint for the past 2 days. On physical examination, her temperature is 38.4°C, pulse is 101/min, respirations are 17/min, and blood pressure is 85/40 mm Hg. She has marked peripheral vasodilation. The serum lactic acid level is 6.8 mg/dL. Which of the following laboratory findings is most likely to be related to the cause of this clinical condition? □ (A) Elevated serum creatine kinase □ (B) Decreased Po2 on blood gas measurement □ (C) Blood culture positive for Escherichia coli □ (D) Increased blood urea nitrogen □ (E) Decreased hematocrit

27 (A) Anthracotic pigmentation is common in lung and hilar lymph nodes and occurs when carbon pigment is inhaled from polluted air. The tar in cigarette smoke is a major source of such carbonaceous pigment. Resolution of hemorrhage can produce hemosiderin pigmentation, which imparts a brown color to tissues. Hepatic failure may result in jaundice, characterized by a yellow color. Older individuals generally have more anthracotic pigment, but this is not inevitable with aging—individuals living in rural areas with good environmental air quality have less pigment. Metastases impart a tan-towhite appearance to tissues.

27 A 69-year-old woman has had a chronic cough for the past year. A chest radiograph shows a 6-cm mass in the left lung, and a needle biopsy specimen of the mass shows carcinoma. A pneumonectomy is performed, and examination of the hilar lymph nodes reveals a uniform, dark black cut surface. Which of the following factors most likely accounts for the appearance of the lymph nodes? □ (A) Smoking □ (B) Bleeding disorder □ (C) Liver failure □ (D) Aging □ (E) Metastases

28 (C) Endothelial injury releases tissue factor that drives the coagulation process and activates platelets. Adjacent intact endothelium generates prostacyclin via arachidonic acid metabolism. Prostacyclin and nitric oxide are powerful vasodilators and inhibitors of platelet aggregation. This limits thrombus formation to the area of injury. Phospholipid and platelet-activating factor are procoagulants that drive thrombosis and platelet activation. Tissue-type plasminogen activator promotes fibrinolytic activity after a thrombus has formed. Thrombomodulin binds to thrombin to form an anticoagulant that activates protein C, which then cleaves activated factor V and factor VIII. Thromboxane is generated via arachidonic acid metabolism in platelets to promote platelet activation and vasoconstriction.

28 A 15-year-old girl incurs a cut to the sole of her foot after stepping on a piece of broken glass. On examination, there is a superficial 0.5-cm laceration that ceases to bleed in 5 minutes after application of local pressure. Which of the following substances is most likely to limit thrombus formation to the local area of injury by inhibiting platelet activation? □ (A) Phospholipid □ (B) Platelet-activating factor □ (C) Prostacyclin □ (D) Tissue-type plasminogen activator □ (E) Thrombomodulin □ (F) Thromboxane

2 8 (E) This woman's original symptoms, although nonspecific, are seen in more than half of adults with acute HIV infection. The average time to development of AIDS is 8 to 10 years; onset of opportunistic infections occurs as the CD4+ cell count decreases to less than 200/μL. Spondyloarthropathies and autoimmune diseases, such as systemic lupus erythematosus or scleroderma, are unlikely to have such a long interval between illnesses, and are not as likely to manifest opportunistic infections without immunosuppressive therapy. The ANA is typical of autoimmune diseases. HLAB27 antigen is associated with spondyloarthropathies. Anticentromere antibody is seen in limited scleroderma. Individuals with AIDS may have a polyclonal gammopathy, but not marked hypogammaglobulinemia.

28 A 19-year-old, previously healthy woman had an acute illness with fever, myalgia, sore throat, and mild erythematous rash over the abdomen and thighs. The symptoms abated after 1 month, and she remained healthy for 8 years. Now, decreased visual acuity and pain in the right eye lead to a finding of cytomegalovirus retinitis on funduscopy. Assuming that the patient's initial illness and the ocular problem are a part of the same disease process, which of the following laboratory findings would most likely be present after her ocular problems began to appear? □ (A) ANA titer 1 : 1024 □ (B) Total serum globulin level 650 mg/dL □ (C) Positive HLA-B27 antigen □ (D) Anticentromere antibody titer 1 : 512 □ (E) CD4+ lymphocyte count 102/μL

28 (B) Prostaglandins are produced through the cyclooxygenase pathway of arachidonic acid metabolism. Aspirin and other nonsteroidal anti-inflammatory drugs block the synthesis of prostaglandins, which can produce pain. Complement C1q is generated in the initial stage of complement activation, which can eventually result in cell lysis. Leukotrienes are generated by the lipoxygenase pathway, which is not blocked by aspirin. Histamine is mainly a vasodilator. Nitric oxide released from endothelium is a vasodilator.

28 A 35-year-old woman takes acetylsalicylic acid (aspirin) for arthritis. Although her joint pain is reduced with this therapy, the inflammatory process continues. The aspirin therapy alleviates her pain mainly through reduction in the synthesis of which of the following mediators? □ (A) Complement C1q □ (B) Prostaglandins □ (C) Leukotriene E4 □ (D) Histamine □ (E) Nitric oxide

28 (D) Decreased tissue perfusion leads to hypoxemia and depletion of ATP when cell metabolism shifts from aerobic to anaerobic glycolysis, resulting in depletion of glycogen stores and increased production and accumulation of lactic acid, reducing intracellular pH. Creatinine would increase with reduced renal function from decreased renal perfusion, but this would not explain the changes in other tissues. An increased glucose would be indicative of poorly controlled diabetes mellitus, not decreased perfusion. Carbon dioxide is likely to be cleared via the lungs, which are still sufficiently perfused. An increase in troponin I suggests irreversible myocardial injury.

28 A 44-year-old man has a history of poorly controlled diabetes mellitus leading to coronary artery disease. He now has decreasing cardiac output. An increase in which of the following substances in his blood is most indicative of reversible cell injury from decreased perfusion of multiple organs and tissues? □ (A) Carbon dioxide □ (B) Creatinine □ (C) Glucose □ (D) Lactic acid □ (E) Troponin I

28 (C) Failure to find amplified product by PCR analysis in such a case could mean that the fetus is not affected, or that there is a full mutation that is too large to be detected by PCR. The next logical step is a Southern blot analysis of genomic DNA from fetal cells. Routine karyotyping of the amniotic fluid cells is much less sensitive than a Southern blot analysis. Karyotyping of the unaffected father cannot provide information about the status of the FMR1 gene in the fetus because amplification of the trinucleotide occurs during oogenesis. For the same reason, PCR analysis of the mother's FMR1 gene is of no value.

28 A pregnant woman. with a family history of fragile X syndrome undergoes prenatal testing of her fetus. PCR analysis to amplify the appropriate region of the FMR1 gene is attempted using DNA from amniotic fluid cells, but no amplified products are obtained. Which of the following is the most appropriate next step? □ (A) Routine karyotyping of the amniotic fluid cells □ (B) Routine karyotyping of the unaffected father □ (C) Southern blot analysis of DNA from the amniotic fluid cells □ (D) PCR analysis of the mother's FMR1 gene □ (E) No further testing

29 (D) This child has features of Angelman syndrome, and the DNA analysis shows uniparental disomy. The Angelman gene encoded on chromosome 15 is subject to genomic imprinting. It is silenced on the paternal chromosome 15, but is active on the maternal chromosome 15. If the child lacks maternal chromosome 15, there is no active Angelman gene in the somatic cells. This gives rise to the abnormalities typical of this disorder. The same effect occurs when there is a deletion of the Angelman gene from the maternal chromosome 15. The other listed options do not occur in uniparental disomy.

29 A 3-year-old boy is brought to the physician because of progressive developmental delay, ataxia, seizures, and inappropriate laughter. The child has a normal karyotype of 46,XY, but DNA analysis shows that he has inherited both of his number 15 chromosomes from his father. These findings are most likely to be present with which of the following? □ (A) X-linked inheritance pattern □ (B) Maternal inheritance pattern □ (C) Mutation of mitochondrial DNA □ (D) Genomic imprinting □ (E) Trinucleotide repeat expansion

29 (A) Opportunistic infections in an intravenous drug abuser suggest a diagnosis of AIDS. The most common neoplasms seen in association with AIDS are B cell non-Hodgkin lymphoma and Kaposi sarcoma. A rare tumor associated with AIDS in children is leiomyosarcoma. Cervical dysplasias and carcinomas are increased in women with HIV infection, but such lesions are less frequent than lymphoma. Lung cancers at this woman's age are uncommon in any circumstance. Opportunistic infections of the brain and central nervous system lymphomas are common in patients with AIDS, but glial neoplasms are not.

29 A 32-year-old woman with a 10-year history of intravenous drug use has developed a chronic watery diarrhea that has persisted for the past week. On physical examination, she is afebrile and has mild muscle wasting. Her body mass index is 18. Laboratory studies of the stool show cysts of Cryptosporidium parvum. One month later, she develops cryptococcal meningitis, which is treated successfully. Oral candidiasis is diagnosed 1 month later. This patient is at greatest risk of developing which of the following neoplasms? □ (A) Intestinal non-Hodgkin lymphoma □ (B) Adenocarcinoma of the lung □ (C) Leiomyosarcoma of retroperitoneum □ (D) Cervical squamous carcinoma □ (E) Cerebral astrocytoma

2 9 (B) Aspirin blocks the cyclooxygenase pathway of arachidonic acid metabolism and generation of eicosanoids, including thromboxane A2, which causes vasoconstriction and promotes platelet aggregation. Platelet adhesion to extracellular matrix is mediated by interactions with von Willebrand's factor. Tissue factor (thromboplastin) is released with tissue injury and is not platelet dependent. Endothelial cells produce von Willebrand's factor independent of platelet action. Antithrombin III has anticoagulant properties because it inactivates several coagulation factors, but its function is not affected by aspirin.

29 A 59-year-old obese woman with a history of diabetes mellitus had a myocardial infarction 3 months ago. She is now taking a low dose of aspirin to reduce the risk of arterial thrombosis. On which of the following steps in hemostasis does aspirin have its greatest effect? □ (A) Adhesion of platelets to collagen □ (B) Aggregation of platelets □ (C) Production of tissue factor □ (D) Synthesis of von Willebrand factor □ (E) Synthesis of antithrombin III

2 9 (E) Bacterial infections often evoke an acute inflammatory response dominated by neutrophils. The extravasated neutrophils attempt to phagocytose and kill the bacteria. In the process, some neutrophils die, and the release of their lysosomal enzymes can cause liquefactive necrosis of the tissue. This liquefied tissue debris and the live and dead neutrophils comprise pus, or purulent exudate. Such an exudate is typical of bacterial infections that involve body cavities. Another term for purulent exudate in the pleural space is empyema. An abscess is a localized collection of neutrophils within tissues. Chronic inflammation occurs when there is a preponderance of mononuclear cells, such as lymphocytes, macrophages, and plasma cells, in a process that has gone on for more than a few days—more likely weeks or months— or that accompanies repeated bouts of acute inflammation. Edema refers to increased fluid collection within tissues, leading to tissue swelling. In fibrinous inflammation, exudation of blood proteins (including fibrinogen, which polymerizes to fibrin) gives a grossly shaggy appearance to surfaces overlying the inflammation. A serous effusion is a watery-appearing transudate that resembles an ultrafiltrate of blood plasma, with a low cell and protein content.

29 A 70-year-old woman has had worsening shortness of breath for the past week. On physical examination, her temperature is 38.3°C. On percussion, there is dullness over the left lung fields. Thoracentesis yields 800 mL of cloudy yellow fluid from the left pleural cavity. Analysis of the fluid reveals a WBC count of 2500/mm3 with 98% neutrophils and 2% lymphocytes. A Gram stain of the fluid shows gram-positive cocci in clusters. Which of the following terms best describes the process occurring in the left pleural cavity? □ (A) Abscess □ (B) Chronic inflammation □ (C) Edema □ (D) Fibrinous inflammation □ (E) Purulent exudate □ (F) Serous effusion

29 (B) These histologic findings are typical of apopotosis. The BCL-2 gene product inhibits cellular apoptosis by binding to Apaf-1. The BAX gene product promotes apoptosis. The C-MYC gene is involved with oncogenesis. The FAS gene encodes for a cellular receptor for FAS ligand, which signals apoptosis. p53 gene activity normally stimulates apoptosis, but mutation favors cell survival.

29 An experiment introduces a "knockout" gene mutation into a cell line. The frequency of shrunken cells with chromatin clumping and cytoplasmic blebbing is increased compared with a cell line without the mutation. Overall survival of the mutant cell line is reduced. Which of the following genes is most likely to be affected by this mutation? □ (A) BAX □ (B) BCL-2 □ (C) C-MYC □ (D) FAS □ (E) p53

3 (C) The mechanism for fat embolism is unknown, in particular, why onset of symptoms is delayed 1 to 3 days after the initial injury (or 1 week for cerebral symptoms). The cumulative effect of many small fat globules filling peripheral pulmonary arteries is the same as one large pulmonary thromboembolus. Hemothorax and cardiac tamponade would be immediate complications after traumatic injury, not delayed events. Pulmonary edema severe enough to cause dyspnea would be unlikely to occur in hospitalized patients because fluid status is closely monitored. Pulmonary infarction may cause dyspnea, but pulmonary thromboembolus from deep venous thrombosis is typically a complication of a longer hospitalization.

3 A 21-year-old woman sustains multiple injuries, including fractures of the right femur and tibia and the left humerus, in a motor vehicle collision. She is admitted to the hospital, and the fractures are stabilized surgically. Soon after admission to the hospital, she is in stable condition. She suddenly becomes severely dyspneic, however, 2 days later. Which of the following complications is the most likely cause of this sudden respiratory difficulty? □ (A) Right hemothorax □ (B) Pulmonary edema □ (C) Fat embolism □ (D) Cardiac tamponade □ (E) Pulmonary infarction

4 9 (B) This patient has cutaneous and visceral manifestations of diffuse systemic sclerosis (diffuse scleroderma). Raynaud phenomenon, skin changes, and esophageal dysmotility also can occur in limited scleroderma (CREST syndrome), but lung and renal involvement typically do not. In diffuse systemic sclerosis, the anti-DNA topoisomerase I antibody is often present, and patients can develop interstitial lung disease and renal disease with hyperplastic arteriolosclerosis. A feature of discoid lupus erythematosus is skin rashes, but usually there is no internal organ involvement. Ankylosing spondylitis is one of the spondyloarthropathies; it is characterized by low back pain from sacroiliitis and positive serology for HLA-B27. Reiter syndrome is characterized by conjunctivitis, arthritis, and nongonococcal urethritis, with a positive serology for HLA-B27. In rheumatoid arthritis, there is often progressive joint deformity; the serologic tests likely to be positive include rheumatoid factor and antibodies to cyclic citrullinated peptide (anti-CCP). The anti-Sm or anti-double-stranded DNA antibodies are more specific for systemic lupus erythematosus, and renal disease in these patients is most likely due to glomerulonephritis.

49 For the past 2 years, a 42-year-old woman has had tightening of the skin of her fingers, making them difficult to bend. She has had increasing difficulty swallowing for the past 8 months. During the past winter, her fingers became cyanotic and painful on exposure to cold. On physical examination, the skin on her face, neck, hands, and forearms appears firm and shiny. Her blood pressure is 150/95 mm Hg. A chest radiograph shows prominent interstitial markings, and lung function tests indicate moderately severe restrictive pulmonary disease. The result of the DNA topoisomerase I antibody test is positive. Which of the following conditions is most likely to produce these findings? □ (A) Ankylosing spondylitis □ (B) Diffuse systemic sclerosis □ (C) Discoid lupus erythematosus □ (D) Limited scleroderma □ (E) Reiter syndrome □ (F) Rheumatoid arthritis □ (G) Systemic lupus erythematosus

3 (B) The patient's signs and symptoms suggest acute bacterial pneumonia. Such infections induce an acute inflammation dominated by neutrophils, which gives the sputum its yellowish, purulent appearance. Macrophages become more numerous after acute events, cleaning up tissue and bacterial debris through phagocytosis. Mast cells are better known as participants in allergic and anaphylactic responses. Lymphocytes are a feature of chronic inflammation. Langhans giant cells are seen with granulomatous inflammatory responses.

3 A 53-year-old woman has had a high fever and cough productive of yellowish sputum for the past 2 days. Her vital signs include temperature of 37.8°C, pulse of 83/min, respirations of 17/min, and blood pressure of 100/60 mm Hg. On auscultation of the chest, crackles are audible in both lung bases. A chest radiograph shows bilateral patchy pulmonary infiltrates and fluid in the right pleural cavity. Thoracentesis yields 500 mL of cloudy yellow fluid. Which of the following inflammatory cell types is most likely to be seen in greatly increased numbers in a sputum specimen? □ (A) Macrophages □ (B) Neutrophils □ (C) Mast cells □ (D) Small lymphocytes □ (E) Langhans giant cells

3 (C) Hepatocytes are stable cells with an extensive ability to regenerate. The ability to restore normal architecture of an organ such as the liver depends on the viability of the supporting connective tissue framework. If the connective tissue cells are not injured, hepatocyte regeneration can restore normal liver architecture. This regeneration occurs in many cases of viral hepatitis. A liver abscess is associated with liquefactive necrosis of hepatocytes and the supporting connective tissue. It heals by scarring. The other options listed may explain the amount of liver injury, but not the nature of the response.

3 An experiment infects one group of test animals with viral hepatitis. Two months later, complete recovery of the normal liver architecture is observed when the livers from these animals are examined microscopically. A second test group is infected with bacterial organisms, and after the same period of time, fibrous scars from resolving abscesses are seen microscopically in the livers. Which of the following factors best explains the different outcomes for the two test groups? □ (A) Nature of the etiologic agent □ (B) Extent of liver cell injury □ (C) Injury to the connective tissue framework □ (D) Location of the lesion □ (E) Extent of damage to the bile ducts

3 (B) Entry of HIV into cells requires binding to the CD4 molecule and coreceptor molecules, such as CCR5 and CXCR4. These HIV coreceptors are receptors for chemokines on the surface of T cells and macrophages. Mutations in genes encoding these coreceptor molecules cause individuals to be resistant to the effects of HIV infection because HIV cannot enter lymphocytes and macrophages. The other cell surface receptors are not relevant for HIV entry into cells. The p24 antigen is contained within the HIV virion and is not part of cell entry mechanisms, although its presence aids in detection of HIV infection.

3 In epidemiologic studies of HIV infection and AIDS, investigators noticed that certain individuals did not develop HIV infection despite known exposure to the virus under conditions that caused HIV disease in all other individuals similarly exposed. When CD4+ lymphocytes from resistant individuals are incubated with HIV-1, they fail to become infected. Such resistance to infection by HIV is most likely caused by a mutation affecting genes for which of the following cellular components? □ (A) T-cell receptor □ (B) Chemokine receptor □ (C) Interleukin-2 receptor □ (D) CD28 receptor □ (E) Fc receptor □ (F) p24 antigen

3 (E) The pressure load on the left ventricle results in an increase in myofilaments in the existing myofibers. The result of continued stress from hypertension is eventual heart failure with decreased contractility, but the cells do not decrease in size. Metaplasia of muscle does not occur, although loss of muscle occurs with aging as myofibers are replaced by fibrous tissue and adipose tissue. Hemosiderin deposition in the heart is a pathologic process resulting from increased iron stores in the body.

3 On a routine visit to the physician, an otherwise healthy 51-year-old man has a blood pressure of 150/95 mm Hg. If his hypertension remains untreated for years, which of the following cellular alterations would most likely be seen in his myocardium? □ (A) Atrophy □ (B) Hyperplasia □ (C) Metaplasia □ (D) Hemosiderosis □ (E) Hypertrophy

3 (B) The findings listed are consistent with Hunter syndrome, one of the mucopolysaccharidoses (MPS) that results from deficiencies of lysosomal enzymes, such as α-l-iduronidase. The glycosaminoglycans that accumulate in MPS include dermatan sulfate, heparan sulfate, keratan sulfate, and chondroitin sulfate. All of the MPS variants are autosomal recessive except for Hunter syndrome, which is X-linked recessive. Adenosine deaminase deficiency is a cause of severe combined immunodeficiency (SCID), an immunodeficiency state in which multiple recurrent infections occur after birth. Glucocerebrosidase deficiency is seen in Gaucher disease; in the most common form of the disease, there is no neurologic impairment, and patients have splenomegaly and skeletal disease as a consequence of increased lysosomal glucocerebrosides in cells of the mononuclear phagocyte system. Glucose-6-phosphatase deficiency leads to von Gierke disease, characterized by hepatomegaly, renomegaly, and impaired gluconeogenesis leading to hypoglycemia and hyperlipidemia. Hexosaminidase A deficiency occurs in Tay-Sachs disease; affected individuals manifest severe neurologic impairment, poor motor development, and blindness beginning in infancy. Lysosomal glucosidase deficiency, seen in Pompe disease, is associated with marked cardiomegaly and heart failure beginning in infancy. Sphingomyelinase deficiency occurs in Niemann-Pick disease type A, characterized by hepatosplenomegaly, lymphadenopathy, and severe motor and mental impairment.

3 The parents of a male infant come to the physician because of their concern that male children over several generations in the mother's family have been affected by a progressive disorder involving multiple organ systems. These children have had coarse facial features, corneal clouding, joint stiffness, hepatosplenomegaly, and mental retardation, and many died in childhood. At autopsy, some of the children had subendothelial coronary arterial deposits that caused myocardial infarction. Laboratory testing of the infant shows increased urinary excretion of mucopolysaccharides. Bone marrow biopsy is performed, and the accumulated mucopolysaccharides are found in macrophages ("balloon cells" filled with minute vacuoles). Which of the following enzyme deficiencies is most likely to be seen in this infant? □ (A) Adenosine deaminase □ (B) α-l-iduronidase □ (C) Glucocerebrosidase □ (D) Glucose-6-phosphatase □ (E) Hexosaminidase A □ (F) Lysosomal glucosidase □ (G) Sphingomyelinase

30 (C) This man has Marfan syndrome, an autosomal-dominant condition that is caused by quantitative and qualitative defects in fibrillin from mutations in the fibrillin gene. Genetic mutations in the dystrophin gene are involved in Duchenne and Becker muscular dystrophies. An abnormal collagen gene can cause osteogenesis imperfecta and Ehlers-Danlos syndrome. The NF1 protein is abnormal in neurofibromatosis type 1. Disordered spectrin causes hereditary spherocytosis.

30 A 22-year-old man has a sudden loss of vision in the right eye. On physical examination, there is a subluxation of the crystalline lens of the right eye. On auscultation of the chest, a midsystolic click is audible. An echocardiogram shows a floppy mitral valve and dilated aortic arch. The patient's brother and his cousin are similarly affected. A genetic defect involving which of the following substances is most likely to be present in this patient? □ (A) Dystrophin □ (B) Collagen □ (C) Fibrillin-1 □ (D) NF1 protein □ (E) Spectrin

30 (A) During acute inflammation, neutrophils extravasate from the blood vessels. This process depends on adhesion molecules expressed on the neutrophils and endothelial cells. In the first stage of extravasation, the neutrophils "roll over" the endothelium. At this stage, the adhesion between the neutrophils and endothelial cells is weak. Rolling is mediated by binding of selectins to sialylated oligosaccharides. The next step, firm adhesion, is mediated by binding of integrins on the leukocytes to their receptors, intracellular adhesion molecule-1 or vascular cell adhesion molecule-1 (VCAM-1), on endothelial cells. Integrins have two chains, α and β. A genetic lack of β chains prevents firm adhesion of leukocytes to endothelial cells. Neutrophil migration to a site of infection depends on the presence of chemotactic factors that bind to the neutrophil and activate phospholipase C to begin a series of events that culminate in the influx of calcium, which triggers contractile proteins. The respiratory burst to kill phagocytized organisms depends on NADPH oxidase, and a deficiency of this enzyme leads to chronic granulomatous disease. Phagocytosis of opsonized organisms depends on engulfment, which requires contractile proteins in the neutrophil cytoplasm. Formation of HOCl− requires myeloperoxidase released from neutrophil granules.

30 A 5-year-old boy has a history of recurrent infections with gram-positive bacteria, including Staphylococcus aureus. Genetic testing shows a defect leading to a lack of β2-integrin production. Which of the following abnormalities of neutrophil function is most likely responsible for these clinical symptoms? □ (A) Normal neutrophil rolling but inadequate sticking on cytokine-activated endothelial cells □ (B) Failure of neutrophils to migrate to the site of infection after leaving the vasculature □ (C) Reduced respiratory burst in neutrophils after phagocytosis of bacteria □ (D) Diminished phagocytosis of bacteria opsonized with IgG □ (E) Failure to generate hydroxy-halide radicals (HOCl−)

30 (F) Venous stasis favors the development of phlebothrombosis (venous thrombosis), particularly in the leg and pelvic veins. This is a common complication in hospitalized patients who are bedridden. The obstruction may produce local pain and swelling, or it may be asymptomatic. Such deep thrombi in large veins create a risk for pulmonary thromboembolism. Anasarca refers to marked generalized edema. Chronic passive congestion refers to capillary, sinusoidal, or venous stasis of blood within an organ such as the lungs or liver. Fat emboli are globules of lipid that are most likely to form after traumatic injury, typically to long bones. Mural thrombi are thrombi that form on the surfaces of the heart or large arteries. After a thrombus has formed, it may become organized with ingrowth of capillaries, fibroblast proliferation, and macrophage infiltration that eventually clears part or most of the clot, forming one or more new lumens (recanalization). Phlebothrombosis occurs when stasis in large veins promotes thrombosis formation, typically in leg and pelvic veins; because there is often clinically apparent swelling, warmth, and pain, the term thrombophlebitis is often employed regardless of whether true vascular inflammation is present. A vegetation is a localized thrombus formation on cardiac endothelium, typically a valve.

30 A 60-year-old woman sustained fractures of the right femur, pelvis, and left humerus in a motor vehicle collision. The fractures were stabilized, and the patient's recovery was uneventful. During a physical examination 3 weeks later, the physician observes swelling and warmth in the left leg, and there is local pain and tenderness in the left thigh. Which of the following processes is most likely occurring in the femoral vein? □ (A) Anasarca □ (B) Chronic passive congestion □ (C) Fat embolus formation □ (D) Mural thrombosis □ (E) Organization with recanalization □ (F) Phlebothrombosis □ (G) Vegetation

30 (A) Irreversible cellular injury is likely to occur when calcium increases within cells. This calcium can influx into cells and be released from mitochondria and endoplasmic reticulum. The calcium activates ATPases, phospholipases, proteases, and endonucleases, which injure cell components. Mitochondrial permeability is increased to release cytochrome c, which activates caspases leading to apoptosis. Of the other ions listed, sodium influxes into the cell, while potassium diffuses out when the sodium pump fails as ATP levels fall, but this is potentially reversible.

30 A tissue preparation is experimentally subjected to a hypoxic environment. The cells in this tissue begin to swell, and chromatin begins to clump in the nucleus. ATPases are activated, and ATP production decreases. Which of the following ions released from mitochondria leads to these findings and to eventual cell death? □ (A) Ca2+ □ (B) Cl− □ (C) HCO3 2− □ (D) K+ □ (E) Na+ □ (F) PO4 2+

30 (C) Three types of cells can carry HIV: dendritic cells, monocyte-macrophages, and CD4+ T cells. Mucosal dendritic cells (i.e., Langerhans cells) can bind to the virus and transport it to CD4+ cells in the lymph nodes. Whether the virus is internalized by mucosal dendritic cells is unclear. Monocyte-macrophages and CD4+ T cells express CD4 and the coreceptors CCR5 and CXCR4; HIV can enter these cells. Follicular dendritic cells are distinct from mucosal or epithelial dendritic cells; they trap antibody-coated HIV virions by means of their Fc receptors. The other listed cells cannot be infected by HIV.

30 An epidemiologic study is conducted to determine risk factors for HIV infection. The study documents that individuals with coexisting sexually transmitted diseases such as chancroid are more likely to become HIV-positive. It is postulated that an inflamed mucosal surface is an ideal location for the transmission of HIV during sexual intercourse. Which of the following cells in these mucosal surfaces is most instrumental in transmitting HIV to CD4+ T lymphocytes? □ (A) CD8+ cells □ (B) Natural killer cells □ (C) Dendritic cells □ (D) Neutrophils □ (E) Plasma cells

31 (A) This woman has androgen insensitivity ("testicular feminization") syndrome, which results from X-linked inheritance of a defect in the androgen receptor. The cells of affected individuals do not respond to dihydrotestosterone, but they do respond to estradiol, producing a phenotypic female. Testes are present, but typically are cryptorchid, and there is an increased risk of testicular neoplasms. Manifestations of fragile X syndrome are more likely to be seen in males. The disorder stems from an abnormality in a triple repeat sequence in the FMR1 gene, and the most significant consequence is mental retardation. Gonadal mosaicism refers to the presence of a genetic mutation acquired during embryogenesis and present only in germ cells; the individual with the mutation does not manifest the disease, but multiple offspring do. Hypopituitarism can lead to hypogonadism and diminished secondary sex characteristics, but an individual with a karyotype of 46,XY would be a phenotypic male. Agenesis of the testes does not occur in individuals with the androgen insensitivity syndrome, but lack of normal descent may make it difficult to observe the testes in these individuals.

31 A 24-year-old woman comes to the physician because she has been unable to bear children during her 3 years of marriage. She has never had a menstrual period. On physical examination, she has normal breast development and scanty axillary and pubic hair. Pelvic examination shows a short, blind-ending vagina with no palpable uterus or adnexa. Chromosomal analysis indicates a 46,XY karyotype. Which of the following findings is most likely to be present? □ (A) Abnormal androgen receptor □ (B) Fragile X syndrome □ (C) Gonadal mosaicism □ (D) Hypopituitarism □ (E) Testicular agenesis

3 1 (C) This patient has graft-versus-host disease. The engrafted marrow contains immunocompetent cells that can proliferate and attack host tissues, usually skin, liver, and gastrointestinal epithelium. Tuberculin skin testing is a form of delayed-type hypersensitivity. Some chemotherapy agents can produce a drug reaction with more acute inflammation than was described in this case. Urticaria with type I hypersensitivity is a typical reaction to penicillin therapy. Patch testing is done to determine the type of allergens to which atopic individuals may react.

31 A 35-year-old woman who has been in the hospital receiving treatment for leukemia has developed an extensive, scaling rash over the past week. A skin biopsy specimen shows keratinocyte apoptosis along the dermal-epidermal junction, with upper dermal lymphocytic infiltrates. She also has jaundice. This patient has most likely recently undergone which of the following procedures? □ (A) Tuberculin skin testing □ (B) Chemotherapy for malignant lymphoma □ (C) Allogeneic bone marrow transplantation □ (D) Penicillin therapy for pneumonia □ (E) Patch testing for allergen detection

3 1 (A) The hydrostatic pressure exerted from standing leads to edema in dependent parts of the body. Lymphatic obstruction from infection or tumor can lead to lymphedema, but this is a chronic process. Secondary aldosteronism results from congestive heart failure and renal hypoperfusion, but this is a generalized process. Hypoalbuminemia leads to more generalized edema, although the effect is more pronounced in dependent areas. In a healthy patient, normal renal function would be sufficient to clear free water ingested orally.

31 A 45-year-old woman who works as a bank teller notices at the end of her 8-hour shift that her lower legs and feet are swollen, although there was no swelling at the beginning of the day. There is no pain or erythema associated with this swelling. The woman is otherwise healthy and takes no medications; laboratory testing reveals normal liver and renal function. Which of the following mechanisms best explains this phenomenon? □ (A) Increased hydrostatic pressure □ (B) Lymphatic obstruction □ (C) Secondary aldosteronism □ (D) Hypoalbuminemia □ (E) Excessive water intake

31 (B) The cheese-like appearance gives this form of necrosis its name—caseous necrosis. In the lung, tuberculosis and fungal infections are most likely to produce this pattern of tissue injury. Apoptosis involves individual cells, without extensive or localized areas of tissue necrosis. Coagulative necrosis is more typical of ischemic tissue injury. Fat necrosis most often occurs in the breast and pancreas. Fatty change is most often a feature of hepatocyte injury, and the cell integrity is maintained. Gangrene characterizes extensive necrosis of multiple cell types in a body region or organ. Liquefactive necrosis is seen in abscesses or ischemic cerebral injury.

31 A chest radiograph of an asymptomatic 37-year-old man showed a 3-cm nodule in the middle lobe of the right lung. The nodule was excised with a pulmonary wedge resection, and sectioning showed the nodule to be sharply circumscribed with a soft, white center. Culture of tissue from the nodule grew Mycobacterium tuberculosis. Which of the following pathologic processes has most likely occurred in this nodule? □ (A) Apoptosis □ (B) Caseous necrosis □ (C) Coagulative necrosis □ (D) Fat necrosis □ (E) Fatty change □ (F) Gangrenous necrosis □ (G) Liquefactive necrosis

31 (E) Macrophages contain cytokine-inducible nitric oxide synthase (iNOS), which generates nitric oxide. Nitric oxide, by itself and on interaction with other reactive oxygen species, has antimicrobial activity. CD4 or CD8 lymphocytes can be the source for interferon-γ (IFN-γ), which stimulates macrophage production of NOS. Endothelial cells contain a form of NOS (eNOS) that acts to promote vasodilation. B lymphocytes produce immunoglobulins that can opsonize bacteria. Basophils release histamine and arachidonic acid metabolites, which participate in the acute inflammatory process. Neutrophils can phagocytize microbes, but use NAPDH oxidase and enzymes other than NOS to kill the microbes. Natural killer cells have Fc receptors and can lyse IgG-coated target cells; they also generate IFN-γ.

31 An experiment isolates peripheral blood cells into a culture medium that preserves their metabolic activity. After interferon-γ is added to this culture, the cells are incubated. Next, a cell-free supernatant from this culture is added to a second culture medium containing Escherichia coli organisms. Which of the following cell types is the most likely source for observed bactericidal activity against E. coli □ (A) Basophil □ (B) B lymphocyte □ (C) CD4 lymphocyte □ (D) CD8 lymphocyte □ (E) Macrophage □ (F) Neutrophil □ (G) Natural killer cell

3 2 (B) These findings are characteristic of immune complex-mediated glomerulonephritis. The immune complexes activate complement and result in acute inflammation. Antibody-dependent cell-mediated cytotoxicity is initiated when IgG or IgE coats a target to attract cells that affect lysis; immune complexes do not form. Localized anaphylaxis is a type I hypersensitivity reaction that is mediated by IgE antibody. Granulomatous inflammation and T cell cytotoxicity are features of type IV hypersensitivity.

32 A 20-year-old woman has had increasing malaise, oliguria, and peripheral edema for the past week. On physical examination, she has 2+ pitting edema to the knees and puffiness around the eyes. Laboratory studies show serum creatinine of 4.6 mg/dL and urea nitrogen of 42 mg/dL. A renal biopsy specimen shows positive immunofluorescent staining for immunoglobulin and complement C3 within the glomeruli. The electron microscopic appearance of the specimen is shown in the figure. Which of the following immunological mechanisms has most likely produced the renal damage seen in this patient? □ (A) Antibody-dependent cell-mediated cytotoxicity □ (B) Immune complex-mediated hypersensitivity □ (C) Localized anaphylaxis □ (D) Granulomatous inflammation □ (E) T cell-mediated cytotoxicity

33 (A) The T-cell receptor (TCR) of a CD8+ cell clone that is specific for a particular peptide reacts and produces a cellmediated immune response to eliminate the virally infected cells through cytotoxic action producing apoptosis. All nucleated cells display class I antigens (HLA A, B, and C). Antigen-presenting cells such as macrophages and dendritic cells (e.g., Langerhans cells) can display class II antigens (HLA DP, DQ, DR). Mast cells have IgE fixed to their surfaces that can react with allergens to cause degranulation and release of biogenic amines in type I hypersensitivity reactions. Neutrophils react nonspecifically to inflammatory stimuli. NK cells are programmed to destroy cells that do not display MHC class I antigens. Plasma cells are fully differentiated B cells that produce immunoglobulin.

33 A macrophage ingests a virion and degrades it so that viral capsid peptides can be linked to class I HLA heavy chains and β2-microglobulin. This trimer is transported to the macrophage cell surface and displayed. Which of the following cell types has receptors that can interact with the displayed MHC-peptide complex? □ (A) CD8+ lymphocyte □ (B) Langerhans cell □ (C) Macrophage □ (D) Mast cell □ (E) Neutrophil □ (F) NK cell □ (G) Plasma cell

32 (D) The infant has cystic fibrosis. The elevated sweat chloride level is related to a defect in the transport of chloride ions across epithelia. The most common genetic defect is a deletion of three base pairs at the ΔF508 position coding for phenylalanine in the CFTR gene. A frameshift mutation involves one or two base pairs, not three, and changes the remaining sequence of amino acids in a protein. A point mutation may change the codon to the sequence of a "stop" codon, which truncates the protein being synthesized, typically leading to degradation of the protein. A point mutation typically is a missense mutation that leads to replacement of one amino acid for another in the protein chain; this can lead to abnormal conformation and function of the protein. A trinucleotide repeat sequence mutation leads to amplification of repeats of three nucleotides, so-called tandem repeats, which prevent normal gene expression.

32 A 21-year-old primigravida gives birth to a term infant after an uncomplicated pregnancy. The infant is of normal height and weight, and no anomalies are noted. The infant fails to pass meconium, however. Laboratory studies show an elevated sweat chloride level. Genetic testing indicates that a critical protein coded by a gene is missing one phenylalanine amino acid in the protein sequence. Which of the following types of gene mutations is most likely to produce these findings? □ (A) Frameshift □ (B) Nonsense (stop codon) □ (C) Point □ (D) Three-base pair deletion □ (E) Trinucleotide repeat

32 (B) Amniotic fluid embolism rarely occurs in pregnancy, but it has a high mortality rate. The fluid reaches torn uterine veins through ruptured fetal membranes. Aggregates of red blood cells are seen in passive congestion. Fat globules are seen in fat embolism, usually after severe trauma. Gas bubbles in vessels from air embolism can be a rare event in some obstetric procedures, but it is an unlikely event in natural deliveries. Peripheral pulmonary thromboemboli are most likely to produce chronic pulmonary hypertension and develop over weeks to months.

32 A 23-year-old woman with an uncomplicated pregnancy develops sudden dyspnea with cyanosis and hypotension during routine vaginal delivery of a term infant. She has a generalized seizure and becomes comatose. Her condition does not improve over the next 2 days. Which of the following findings is most likely to be present in the peripheral pulmonary arteries? □ (A) Aggregates of red blood cells □ (B) Amniotic fluid □ (C) Fat globules □ (D) Gas bubbles □ (E) Thromboemboli

32 (E) Ongoing activation of coagulation generates an inflammatory response that further amplifies coagulation, creating a vicious cycle. Protein C antagonizes coagulation factor V, which catalyzes activation of prothrombin to thrombin, thereby breaking the cycle of thrombin generation. Complement components can become activated by plasmin (C3) and kallikrein (C5) forming anaphlytoxins (C3a and C5a) that promote inflammation. Fibrin is the end product of coagulation pathways that forms a meshwork entrapping platelets and forming a plug. Kallikrein is generated by activation of Hageman factor (XII) and leads to formation of bradykinin. Plasmin is generated from plasminogen activated by thrombosis to promote clot lysis.

32 Patients with extensive endothelial injury from Escherichia coli sepsis have consumption of coagulation factors as well as an extensive inflammatory response. Administration of activated protein C decreases this inflammatory response by reducing the amount of a substance that normally binds to protease-activated receptors to trigger expression of adhesion molecules, cytokines, and chemokines. What is this substance most likely to be? □ (A) Complement □ (B) Fibrin □ (C) Kallikrein □ (D) Plasmin □ (E) Thrombin

3 2 (D) The increase in uterine size is primarily the result of an increase in myometrial smooth muscle cell size. The endometrium also increases in size, but it remains as a lining to the muscular wall and does not contribute as much to the change in size. There is little stroma in myometrium and a greater proportion in endometrium, but this contributes a smaller percentage to the gain in size than muscle. The vessels are a minor but essential component in this process.

32 The nonpregnant uterus of a 20-year-old woman measures 7 × 4 × 3 cm. The woman becomes pregnant and just before delivery of a term infant, the uterus measures 34 × 18 × 12 cm. Which of the following cellular processes has contributed most to the increase in uterine size? □ (A) Endometrial glandular hyperplasia □ (B) Myometrial fibroblast proliferation □ (C) Endometrial stromal hypertrophy □ (D) Myometrial smooth muscle hypertrophy □ (E) Vascular endothelial hyperplasia

3 3 (A) Heterophagocytosis by macrophages requires that endocytosed vacuoles fuse with lysosomes to degrade the engulfed material. With congestive failure, extravasation of RBCs into alveoli occurs, and pulmonary macrophages must phagocytose the RBCs, breaking down the hemoglobin and recycling the iron by hemosiderin formation.

33 A 40-year-old woman has had chronic congestive heart failure for the past 3 years. In the past 2 months, she developed a cough productive of rust-colored sputum. A sputum cytology specimen shows numerous hemosiderin-laden macrophages. Which of the following subcellular structures in macrophages is most important for the accumulation of this pigment? □ (A) Lysosome □ (B) Endoplasmic reticulum □ (C) Ribosome □ (D) Golgi apparatus □ (E) Chromosome

33 (F) This child has nephrotic syndrome with loss of albumin into the urine and hypoalbuminemia that decreases plasma oncotic pressure, leading to movement of intravascular water into the extravascular compartment to produce edema. In response, hypovolemia with renal hypoperfusion induces increased production of renin, angiotensin, and aldosterone, which promote sodium and water retention, further exacerbating his edema. Thiazide diuretics increase renal excretion of sodium. Hypovolemia would increase antidiuretic hormone output. Though corticosteroids are used to treat nephrotic syndrome caused by minimal change disease, the effect is probably to diminish abnormal T-cell function that is driving the glomerular damage. Cortisol leads to sodium retention, but not in response to hypovolemia.

33 A 7-year-old boy has had increasing lethargy for a week. On physical examination he has periorbital edema and pitting edema at the ankles, but is normotensive and afebrile. Laboratory studies show marked albuminuria. He is given a thiazide diuretic and his urine output increases and his edema resolves. Which of the following changes in his blood most likely potentiated his edema? □ (A) Decreased aldosterone □ (B) Decreased antidiuretic hormone □ (C) Decreased renin □ (D) Increased albumin □ (E) Increased cortisol □ (F) Increased sodium

33 (A) The infant has Tay-Sachs disease, an autosomal recessive condition that has a gene frequency higher in some populations, including Ashkenazi Jews and Quebecois. The gene that encodes for the enzyme hexosaminidase A has a four-base pair insertion, leading to an altered reading frame and appearance of a stop codon that prematurely terminates reading and amino acid sequence to produce a nonfunctional enzyme. A frameshift mutation involves one or two base pairs and changes the remaining sequence of amino acids in a protein. A point mutation may change the codon to the sequence of a "stop" codon that truncates the protein being synthesized, typically leading to degradation of the protein. A point mutation typically is a missense mutation that leads to replacement of one amino acid for another in the protein chain; this can lead to abnormal conformation and function of the protein. A deletion of three base pairs leads to loss of a single amino acid in a protein. A three-base pair deletion, as occurs in cystic fibrosis, results in a frameshift involving just a single amino acid. A trinucleotide repeat is the inheritance pattern for fragile X syndrome, which is caused by triple repeat expansions in the FMR1 gene.

33 An infant born to a family living in Belarus appeared normal at birth, but at 6 months is noted to have worsening motor incoordination and mental obtundation. On examination, the infant has retinal pallor with a prominent macula. Flaccid paralysis develops by 1 year of age. Based on the figure, which of the following mutations most likely occurred? □ (A) Four-base pair insertion □ (B) Nonsense (stop codon) □ (C) Point □ (D) Three-base pair deletion □ (E) Trinucleotide repeat

34 (B) This patient has polymyositis. Muscle weakness in polymyositis tends to be symmetric, and proximal muscles are involved first. This condition differs from dermatomyositis in that there is no skin involvement, and polymyositis typically affects adults. On biopsy, the skeletal muscle shows infiltration by lymphocytes along with degeneration and regeneration of muscle fibers. The lymphocytes are cytotoxic CD8+ cells. Some patients may have myocarditis, vasculitis, or pneumonitis, but in contrast to dermatomyositis, the risk of cancer is equivocal. Bony ankylosis is a feature of progressive or recurrent joint inflammation with rheumatoid arthritis. Pericarditis is most likely to be a feature of systemic lupus erythematosus or diffuse systemic sclerosis. Raynaud phenomenon is seen in many autoimmune phenomena, but it is most often a feature of scleroderma. Sclerodactyly is a feature of scleroderma. When not extensive, it typically indicates limited scleroderma (CREST syndrome); when extensive, it indicates diffuse scleroderma, which has a poorer prognosis. Nongonococcal urethritis, conjunctivitis, and arthritis are seen in Reiter syndrome. Xerophthalmia (usually with accompanying xerostomia) is seen in Sjögren syndrome.

34 A 29-year-old woman has had increasing weakness over the past year, and now has difficulty climbing a single flight of stairs. Her muscles are sore most of the time. She has little difficulty writing or typing, however. During the past 3 months, she has had increasing difficulty swallowing. She has experienced chest pain for the past week. On physical examination, she is afebrile. Her blood pressure is 115/75 mm Hg. Muscle strength is 4/5 in all extremities. No rashes are present. She has 2+ pitting edema to the knees. Rales are auscultated over lower lung fields. Laboratory studies show serum creatine kinase level of 458 U/L and Jo-1 antibodies. Which of the following conditions is she most likely to have? □ (A) Bony ankylosis □ (B) Myocarditis □ (C) Pericarditis □ (D) Raynaud phenomenon □ (E) Sclerodactyly □ (F) Urethritis □ (G) Xerophthalmia

34 (A) His elevated prothrombin time that corrects with normal plasma points to coagulation factor deficiency, and factors II, VII, IX, and X are synthesized in the liver and affect this "extrinsic" pathway. They are vitamin K dependent and are therefore affected by coumadin therapy. His hepatic steatosis diminishes liver function. Hemophilia A results from loss of factor VIII function and affects just the partial thromboplastin time. Antiphospholipid syndrome has an inhibitory effect upon in vitro coagulation tests and does not correct with addition of normal plasma. The factor V Leiden mutation leads to difficulty inactivating factor V by the action of protein C, thus causing thrombosis, not bleeding. Gram negative sepsis releases lipopolysaccharide that activates coagulation on a wide scale, consuming coagulation factors and platelets, so both the prothrombin time and partial thromboplastin time are elevated while the platelet count is decreased, typical for disseminated intravascular coagulopathy. The lack of vonWillebrand factor leads to impaired platelet function.

34 A 55-year-old man has had episodes of prolonged epistaxis in the past 6 months. On examination he has occult blood detected in his stool. Coagulation studies show that his prothrombin time is elevated, but his partial thromboplastin time, platelet count, and platelet function are all normal. When his plasma is mixed with an equal amount of normal plasma, the prothrombin time corrects to normal. Which of the following underlying diseases is most likely to be associated with his findings? □ (A) Nonalcoholic steatohepatitis □ (B) Hemophilia A □ (C) Antiphospholipid syndrome □ (D) Factor V Leiden mutation □ (E) Sepsis with Escherichia coli □ (F) Von Willebrand disease

34 (B) The infant has DiGeorge syndrome, resulting from a chromosome 22q11.2 microdeletion. This is indicated in the metaphase spread by the presence of only three dots because this region is deleted on one chromosome 22, but both number 22 chromosomes are present. With aneuploidy, there is an abnormal number of chromosomes (trisomy, monosomy), and loss or gain of autosomes tends to produce fetal loss except for some cases of trisomies 13, 18, and 21 and monosomy X. A chromosome inversion would shift the marked region to a different part of the same chromosome. In monosomy, only one of a pair of chromosomes is present. A translocation is the swapping of genetic material between two chromosomes.

34 A 9-month-old infant has had numerous viral and fungal infections since birth. On physical examination, no congenital anomalies are noted. Laboratory studies show hypocalcemia. FISH analysis of the infant's cells is performed. A metaphase spread is shown in the figure, with probes to two different regions on chromosome 22. Which of the following cytogenetic abnormalities is most likely to be present? □ (A) Aneuploidy □ (B) Deletion □ (C) Inversion □ (D) Monosomy □ (E) Translocation

34 (C) Glutathione in the cytosol helps to reduce cellular injury from many toxic metabolites and free radicals. ADP is converted to ATP by oxidative and glycolytic cellular pathways to provide energy that drives cellular functions, and a reduction in ATP leaves the cell vulnerable to injury. Calcium influx into a cell promotes injury. NADPH oxidase generates superoxide, which is used by neutrophils in killing bacteria. Nitric oxide synthase in macrophages produces nitric oxide, which aids in destroying organisms undergoing phagocytosis. Protein synthesis in cells depends on mRNA for longer survival and to recover from damage from free radicals. Failure of the sodium pump leads to increased cytosolic sodium and cell swelling with injury.

34 In an experiment, a large amount of a drug is administered to subjects and is converted by cytochrome P-450 to a toxic metabolite. The accumulation of this metabolite leads to increased lipid peroxidation within cells, causing damage to cell membranes and cell swelling. Depletion of which of the following substances by this mechanism within the cytosol exacerbates the cellular injury? □ (A) ADP □ (B) Calcium □ (C) Glutathione □ (D) NADPH oxidase □ (E) Nitric oxide synthase □ (F) mRNA □ (G) Sodium

9 (D) CDK1 controls an extremely important transition point, the G2 to M transition during the cell cycle, which can be regulated by CDK inhibitors. The other checkpoints are regulated by a distinct set of proteins.

9 An experiment analyzes factors involved in the cell cycle during growth factor-induced cellular regeneration in a tissue culture. Cyclin B synthesis is induced; the cyclin B binds and activates cyclin-dependent kinase 1 (CDK1). The active kinase produced by this process is most likely to control progression in which of the following phases of the cell cycle? □ (A) G0 to G1 □ (B) G1 to S □ (C) S to G2 □ (D) G2 to M □ (E) M to G1

35 (E) This patient has psoriatic arthropathy. The arthritis in this condition may resemble rheumatoid arthritis clinically and pathologically. Similar to other spondyloarthropathies, sacroiliitis occurs in patients with psoriatic arthropathy. Hereditary angioedema is associated with a deficiency of complement C1 inhibitor. Cerebral lymphomas are rare, but they can occur in immunodeficiency states, particularly in patients with AIDS. Hemolytic anemia can accompany a wide variety of autoimmune phenomena and can be related to formation of specific RBC antibodies or coating of RBCs with antibody. Keratoconjunctivitis can be seen in Sjögren syndrome as a result of decreased tear production from lacrimal gland inflammation. Sclerodactyly is seen in scleroderma. When extensive, it is usually part of the spectrum of findings in diffuse scleroderma; when it involves only a few areas of the skin (e.g., just the hands), it is more likely a feature of limited scleroderma (CREST syndrome).

35 A 28-year-old woman has a 5-year history of severe psoriasis involving the skin of the face, trunk, and extremities and uveitis. She sees her physician because she has had increasing pain in her hands and left hip for the past 6 months. On physical examination, there is decreased range of motion of the left hip and the distal interphalangeal joints. Her fingers have a sausage-like appearance. Laboratory studies show that she is positive for HLA-B27, but ANA and rheumatoid factor test results are negative. Which of the following complications is most characteristic of this patient's illness? □ (A) Angioedema of skin □ (B) Cerebral lymphoma □ (C) Hemolytic anemia □ (D) Keratoconjunctivitis □ (E) Sacroiliitis □ (F) Sclerodactyly

35 (F) Heat-shock proteins provide for a variety of cellular "housekeeping" activities, including recycling and restoration of damaged proteins and removal of denatured proteins. Ubiquitin targets denatured proteins and facilitates their binding to proteasomes, which then break down the proteins to peptides. ADP increases when ATP is depleted, helping to drive anaerobic glycolysis. Cytosolic calcium levels may increase with cell injury that depletes ATP; the calcium activates phospholipases, endonucleases, and proteases, which damage the cell membranes, structural proteins, and mitochondria. Caspases are enzymes that facilitate apoptosis. Granzyme B is released from cytotoxic T lymphocytes and triggers apoptosis. Hydrogen peroxide is one of the activated oxygen species generated under conditions of cellular ischemia, producing nonspecific damage to cellular structures, particularly membranes.

35 An experiment is conducted in which cells in tissue culture are subjected to high levels of ultraviolet radiant energy. Electron microscopy shows cellular damage in the form of increased cytosolic aggregates of denatured proteins. In situ hybridization reveals that protein components in these aggregates also are found in proteasomes. Which of the following substances is most likely to bind to the denatured proteins, targeting them for catabolism by cytosolic proteasomes? □ (A) Adenosine monophosphate □ (B) Calcium □ (C) Caspase □ (D) Granzyme B □ (E) Hydrogen peroxide □ (F) Ubiquitin

3 6 (D) This patient has systemic lupus erythematosus (SLE). Many patients with SLE have glomerulonephritis and eventually develop renal failure. Blindness is uncommon in SLE. Raynaud phenomenon is associated with many autoimmune diseases, but it is most troublesome in scleroderma. Although synovial inflammation is common in SLE, joint deformity is rare. Libman-Sacks endocarditis associated with SLE tends to be nondeforming and limited, and there is minimal valve damage. It is now uncommon because of the use of corticosteroid therapy in the treatment of SLE.

36 A 26-year-old woman has had bouts of joint pain for the past 2 years. She also has a rash on the cheeks and bridge of the nose. On physical examination, there is no joint swelling or deformity, although generalized lymphadenopathy is present. Laboratory studies indicate anemia, leukopenia, a polyclonal gammopathy, and proteinuria. The serum ANA test result is positive at a titer of 1 : 1024 with a rim pattern identified by immunofluorescence. The light microscopic and immunofluorescent (with antibody to IgG) appearances of a skin biopsy specimen are shown in the figure. Which of the following is the best information to give this patient about her disease? □ (A) Blindness is likely to occur within 5 years □ (B) Avoid exposure to cold environments □ (C) Joint deformities will eventually occur □ (D) Chronic renal failure is likely to occur □ (E) Cardiac valve replacement will eventually be required

36 (C) Autophagy is a form of cellular downsizing in response to stress, as the cell consumes itself, by upregulating Atgs genes. Lipofuscin granules are residual bodies left over from this process. There may be eventual cell death triggered by autophagy, but by a different mechanism than apoptosis, a form of single cell necrosis in which cell fragmentation occurs. Cancer cells acquire the ability to avoid autophagy, perhaps by downregulating PTEN gene expression, and maintain a survival advantage even as the patient is dying. There is slow autophagy with aging, but autophagy is accelerated with stressors such as malnutrition and chronic disease. Hyaline is a generic term for intracellular or extracellular protein accumulations appearing pink and homogenous with H&E staining. Karyorrhexis is nuclear fragmentation in a necrotic cell.

36 A 71-year-old man diagnosed with pancreatic cancer is noted to have decreasing body mass index. His normal cells comprising skeletal muscle undergo atrophy by sequestering organelles and cytosol in a vacuole followed by fusion with a lysosome. However, the cancer continues to increase in size. Which of the following processes is most likely occurring in the normal cells but inhibited in the cancer cells of this man? □ (A) Aging □ (B) Apoptosis □ (C) Autophagy □ (D) Hyaline change □ (E) Karyorrhexis

37 (C) Excessive iron ingestion, particularly in a child, can overwhelm the bodys ability to bind the absorbed free iron with the transport protein transferrin. The free iron contributes to generation of free radicals via the Fenton reaction. Ascorbic acid (vitamin C) and vitamin E both act as antioxidants to protect against free radical injury. Hemosiderin is a storage form of iron from excess local or systemic accumulation of ferritin, and by itself does not cause cell injury until large amounts are present, as with hemochromatosis. Nitric oxide generated within macrophages can be utilized to kill microbes. It can be converted to a highly reactive peroxynitrite anion. Superoxide dismutase helps break down superoxide anion to hydrogen peroxide, thus scavenging free radicals.

37 A 5-year-old child ingests 50 iron tablets, each with 27 mg of iron. Within 6 hours the child develops abdominal pain and lethargy. On physical examination he is hypotensive. Laboratory studies show metabolic acidosis. Through formation of which of the following compounds is the cell injury in this child most likely mediated? □ (A) Ascorbic acid □ (B) Hemosiderin □ (C) Hydroxyl radical □ (D) Nitric oxide □ (E) Superoxide dismutase

3 7 (B) Blood monocytes express MHC class II antigens and can migrate into tissues to become macrophages. In tuberculosis, these macrophages transform into epithelioid cells, forming a granuloma. Macrophages play an important role in delayed hypersensitivity reactions associated with cell-mediated immunity. Neutrophils are important mainly in acute inflammatory responses, although some neutrophils may be present in a granulomatous reaction. B cells form plasma cells that secrete immunoglobulin on stimulation and are essential to humoral immunity. Natural killer cells can function without prior sensitization. Basophils may play a role in IgE-mediated responses.

37 In a study that examines granuloma formation in the lung in response to infection with Mycobacterium tuberculosis, it is observed that cells within the granuloma express MHC class II antigens. These cells elaborate cytokines that promote fibroblastic production of collagen within the granuloma. These class II antigen-bearing cells are most likely derived from which of the following peripheral blood leukocytes? □ (A) Neutrophil □ (B) Monocyte □ (C) B cell □ (D) Natural killer cell □ (E) Basophil

38 (F) The reverse transcriptase gene of HIV undergoes mutation on average once per 2000 replications, a very high rate, which can account for the appearance of drug resistance. The absence of CD40 ligand interaction with CD40 explains the hyper-IgM syndrome. Chemokine receptors are important in facilitating initial HIV entry into cells, and mutations in these receptors may help explain variable susceptibility to and progression of HIV infection. The cytokine receptor γ chain is abnormal in severe combined immunodeficiency. The p24 antigen is a component of the HIV virion and is used to detect infection, but it is not a target of drug therapy. Protein tyrosine kinases are involved in signal transduction. They can be abnormal in conditions such as Bruton agammaglobulinemia, but not in patients receiving HIV drug therapy; ritonavir is an HIV protease inhibitor.

38 A 38-year-old man has been infected with HIV for the past 8 years. He has been receiving highly active antiretroviral therapy for the past 18 months with a regimen that includes zidovudine, stavudine, and ritonavir. His HIV-1 RNA level initially decreased to less than 50 copies/μL after initiation of therapy; the current level is 5120 copies/μL. A mutation in the gene for which of the following substances is most likely to have occurred? □ (A) CD40 ligand □ (B) Chemokine receptor □ (C) Cytokine receptor γ chain □ (D) p24 antigen □ (E) Protein tyrosine kinase □ (F) Reverse transcriptase

38 (C) The one sure way to increase life span is calorie restriction. But why do without the things we like to do without them longer? Dietary excesses lead to increased morbidity with reduced quality of life, as well as mortality, from chronic diseases such as diabetes mellitus. The histone deacetylase activity of sirtuins may promote transcription of genes encoding for proteins that increase metabolic activity and inhibit effects of free radicals. Red wines have been shown to increase sirtuins, but dont drink too much! Moderation is the key. Glutathione promotes free radical breakdown, though chronic excessive alcohol consumption depletes hepatocyte glutathione. Caspases trigger apoptosis and cell death. Telomerases aid in promoting continued cell division, but cannot be altered by lifestyle, and turning them on is one feature of neoplasia. Ubiquitin is a peptide that is part of the ubiquitin-proteasome pathway of protein degradation seen with nutrient deficiencies, so when you eat less, be sure to eat a balanced diet.

38 A proponent of Malbec, Syrah, and Merlot wines (all reds) touts their contribution to longevity, but this wine aficionado also controls the caloric content of his diet such that his body mass index is <22. This lifestyle promotes increased insulin sensitivity and glucose utilization. He fully expects to live long because he has read that caloric restriction prolongs life. In this man, which of the following proteins will most likely mediate the effect of calorie restriction upon increased longevity? □ (A) Caspase □ (B) Glutathione □ (C) Sirtuin □ (D) Telomerase □ (E) Ubiquitin

3 9 (D) This patient is experiencing a major transfusion reaction resulting from a type II hypersensitivity reaction. The patient's serum contains naturally occurring antibodies to the incompatible donor RBCs. They attach to the donor RBCs and induce complement activation that results in generation of the C5-9 membrane attack complex. Major transfusion reactions are rare, and most result from clerical errors. Tumor necrosis factor α is not part of hypersensitivity reactions. Natural killer cell lysis is seen with antibody-mediated diseases. Antigen-antibody complex formation is typical of a type III hypersensitivity reaction. Mast cells degranulate with antigen attachment to IgE in type I hypersensitivity reactions.

39 Laboratory tests are ordered for two hospitalized patients. During the phlebotomy procedure, the Vacutainer tubes drawn from these patients are mislabeled. One of the patients receives a blood transfusion later that day. Within 1 hour after the transfusion of RBCs begins, the patient becomes tachycardic and hypotensive and passes pink-colored urine. Which of the following statements best describes how this reaction is mediated? □ (A) Release of tumor necrosis factor α into the circulation □ (B) Antibody-dependent cellular cytotoxicity by natural killer cells □ (C) Antigen-antibody complex deposition in glomeruli □ (D) Complement-mediated lysis of RBCs □ (E) Mast cell degranulation

4 (B) These are features of the hyper-IgM syndrome, which results from lack of isotype switching from IgM to other immunoglobulins. Patients are particularly susceptible to Pneumocystis and to bacterial infections. The abnormal IgM antibodies in excess can attach to circulating cells and lead to cytopenias. An absence of adenosine deaminase characterizes a form of severe combined immunodeficiency. The deletion of chromosome 22q11 is a feature of the DiGeorge anomaly, which affects T cell differentiation and maturation. HIV infection can be accompanied by opportunistic infections, particularly Pneumocystis, but abnormal immunoglobulin production generally is not seen. A lack of IgA production alone is seen with selective IgA deficiency. Mutations in the BTK gene account for Bruton agammaglobulinemia, in which levels of all immunoglobulins are reduced.

4 A 12-year-old boy has had multiple recurrent infections for the past 10 years, including Pneumocystis carinii pneumonia, Streptococcus pneumoniae otitis media, and Pseudomonas aeruginosa urinary tract infection. On physical examination, he has a temperature of 38.5°C and pharyngeal erythema with exudate. Laboratory studies show hemoglobin, 9.1 g/dL; hematocrit, 27.6%; platelet count, 130,900/mm3; and WBC count, 3440/mm3 with 47% segmented neutrophils, 3% bands, 40% lymphocytes, and 10% monocytes. Serum immunoglobulin levels are IgG, 88 mg/dL; IgM, 721 mg/dL; and IgA, undetectable. A peripheral blood smear shows nucleated RBCs. Which of the following immunological defects is most likely to produce this disease? □ (A) Absence of adenosine deaminase □ (B) Abnormal CD40-CD40L interaction □ (C) Deletion of chromosome 22q11 □ (D) HIV infection □ (E) Lack of IgA production by B lymphocytes □ (F) Mutation in the BTK gene

4 (F) This woman has McArdle syndrome, a form of glycogen storage disease with onset in young adulthood. In this disorder, a deficiency of muscle phosphorylase enzyme causes glycogen to accumulate in skeletal muscle. Because strenuous exercise requires glycogenolysis and use of anaerobic metabolism, muscle cramps ensue, and the blood lactate level does not rise. Myoglobinuria is seen in about half of cases. A lack of the muscle membrane protein dystrophin characterizes Duchenne muscular dystrophy. A fibrillin gene mutation can lead to Marfan syndrome. Glucose-6- phosphatase deficiency leads to von Gierke disease, characterized by hepatomegaly, renomegaly, and impaired gluconeogenesis leading to hypoglycemia and hyperlipidemia. Hexosaminidase A deficiency occurs in Tay-Sachs disease, causing severe neurologic impairment, poor motor development, and blindness beginning in infancy. Lysosomal glucosidase deficiency is seen in Pompe disease, characterized by marked cardiomegaly and heart failure beginning in infancy. Abnormal spectrin, a red blood cell membrane cytoskeletal protein, leads to a condition known as hereditary spherocytosis.

4 A 25-year-old woman stops going to her aerobic exercise class because of severe muscle cramps that have occurred during every session for the past 2 months. Several hours after each session, she notices that her urine is a brown color. On physical examination, she has normal muscle development and strength. An inherited defect in which of the following substances is most likely to explain these findings? □ (A) Dystrophin □ (B) Fibrillin □ (C) Glucose-6-phosphatase □ (D) Hexosaminidase □ (E) Lysosomal glucosidase □ (F) Muscle phosphorylase □ (G) Spectrin

4 (A) Macrophages, present in such lesions, play a prominent role in the healing process. Activated macrophages can secrete various cytokines that promote angiogenesis and fibrosis, including platelet-derived growth factor, fibroblast growth factor, interleukin-1, and tumor necrosis factor. Plasma cells can secrete immunoglobulins and are not instrumental to healing of an area of tissue injury. Neutrophils are most numerous within the initial 48 hours after infarction, but are not numerous after the first week. Eosinophils are most prominent in allergic inflammations and in parasitic infections. Epithelioid cells, which are aggregations of activated macrophages, are typically seen with granulomatous inflammation. The healing of acute inflammatory processes does not involve granulomatous inflammation.

4 A 63-year-old man develops worsening congestive heart failure 2 weeks after an acute myocardial infarction. An echocardiogram shows a markedly decreased ejection fraction. He dies 1 day later. At autopsy, a section of the infarct shows that the necrotic myocardium has largely been replaced by capillaries, fibroblasts, and collagen. Various inflammatory cells are present. Which of the following inflammatory cell types in this lesion plays the most important role in the healing process? □ (A) Macrophages □ (B) Plasma cells □ (C) Neutrophils □ (D) Eosinophils □ (E) Epithelioid cells

4 (B) The valve is stenotic because of nodular deposits of calcium. The process is "dystrophic" because calcium deposition occurs in damaged tissues. The damage in this patient is a result of the wear and tear of aging. Amyloid deposition in the heart typically occurs within the myocardium and the vessels. The amount of lipofuscin increases within myocardial fibers (not valves) with aging. Hereditary hemochromatosis is a genetic defect in iron absorption that results in extensive myocardial iron deposition (hemosiderosis). Fatty change is uncommonly seen in myocardium, but infiltration of fat cells between myofibers can occur.

4 A 72-year-old man died suddenly from congestive heart failure. At autopsy, the heart weighed 580 g and showed marked left ventricular hypertrophy and minimal coronary arterial atherosclerosis. A serum chemistry panel ordered before death showed no abnormalities. Which of the following pathologic processes best accounts for the appearance of the aortic valve seen in the figure? □ (A) Amyloidosis □ (B) Dystrophic calcification □ (C) Lipofuscin deposition □ (D) Hemosiderosis □ (E) Fatty change

4 (E) The figure shows a so-called nutmeg liver caused by chronic passive congestion. The elevated enzyme levels suggest that the process is so severe that hepatic centrilobular necrosis also has occurred. The physical findings suggest right-sided heart failure. The regular pattern of red lobular discoloration seen in the figure is unlikely to occur in hemorrhage from thrombocytopenia, characterized by petechiae and ecchymoses. A portal vein thrombus would diminish blood flow to the liver, but it would not be likely to cause necrosis because of that organ's dual blood supply. Hepatic congestion is not directly related to renal failure, and hepatorenal syndrome has no characteristic gross appearance. Biliary tract obstruction would produce bile stasis (cholestasis) with icterus.

4 For the past week, a 61-year-old man has had increasing levels of serum AST and ALT. On physical examination, he has lower leg swelling with grade 2+ pitting edema to the knees and prominent jugular venous distention to the level of the mandible. Based on the gross appearance of the liver, seen in the figure, which of the following underlying conditions is most likely to be present? □ (A) Thrombocytopenia □ (B) Portal vein thrombosis □ (C) Chronic renal failure □ (D) Common bile duct obstruction □ (E) Congestive heart failure

5 (A) Aspirin (acetylsalicylic acid) blocks the cyclooxygenase pathway of arachidonic acid metabolism, which leads to reduced prostaglandin generation. Prostaglandins promote vasodilation at sites of inflammation. Chemotaxis is a function of various chemokines, and complement C3b may promote phagocytosis, but neither is affected by aspirin. Leukocyte emigration is aided by various adhesion molecules. Leukocyte release from the marrow can be driven by the cytokines Robbins & Cotran Review of Pathology Pg. 25 interleukin-1 and tumor necrosis factor.

5 A 10-year-old child developed a sore throat and fever over 24 hours. Physical examination shows pharyngeal erythema and swelling. Laboratory findings include leukocytosis. The child is given acetylsalicylic acid (aspirin). Which of the following features of the inflammatory response is most affected by this drug? □ (A) Vasodilation □ (B) Chemotaxis □ (C) Phagocytosis □ (D) Emigration of leukocytes □ (E) Release of leukocytes from bone marrow

40 (C) Pneumocystis jiroveci pneumonia is a common finding in patients with AIDS. This patient's low CD4+ count is characteristic of AIDS. Antineutrophil cytoplasmic autoantibody (C-ANCA or P-ANCA) can be seen in patients with vasculitis. Rheumatoid factor is present in most patients with rheumatoid arthritis; however, significant immunosuppression is not seen, unless the patients are treated with highly potent immunosuppressive drugs such as cyclosporine. The antistreptolysin O titer is elevated in patients with rheumatic fever, but there is no serious immunosuppression. The ANA test result is positive in various autoimmune diseases, but a decrease in CD4+ count is not typical of such conditions.

40 A 45-year-old man has had a fever, cough, and worsening dyspnea for the past few days. On physical examination, his temperature is 39.2°C. Auscultation of the chest shows decreased breath sounds over all lung fields. A bronchoalveolar lavage is performed, and the fluid obtained yields cysts of Pneumocystis carinii. Laboratory studies show a CD4+ lymphocyte count of 135/μL; total serum globulin concentration of 2.5 g/dL; and WBC count of 7800/mm3 with 75% segmented neutrophils, 8% bands, 6% lymphocytes, 10% monocytes, and 1% eosinophils. Which of the following serologic laboratory findings is most likely to be positive in this patient? □ (A) Antineutrophil cytoplasmic autoantibody □ (B) Rheumatoid factor □ (C) Antibodies to HIV □ (D) Antistreptolysin O □ (E) Anti-double-stranded DNA antibody

4 1 (B) This child had severe combined immunodeficiency (SCID), which is now treated with allogeneic bone marrow transplantation. The transplanted stem cells in the bone marrow give rise to normal T and B cells. Half of SCID cases are caused by an X-linked mutation in the common γ chain for cytokine receptors, and the rest are due to autosomal recessive mutations in the gene encoding for adenosine deaminase, which leads to accumulation of metabolites toxic to lymphocytes. An abnormal CD40 ligand interaction with CD40 leads to lack of isotype switching in patients with hyper-IgM syndrome. The BTK gene product is required for differentiation of pro-B and pre-B cells, and a mutation leads to agammaglobulinemia. Individuals lacking C2 have some increase in infections, but mainly develop a disease resembling systemic lupus erythematosus. The 22q11 deletion is seen in infants with DiGeorge anomaly and results in lack of T cell development. HIV infection leads to many opportunistic infections, which sometimes occur in infancy and early childhood, but it is mainly CD4+ lymphocytes that are diminished.

41 A 14-month-old child has had multiple infections since birth, including pneumonia with Pseudomonas aeruginosa, adenovirus, and Aspergillus fumigatus; diarrhea with Isospora belli; otitis media with Haemophilus influenzae; and urinary tract infection with Candida albicans. Laboratory studies show hemoglobin, 13.2 g/dL; hematocrit, 39.7%; platelet count, 239,100/mm3; and WBC count, 3450/mm3 with 85% segmented neutrophils, 6% bands, 2% lymphocytes, and 7% monocytes. Serum immunoglobulin levels are IgG, 118 mg/dL; IgM, 14 mg/dL; and IgA, 23 mg/dL. The child dies of pneumonia. At autopsy, a hypoplastic thymus, small lymph nodes that lack germinal centers, and scant gut-associated lymphoid tissue are seen. Which of the following is the most likely cause of this disease? □ (A) Abnormal CD40 ligand □ (B) Adenosine deaminase deficiency □ (C) BTK gene mutation C2 □ (D) Complement component deficiency □ (E) Chromosome 22q11 deletion □ (F) HIV infection

42 (B) About 70% to 90% of juvenile rheumatoid (idiopathic) arthritis cases resolve without joint deformity. In contrast to rheumatoid arthritis, juvenile idiopathic arthritis tends to involve lower and larger joints, and rheumatoid factor is often absent. Systemic sclerosis is a disease of adults that may have features resembling early rheumatoid arthritis, but joint destruction is rare. Psoriatic arthropathy is a disease of adults with features similar to rheumatoid arthritis, but joint involvement is more irregular. Ankylosing spondylitis occurs in older adults and principally affects the vertebral column. Reiter syndrome occurs in young to middle-aged adults; its major features are urethritis, arthritis, and conjunctivitis.

42 A 9-year-old girl has experienced pain and swelling of the elbows and knees and a fever for 1 month. On physical examination, her temperature is 38°C. There is decreased range of motion at the elbows and knees, with joint swelling and warmth. Laboratory studies show hemoglobin, 13.4 g/dL; hematocrit, 40.3%; platelet count, 288,200/mm3; and WBC count, 12,560/mm3. The ANA test result is positive, but results for rheumatoid factor, Scl-70, and SS-A serologies are negative. The girl's symptoms abate after 4 years and never recur. Which of the following is the most likely diagnosis? □ (A) Systemic sclerosis □ (B) Juvenile rheumatoid arthritis □ (C) Psoriatic arthropathy □ (D) Ankylosing spondylitis □ (E) Reiter syndrome

4 3 (E) This patient is susceptible to bacterial, fungal, and viral infections and most likely has severe combined immunodeficiency (SCID). The autosomal recessive pattern of inheritance implicates adenosine deaminase (ADA) deficiency rather than mutations in the γ chain of cytokine receptors. Low ADA levels in the leukocytes are diagnostic. The other listed options are relevant to the work-up of primary immunodeficiencies, but they are not specific to SCID.

43 A 9-month-old child has a history of recurrent infections with multiple agents, including cytomegalovirus, Candida albicans, Staphylococcus aureus, and Staphylococcus epidermidis. A careful family history and pedigree analysis show this to be a genetic disorder that is inherited in an autosomal recessive pattern. Which of the following laboratory studies is likely to be most useful in establishing the underlying mechanism of immunodeficiency in this infant? □ (A) Quantitative serum immunoglobulin levels □ (B) Enumeration of B cells in blood □ (C) Enumeration of CD3+ cells in blood □ (D) Tests of neutrophil function □ (E) Adenosine deaminase levels in leukocytes

44 (D) Streptococcal M proteins cross-react with cardiac glycoproteins, resulting in rheumatic heart disease, a form of autoimmunity. Breakdown of T-cell anergy usually occurs when localized tissue damage and inflammation cause upregulation of costimulatory molecules on the target tissues. This is a possible mechanism of autoimmunity in the brain and in pancreatic β cells. Polyclonal lymphocyte activation may be caused by microbial products such as endotoxin or bacterial superantigens. Release of sequestered antigens can cause autoimmunity; this mechanism is likely in autoimmune uveitis. Failure of T cell-mediated suppression has not yet been shown to cause any autoimmune disease; it remains a potential mechanism. The other listed options are not major immune responses to streptococcal infection.

44 A 12-year-old girl has complained of a sore throat for the past 3 days. On physical examination, she has a temperature of 38.4°C and pharyngeal erythema with minimal exudate. A throat culture grows group A β-hemolytic streptococcus. The pharyngitis resolves, but 3 weeks later, the girl develops fever and chest pain. Her antistreptolysin O titer is 1 : 512. Which of the following immunological mechanisms has most likely produced the chest pain? □ (A) Breakdown of T-cell anergy □ (B) Polyclonal lymphocyte activation □ (C) Release of sequestered antigens □ (D) Molecular mimicry □ (E) Failure of T cell-mediated suppression

45 (C) Virus-infected cells are recognized and killed by cytotoxic CD8+ T cells. The T-cell receptor on the CD8+ T cells binds to the complex of viral peptide and MHC class I molecules on the surface of the infected cell. Natural killer (NK) cells also recognize MHC class I molecules with self-peptides. This recognition inhibits NK cell killing. The other listed options are not the major immune response to hepatitis infection.

45 Over the past week, a 32-year-old man has experienced nausea and vomiting and has become mildly icteric. On physical examination, his temperature is 37.4°C. Laboratory studies show serum AST of 208 U/L and ALT of 274 U/L. Serologic findings for HBsAg and HBcAb are positive. A liver biopsy specimen examined microscopically shows focal death of hepatocytes with a portal infiltrate composed mainly of lymphocytes. Which of the following is the most likely mechanism by which the liver cell injury occurs under these conditions? □ (A) Recognition of HBsAg by the CD8 molecule of T cells □ (B) Recognition of an antigenic peptide presented by MHC class I molecules to natural killer cells □ (C) Recognition of an antigenic peptide presented by MHC class I molecule to CD8+ cells □ (D) Destruction of HBsAg-expressing cells by anti-HBs IgG antibody □ (E) Apoptosis of the liver cells by cytokines released by activated macrophages

4 6 (D) Mixed connective tissue disease can have features of systemic lupus erythematosus (SLE), polymyositis, rheumatoid arthritis, and Sjögren syndrome. In contrast to SLE or diffuse scleroderma, serious renal disease is unlikely. Dermatomyositis causes muscle pain, and the rash is typically a subtle heliotrope rash with a violaceous appearance to the eyelids; Jo-1 antibody is a more typical finding. Discoid lupus erythematosus (DLE) is characterized by a rash similar to SLE, but with immune complex deposition only in sun-exposed areas of the skin, a positive ANA test result in a few cases, absence of anti-Sm or anti-double-stranded DNA antibodies, and absence of serious renal disease. Some cases of DLE can progress to SLE. In limited scleroderma (CREST syndrome), anticentromeric antibody is often present. In Reiter syndrome (with conjunctivitis, arthritis, and nongonococcal urethritis), there is often a positive serology for HLA-B27. In Sjögren syndrome, antibodies to SS-A and SS-B are often present. The anti-Sm or anti-double-stranded DNA antibodies are more specific for SLE.

46 A 41-year-old man has been bothered by a feeling of dryness in his mouth for the past 3 years. During this time, erythematous rashes have appeared on the skin of his face and upper neck. In the past 6 months, he has developed arthralgias. On physical examination, he is afebrile. There are no joint deformities. Laboratory findings include a positive ANA test result, with a speckled pattern, and high titers of antibodies to U1-ribonucleoprotein. The serum creatinine is 1.1 mg/dL, and the urea nitrogen is 17 mg/dL. Which of the following diseases is most likely to produce these findings? □ (A) Dermatomyositis □ (B) Discoid lupus erythematosus □ (C) Limited scleroderma □ (D) Mixed connective tissue disease □ (E) Reiter syndrome □ (F) Sjögren syndrome □ (G) Systemic lupus erythematosus

47 (F) This patient has Goodpasture syndrome, in which an antibody is directed against type IV collagen in basement membranes of the glomeruli and in the lung. This is a form of type II hypersensitivity reaction. The antibodies attach to the basement membrane and fix complement, damaging the glomeruli. Anti-double-stranded DNA antibodies have specificity for systemic lupus erythematosus (SLE), whereas antihistone antibodies are characteristic of drug-induced SLE. Anti-Jo-1 antibody is found in dermatomyositis and polymyositis. The anti-U1-ribonucleoprotein antibody is seen in mixed connective tissue diseases. Anti-SS-A antibody is seen in Sjögren syndrome. Antiphospholipid antibodies are sometimes called "lupus anticoagulant" because they may appear in SLE; such patients have coagulopathies with thrombosis or bleeding, or both.

47 A 28-year-old man has had hemoptysis and hematuria for the past 2 days. On physical examination, his temperature is 36.8°C, pulse is 87/min, respirations are 19/min, and blood pressure is 150/90 mm Hg. Laboratory studies show creatinine of 3.8 mg/dL and urea nitrogen of 35 mg/dL. Urinalysis shows 4+ hematuria, 2+ proteinuria, and no glucose. A renal biopsy specimen examined microscopically shows glomerular damage and linear immunofluorescence with labeled anticomplement and anti-IgG antibody. Which of the following autoantibodies has the greatest specificity for this patient's condition? □ (A) Anti-double-stranded DNA □ (B) Antihistone □ (C) Anti-Jo-1 □ (D) Anti-U1-ribonucleoprotein □ (E) Anti-SS-A □ (F) Anti-basement membrane □ (G) Antiphospholipid

48 (A) As HIV infection progresses, there is continuing, gradual loss of CD4+ cells. The stage of clinical AIDS is reached when the peripheral CD4+ count decreases to less than 200/μL, which usually occurs over 8 to 10 years. At this point, the risk of development of opportunistic infections and neoplasms typical of AIDS increases greatly. The extent of viremia also is an indication of the progression of HIV infection; an increase in HIV-1 RNA levels is seen as immunological containment of HIV fails. In HIV infection, the numbers of CD8+ lymphocytes tend to be maintained. Cells of the granulocytic series, including eosinophils, are relatively unaffected, although patients with AIDS may have cytopenias. Follicular dendritic cells can be infected by HIV and pass the virions to CD4+ cells and macrophages, but the follicular dendritic cells and the macrophages are not destroyed by the virus and become a reservoir for infection. The natural killer cells and plasma cells are not directly affected by HIV.

48 A 40-year-old man has been infected with HIV for the past 10 years. During this time, he has had several bouts of oral candidiasis, but no major illnesses. He is now diagnosed with Kaposi sarcoma involving the skin. He has had a 7-kg weight loss in the past 6 months. Laboratory studies show the HIV-1 RNA viral load is currently 60,000 copies/mL. Which of the following types of cells is most depleted in his lymph nodes? □ (A) CD4+ lymphocyte □ (B) CD8+ lymphocyte □ (C) Eosinophil □ (D) Follicular dendritic cell □ (E) Macrophage □ (F) Natural killer cell □ (G) Plasma cell

5 (E) The most important and the most common cause of venous thrombosis is vascular stasis, which often occurs with immobilization. Turbulent blood flow may promote thrombosis, but this risk factor is more common in fast-flowing arterial circulation. Nitric oxide is a vasodilator and an inhibitor of platelet aggregation. Aspirin inhibits platelet function and limits thrombosis. Calcium is a cofactor in the coagulation pathway, but an increase in calcium has minimal effect on the coagulation process.

5 A 55-year-old woman has had discomfort and swelling of the left leg for the past week. On physical examination, the leg is slightly difficult to move, but on palpation, there is no pain. A venogram shows thrombosis of deep left leg veins. Which of the following mechanisms is most likely to cause this condition? □ (A) Turbulent blood flow □ (B) Nitric oxide release □ (C) Ingestion of aspirin □ (D) Hypercalcemia □ (E) Immobilization

5 (E) Cholesterol is a form of lipid commonly deposited within atheromas in arterial walls, imparting a yellow color to these plaques. Glycogen is a storage form of carbohydrate seen mainly in liver and muscle. Lipofuscin is a brown pigment that increases with aging in cell cytoplasm, mainly in cardiac myocytes and in hepatocytes. Hemosiderin is a storage form of iron that appears in tissues of the mononuclear phagocyte system (e.g., marrow, liver, spleen), but can be widely deposited with hereditary hemochromatosis. Immunoglobulin occasionally may be seen as rounded globules in plasma cells (i.e., Russell bodies).

5 A 69-year-old woman has had transient ischemic attacks for the past 3 months. On physical examination, she has an audible bruit on auscultation of the neck. A right carotid endarterectomy is performed. The curetted atheromatous plaque has a grossly yellow-tan, firm appearance. Microscopically, which of the following materials can be found in abundance in the form of crystals that produce long, cleft-like spaces? □ (A) Glycogen □ (B) Lipofuscin □ (C) Hemosiderin □ (D) Immunoglobulin □ (E) Cholesterol

5 (A) At 1 week, wound healing is incomplete, and granulation tissue is still present. More collagen is synthesized in the following weeks. Wound strength peaks at about 80% by 3 months. Type IV collagen is found in basement membranes.

5 A cesarean section is performed on a 20-year-old woman to deliver a term infant, and the lower abdominal incision is sutured. The sutures are removed 1 week later. Which of the following statements best describes the wound site at the time of suture removal? □ (A) Granulation tissue is still present □ (B) Collagen degradation exceeds synthesis □ (C) Wound strength is 80% of normal tissue □ (D) Type IV collagen predominates □ (E) No more wound strength will be gained

5 (D) Most inborn errors of metabolism involve mutations in genes encoding for enzymes. Because one active allele produces half the needed enzyme, this is likely sufficient to avoid disease. Inheritance of two mutant alleles, one from each parent, is required for appearance of disease, so the pattern is autosomal recessive, and the recurrence risk is 25%. Most autosomal recessive genes are infrequent in the population, so a family history is unlikely. The standard recurrence risk for any pregnancy is 3%. The recurrence risk is increased to 7% in diseases such as diabetes mellitus, or when a syndrome is identified without a defined inheritance pattern, or with multifactorial inheritance. Autosomal dominant conditions usually result from mutations in genes encoding for structural genes and have a recurrence risk of 50%.

5 A female infant shows failure to thrive and failure to achieve developmental milestones. A pedigree reveals only this child is affected out of four generations on both sides of the family. Tissue fibroblasts obtained from this child shows a 46,XX karyotype. Cultured fibroblasts have increased amounts of an intermediate product in a multiple enzymic metabolic pathway from substrate to end product. What is the most likely recurrence risk for this condition in siblings of this infant? □ (A) 3% □ (B) 8% □ (C) 15% □ (D) 25% □ (E) 50% □ (F) 100%

5 (B) This girl has experienced a systemic anaphylactic reaction from a type I hypersensitivity reaction. Epinephrine is the fastest acting agent to treat this life-threatening condition. Cyclosporine is used to minimize transplant rejection. Penicillin is an antibiotic that often induces a type I hypersensitivity reaction. Glucocorticoids can reduce immune reactions, although this occurs over days to weeks, not minutes. Methotrexate is useful in the treatment of graft-versus-host disease.

5 Within minutes after a bee sting, a 15-year-old girl suddenly has difficulty breathing. There is marked urticaria and marked edema of the hand that was stung. Which of the following is the best pharmacologic agent to treat these signs and symptoms? □ (A) Cyclosporine □ (B) Epinephrine □ (C) Penicillin □ (D) Glucocorticoids □ (E) Methotrexate

50 (B) Amyloidosis is most often caused by excessive light-chain production with plasma cell dyscrasias such as multiple myeloma (AL amyloid). Chronic inflammatory conditions, such as rheumatic fever, ankylosing spondylitis, and systemic sclerosis, also may result in amyloidosis (AA amyloid), but not in secretion of light chains in urine (i.e., Bence Jones proteinuria). Immunoglobulin levels generally are reduced in patients with common variable immunodeficiency.

50 A 61-year-old man has had increasing malaise for the past 4 months. On physical examination, he is afebrile and has mild muscle wasting. Laboratory studies show serum creatinine of 4.5 mg/dL and urea nitrogen of 44 mg/dL. Urine dipstick analysis shows no blood, protein, or glucose, but a specific test for Bence Jones proteins yields a positive result. A renal biopsy specimen has the microscopic appearance shown in the figure. Which of the following underlying conditions is most likely to be present in this patient? □ (A) Rheumatic fever □ (B) Multiple myeloma □ (C) Ankylosing spondylitis □ (D) Systemic sclerosis □ (E) Common variable immunodeficiency

51 (D) Perivascular accumulation of T cells, particularly CD4+ cells, is typical of delayed hypersensitivity skin reactions, driven by a TH1 response mediated largely by release of the cytokine interleukin-2. Systemic anaphylaxis typically occurs within minutes after an encounter with the antigen. Systemic and localized immune complex diseases (serum sickness and Arthus reactions) are type III hypersensitivity reactions; they often exhibit vasculitis. Graft-versus-host disease is characterized by epidermal apoptosis and rash.

51 A 40-year-old laboratory technician accidentally injects a chemical into his skin. The next day, he notes that an area of erythematous, indurated skin is forming around the site of injection. Two days later, the induration measures 10 mm in diameter. A microscopic section from this area, with immunostaining using antibody to CD4, shows many positive lymphocytes. Which of the following immunological reactions is most consistent with this appearance? □ (A) Systemic anaphylaxis □ (B) Arthus reaction □ (C) Graft-versus-host disease □ (D) Delayed-type hypersensitivity □ (E) Serum sickness

52 (A) Acute rejection of kidney transplants occurs weeks, months, or years after transplantation. It is characterized by infiltration with CD3+ T cells that include the CD4+ and CD8+ subsets. These cells damage tubular epithelium by direct cytotoxicity and by release of cytokines, such as interferon-γ, which activate macrophages. The reaction is called acute cellular rejection, and it can be readily treated with corticosteroids. Interstitial and glomerular fibrosis and blood vessel thickening occur in chronic rejection. Fibrinoid necrosis and thrombosis are more typical of hyperacute rejection, which occurs within minutes of placement of the transplant into the recipient. Eosinophils accumulate in acute interstitial nephritis owing to drug reactions. Amyloid derived from serum amyloid-associated protein can occur in chronic infections and inflammation.

52 A 19-year-old woman with chronic renal failure received a cadaver renal transplantation. One month later, she experienced increasing serum creatinine and urea nitrogen levels, and a renal biopsy was performed. She was treated with corticosteroids, and her renal function improved. Which of the following changes was most likely seen in the biopsy specimen before corticosteroid therapy was initiated? □ (A) Interstitial infiltration by CD3+ lymphocytes and tubular epithelial damage □ (B) Extensive fibrosis of the interstitium and glomeruli with markedly thickened blood vessels □ (C) Fibrinoid necrosis of renal arterioles with thrombotic occlusion □ (D) Interstitial infiltration by eosinophils with tubular epithelial damage □ (E) Glomerular deposition of serum amyloid-associated protein

53 (E) This patient has rheumatoid arthritis. The pannus of rheumatoid arthritis leads to joint destruction and ankylosis with marked deformity. There are few other organ-specific lesions, although rheumatoid nodules can be found under the skin over bony prominences and in organs such as the lung and heart. Renal failure is more likely in systemic lupus erythematosus. Aortic dissection is more likely in Reiter syndrome. Ankylosing spondylitis is marked by kyphosis. The risk of malignancies is increased in patients with autoimmune diseases, although malignancies are still uncommon.

53 A 35-year-old woman has had bouts of severe pain and swelling of the small joints of her hands and feet for the past 10 years, although she has had remissions during each of her three pregnancies. She has been bedridden for the past 2 months. On physical examination, there is warmth, swelling, and tenderness of the hands and feet, deformity of the hands, and ulnar deviation and decreased range of motion of metacarpophalangeal and interphalangeal joints. She has no muscle pain or skin rash. A painless 1.5-cm subcutaneous nodule is present behind the elbow joint over the olecranon process. A joint aspirate shows turbid fluid with many neutrophils containing phagocytized immune complexes. Which of the following long-term outcomes of this patient's disease is most likely to occur? □ (A) Chronic renal failure □ (B) Aortic dissection □ (C) Vertebral kyphosis □ (D) Adenocarcinoma of the colon □ (E) Joint deformities

54 (C) This patient has Sjögren syndrome, which primarily involves salivary and lacrimal glands. Antibodies to SS-B are found in 60% to 90% of patients. Anti-double-stranded DNA is a specific autoantibody for systemic lupus erythematosus. Anticentromere antibody is seen in systemic sclerosis. Scl-70 is a marker for diffuse systemic sclerosis. Jo-1 is a marker for polymyositis.

54 A 45-year-old woman has had a burning sensation with increasing blurring of vision for the past 5 years. On physical examination, she has keratoconjunctivitis. Atrophy of the oral mucosa, with buccal mucosal ulcerations, also is present. A biopsy specimen of the lip shows marked lymphocytic and plasma cell infiltrates in minor salivary glands. Which of the following antibodies is most likely to be identified on laboratory testing? □ (A) Anti-double-stranded DNA □ (B) Anticentromere □ (C) SS-B □ (D) Scl-70 □ (E) Jo-1

55 (E) This patient is experiencing a systemic anaphylactic reaction, a form of type I hypersensitivity. IgE is bound to mast cells, after previous sensitization, so that a repeat encounter with the antigen results in mast cell degranulation and the release of mediators, such as histamine, which lead to anaphylaxis. IgE also is important in mediating more localized inflammatory reactions such as allergic rhinitis (hay fever). Other immunoglobulins do not bind so readily to mast cells.

55 A 22-year-old man has had a urethral discharge for the past week. A culture of the exudate from the urethra grows Neisseria gonorrhoeae. He is treated with penicillin G, but within minutes after injection, he develops itching and erythema of the skin. This is quickly followed by severe respiratory difficulty with wheezing and stridor. Which of the following immunoglobulins has most likely become attached to the penicillin G and mast cells to produce these symptoms? □ (A) IgA □ (B) IgG □ (C) IgM □ (D) IgD □ (E) IgE

5 6 (D) These findings point to the X-linked disorder known as Wiskott-Aldrich syndrome, which is characterized by thrombocytopenia, eczema, and decreased IgM. IgA may be increased. As in many immunodeficiency disorders, there is an increased risk of non-Hodgkin lymphoma. Hypocalcemia is seen in neonates with DiGeorge syndrome. Rheumatoid arthritis can complicate isolated IgA deficiency and common variable immunodeficiency, conditions with survival to adulthood. A deficiency of complement component C3 may be complicated by immune complex glomerulonephritis. Dementia can be seen in patients with AIDS.

56 A 4-year-old boy has had recurrent respiratory infections with multiple bacterial and viral pathogens for the past 3 years. On physical examination, he has eczema involving the trunk and extremities. Laboratory findings include a platelet count of 71,000/mm3 and WBC count of 3800/mm3 with 88% segmented neutrophils, 6% bands, 3% lymphocytes, and 3% monocytes. Serum immunoglobulin levels are IgG, 1422 mg/dL; IgM, 11 mg/dL; and IgA, 672 mg/dL. This patient is at an increased risk of developing which of the following conditions? □ (A) Hypocalcemia □ (B) Rheumatoid arthritis □ (C) Glomerulonephritis □ (D) Malignant lymphoma □ (E) Dementia

57 (A) Individuals infected with HIV are infected for life. They can transmit the virus to others via sexual intercourse even if they appear to be well. The average time for the development of AIDS after HIV infection is 8 to 10 years. Seroreversion in HIV infection does not occur. Screening questionnaires and serologic testing can prevent this individual from being a blood donor. HIV infection affects mainly CD4+ lymphocytes, with declining CD4+ counts presaging the development of clinically apparent AIDS. Antibody titers do not predict clinical illness or complications. Progression of HIV disease is monitored by levels of HIV-1 mRNA in the blood and by CD4+ cell counts.

57 A 17-year-old boy has been sexually active for the past 3 years. Serologic testing shows that he is HIV-positive. He is currently healthy and is not an intravenous drug user. Which of the following is the best information to give this patient about his disease? □ (A) You should not have unprotected sex with other individuals □ (B) You will probably develop AIDS within 1 year □ (C) Your HIV test may become negative within 1 year □ (D) As long as you are clinically well, you can donate blood □ (E) The course of your infection is best followed by titers of anti-HIV antibodies

5 8 (C) These findings are characteristic of cardiac amyloidosis. Because of the patient's age, a senile cardiac amyloidosis, resulting from deposition of transthyretin, is most likely. α-Fetoprotein is present during fetal life, but it is best known in adults as a serum tumor marker. β2-Microglobulin contributes to the development of amyloidosis associated with long-term hemodialysis. Calcitonin forms the precursor for amyloid deposited in thyroid medullary carcinomas. Amyloidosis associated with plasma cell dyscrasias results from light-chain production. Although the heart is commonly involved in light-chain amyloidosis, the normal laboratory values and absence of plasma cell collections in the marrow argue against a plasma cell dyscrasia. IgE is not a component of amyloid.

58 A 79-year-old man has experienced worsening congestive heart failure and pulmonary and peripheral edema for the past 4 years. On physical examination, his temperature is 36.9°C, pulse is 70/min, respirations are 16/min, and blood pressure is 120/75 mm Hg. Echocardiography shows cardiomegaly with four-chamber dilation. All laboratory studies, including serum protein electrophoresis and examination of bone marrow smear, are normal. An endomyocardial biopsy specimen has the histologic appearance shown in the figure. Which of the following proteins is most likely to be found in this lesion? □ (A) α-Fetoprotein □ (B) β2-Microglobulin □ (C) Transthyretin □ (D) Calcitonin □ (E) IgE

59 (D) This boy most likely has Bruton agammaglobulinemia, an X-linked primary immunodeficiency marked by recurrent bacterial infections that begin after maternal antibody levels diminish. Selective IgA deficiency is marked by a more benign course, with sinopulmonary infections and diarrhea that are not severe. Deficiency of complement C3 is rare; it leads to greater numbers of infections in children and young adults, but Giardia infections are not a feature of this disease. Lack of cell-mediated immunity is more likely to be seen in HIV infection in children. Although some patients with Bruton agammaglobulinemia can develop features of systemic lupus erythematosus, they generally do not have a positive test result for ANA.

59 A 2-year-old boy has had almost continuous infections since he was 6 months old. These infections have included otitis media, pneumonia, and impetigo. Organisms cultured have included Haemophilus influenzae, Streptococcus pneumoniae, and Staphylococcus aureus. He also has had diarrhea, with Giardia lamblia cysts identified in stool specimens. The family history indicates that an older brother with a similar condition died because of overwhelming infections. The boy's two sisters and both parents are normal. Which of the following laboratory findings would most likely be seen in this boy? □ (A) Absence of IgA □ (B) Decreased complement C3 □ (C) High titer of HIV-1 RNA □ (D) Agammaglobulinemia □ (E) Positive ANA test result

63 (E) The TH17 subset of CD4 cells plays a role in delayed-type hypersensitivity reactions. Many persons react to nickel, particularly with body piercing jewelry. IL-17 may also be useful in recruiting neutrophils to fight bacterial as well as fungal infections such as aspergillosis and candidiasis. IL-2 acts as an autocrine growth factor promoting T cell proliferation. IL-5 activates eosinophils as part of a TH2 response. IL-10 is an immunosuppressive cytokine that diminishes lymphocyte activation. NK cells may secrete interferon-gamma in response to stimulation by IL-12.

63 Persons with sensitivity to body jewelry undergo skin patch testing with nickel compounds. Within 24 to 48 hours there is focal erythema and induration of their skin. A subpopulation of CD4+ T lymphocytes is secreting a cytokine that is recruiting neutrophils and monocytes to this inflammatory reaction, accompanied by itching and pain. Which of the following interleukins is this cytokine most likely to be? □ (A) IL-2 □ (B) IL-5 □ (C) IL-10 □ (D) IL-12 □ (E) IL-17

6 (D) Most congenital malformations, including cleft lip, are not determined by a single gene and may be conditioned by environmental influences. This type of multifactorial inheritance accounts for 20% to 25% of all anomalies noted. Of the remaining options, all are more likely to produce multiple defects and to reduce fetal growth. Most chromosomal anomalies are not compatible with survival; the few fetuses (e.g., with sex chromosome aneuploidies and autosomal triploidies such as 13, 18, and 21) that do survive to term and beyond manifest multiple anomalies. Early amnion disruption may result in clefts, but more severe defects are present (e.g., gastroschisis and missing digits or limbs), and stillbirth is the usual consequence. Maternal malnutrition typically results in an infant who is small for gestational age. Single gene defects account for less than 10% of anomalies noted at birth. Teratogens account for no more than 1% of congenital anomalies.

6 A 19-year-old woman, G3, P2, gives birth after an uncomplicated pregnancy to a term male infant. On physical examination soon after birth, the infant is recorded at the 60th percentile for height and weight. The only abnormal finding is a cleft lip. There is no family history of birth defects, and the mother's other two children are healthy with no apparent abnormalities. Which of the following factors is most likely to influence the appearance of this infant? □ (A) Chromosomal anomaly □ (B) Early amnion disruption □ (C) Maternal malnutrition □ (D) Multifactorial inheritance □ (E) Single gene defect □ (F) Teratogenicity

6 (D) These findings are characteristic of a hypercoagulable state. The patient has antibodies that react with cardiolipin, a phospholipid antigen used for the serologic diagnosis of syphilis. These so-called antiphospholipid antibodies are directed against phospholipid-protein complexes and are sometimes called lupus anticoagulant because they are present in some patients with systemic lupus erythematosus (SLE). Lupus anticoagulant may occur in individuals with no evidence of SLE, however. Patients with lupus anticoagulant have recurrent arterial and venous thrombosis and repeated miscarriages. In vitro, these antibodies inhibit coagulation by interfering with the assembly of phospholipid complexes. In vivo, the antibodies induce a hypercoagulable state by unknown mechanisms. Disseminated intravascular coagulation is an acute consumptive coagulopathy characterized by elevated prothrombin time and partial thromboplastin time, and decreased platelet count. The prothrombin time and partial thromboplastin time are normal in patients with factor V (Leiden) mutation. Hypercholesterolemia promotes atherosclerosis over many years, and the risk of arterial thrombosis increases. Von Willebrand's disease affects platelet adhesion and leads to a bleeding tendency, not to thrombosis.

6 A 25-year-old woman who has had altered consciousness and slurred speech for the past 24 hours is brought to the emergency department. A head CT scan shows a right temporal hemorrhagic infarction. Cerebral angiography shows a distal right middle cerebral arterial occlusion. Within the past 3 years, she has had an episode of pulmonary embolism. A pregnancy 18 months ago ended in miscarriage. Laboratory studies show a false-positive serologic test for syphilis, normal prothrombin time, elevated partial thromboplastin time, and normal platelet count. Which of the following is the most likely cause of these findings? □ (A) Disseminated intravascular coagulation □ (B) Factor V mutation □ (C) Hypercholesterolemia □ (D) Lupus anticoagulant □ (E) Von Willebrand disease

6 (C) This woman has systemic lupus erythematosus (SLE). Patients with SLE can develop anti-RBC antibodies, which can cause hemolytic anemia. Cytopenias, including leukopenia, thrombocytopenia, and anemia, also are common. Bronchoconstriction is a feature of bronchial asthma and can occur in allergies as a predominantly type I hypersensitivity reaction. Cerebral lymphomas are rare, but may occur in immunodeficient patients, particularly patients with AIDS. Keratoconjunctivitis can be seen in Sjögren syndrome as a result of decreased tear production from lacrimal gland inflammation. Sacroiliitis is a feature of many of the spondyloarthropathies, such as ankylosing spondylitis. Sclerodactyly is seen in scleroderma. When extensive, it is usually part of the spectrum of findings associated with diffuse scleroderma; when it involves only a few areas of the skin (e.g., just the hands), it is more likely to indicate limited scleroderma (CREST when it involves only a few areas of the skin (e.g., just the hands), it is more likely to indicate limited scleroderma (CREST syndrome).

6 A 31-year-old woman notices that when she is outside in the sun for more than 1 hour, she develops a rash on her face. Laboratory studies show hemoglobin, 10.9 g/dL; hematocrit, 32.9%; platelet count, 156,800/mm3; and WBC count, 4211/mm3. Urinalysis shows no blood or glucose; there is 3+ proteinuria. The ANA test result is positive with a titer of 1 : 2048 and a diffuse homogeneous immunofluorescent staining pattern. Which of the following complications is most characteristic of her illness? □ (A) Bronchoconstriction □ (B) Cerebral lymphoma □ (C) Hemolytic anemia □ (D) Keratoconjunctivitis □ (E) Sacroiliitis □ (F) Sclerodactyly

6 (E) The focal, chalky white deposits are areas of fat necrosis resulting from the release of pancreatic lipases in a patient with acute pancreatitis. Viral hepatitis does not cause necrosis in other organs, and hepatocyte necrosis from viral infections occurs mainly by means of apoptosis. Intestinal infarction is a form of coagulative necrosis. Tuberculosis produces caseous necrosis. Gangrenous necrosis is mainly coagulative necrosis, but occurs over an extensive area.

6 A 38-year-old woman experienced severe abdominal pain with hypotension and shock that led to her death within 36 hours after the onset of the pain. From the gross appearance of the mesentery, seen in the figure at the bottom of the previous column, which of the following events has most likely occurred? □ (A) Hepatitis B virus infection □ (B) Small intestinal infarction □ (C) Tuberculous lymphadenitis □ (D) Gangrenous cholecystitis □ (E) Acute pancreatitis

6 (B) Wound contraction is a characteristic feature of healing by second intention that occurs in larger wounds. Collagen synthesis helps fill the defect, but does not contract it. The inhibition of metalloproteinases leads to decreased degradation of collagen and impaired connective tissue remodeling in wound repair. Edema diminishes over time, but this does not result in much contraction. Adhesive glycoproteins such as fibronectin help to maintain a cellular scaffolding for growth and repair, but they do not contract.

6 A 40-year-old man underwent laparotomy for a perforated sigmoid colon diverticulum. A wound infection complicated the postoperative course, and surgical wound dehiscence occurred. Primary closure was no longer possible, and the wound "granulated in." Six weeks later, the wound is only 10% of its original size. Which of the following processes best accounts for the observed decrease in wound size over the past 6 weeks? □ (A) Increase in synthesis of collagen □ (B) Myofibroblast contraction □ (C) Inhibition of metalloproteinases □ (D) Resolution of subcutaneous edema □ (E) Elaboration of adhesive glycoproteins

6 (C) Histamine is found in abundance in mast cells, which are normally present in connective tissues next to blood vessels beneath mucosal surfaces in airways. Binding of an antigen (allergen) to IgE antibodies that have previously attached to the mast cells by the Fc receptor triggers mast cell degranulation, with release of histamine. This response causes increased vascular permeability and mucous secretions. Bradykinin, generated from the kinin system on surface contact of Hageman factor with collagen and basement membrane from vascular injury, promotes vascular permeability, smooth muscle contraction, and pain. Complement C5a is a potent chemotactic factor for neutrophils. Interleukin-1 and tumor necrosis factor, both produced by activated macrophages, mediate many systemic effects, including fever, metabolic wasting, and hypotension. Phospholipase C, which catalyzes the release of arachidonic acid, is generated from platelet activation. Platelet-activating factor (PAF) can be released by neutrophils, mast cells, monocytes, macrophages, endothelial cells, and platelets. PAF promotes vascular permeability, neutrophil aggregation, and platelet activation.

6 A woman who is allergic to cats visits a neighbor who has several cats. During the visit, she inhales cat dander, and within minutes, she develops nasal congestion with abundant nasal secretions. Which of the following substances is most likely to produce these findings? □ (A) Bradykinin □ (B) Complement C5a □ (C) Histamine □ (D) Interleukin-1 □ (E) Phospholipase C □ (F) Platelet-activating factor □ (G) Tumor necrosis factor

60 (B) This woman has hereditary angioedema, a rare autosomal-recessive disorder in which there is a deficiency of antigenic or functional C1 inhibitor, resulting in recurrent episodes of edema. Of the remaining choices, only C3 and IgA have a deficiency state. C3 deficiency is accompanied by recurrent infections with pyogenic bacteria. IgA deficiency leads to mild recurrent gastrointestinal and respiratory tract infections and predisposes to anaphylactic transfusion reaction. β2- Microglobulin is a component of MHC class I; it can be increased with HIV infection and can be a substrate for amyloid fibrils in patients receiving long-term hemodialysis. 5-Hydroxytryptamine (serotonin) has an effect similar to histamine, which drives vasodilation and edema. IgE participates in localized or systemic anaphylaxis with edema.

60 A 39-year-old woman sees her physician because of acute onset of severe dyspnea. On physical examination, she is afebrile and has marked laryngeal stridor and severe airway obstruction. The medical history indicates that she has had similar episodes since childhood and episodes of colicky gastrointestinal pain. Her mother and her brother are similarly affected. There is no history of severe or recurrent infections. She does not have urticaria. Laboratory studies show normal WBC count, hematocrit, and platelet count. A deficiency in which of the following plasma components is most likely to produce these findings? □ (A) β2-Microglobulin □ (B) C1 inhibitor □ (C) C3 □ (D) 5-Hydroxytryptamine □ (E) IgA □ (F) IgE

61 (F) Natural killer (NK) cells have CD16, an Fc receptor that allows them to bind to opsonized cells and lyse them. This is a form of type II hypersensitivity with antibody-mediated disease. NK cells comprise 10% to 15% of circulating lymphocytes. NK cells also may lyse human cells that have lost MHC class I expression as a result of viral infection or neoplastic transformation. B cells have surface immunoglobulin, are CD19 positive, and participate in humoral immunity. CD4+ cells are T lymphocytes that are "helper" cells; they have T cell receptors and are CD3 positive. Likewise, CD8+ cells have T cell receptors and mark with CD3, but they act as cytotoxic T lymphocytes. Dendritic cells are a form of antigen-presenting cell that expresses large amounts of MHC class II molecules. Macrophages express MHC II and act as antigen-presenting cells to CD4+ cells; they can phagocytize opsonized cells. Stem cells are CD34 positive and can give rise to the whole range of cells in the immune system and blood-forming cells.

61 In an experiment, a cell line derived from a human malignant neoplasm is grown in culture. A human IgG antibody is added to the culture, and the tumor cells are observed to be coated by the antibody, but they do not undergo lysis. Next, human cells are added that are negative for CD3, CD19, surface immunoglobulin, and T-cell receptor markers, but are positive for CD16 and CD56. The tumor cells are observed to undergo lysis. Which of the following human cell types added to the tumor cell culture is most likely to produce these findings? □ (A) B cell □ (B) CD4+ cell □ (C) CD8+ cell □ (D) Dendritic cell □ (E) Macrophage □ (F) Natural killer cell □ (G) Stem cell

62 (C) Dendritic cells are a form of antigen-presenting cell. Dendritic cells in epithelia are known as Langerhans cells, and those within germinal centers are called follicular dendritic cells (FDCs). The FDCs may become infected, but not killed by HIV. They have cell surface Fc receptors that capture antibody-coated HIV virions through the Fc portion of the antibody. These virions attached to the FDCs can infect passing CD4+ lymphocytes. B cells are a component of humoral immunity, and antibody to HIV does not serve a protective function, but allows serologic detection of infection. CD8+ cells are cytotoxic lymphocytes that lack the receptor necessary for infection by HIV. Because they survive selectively, the CD4+:CD8+ ratio is reversed; it is typically less than 1 with advanced HIV infection. Langhans giant cells are "committees" of activated macrophages that are part of a granulomatous response. Macrophages are a type of antigen-presenting cell that can become infected by HIV without destruction. Mast cells have surface-bound IgE, which can be cross-linked by antigens (allergens) to cause degranulation and release of vasoactive amines, such as histamine, as part of anaphylaxis with type I hypersensitivity. Natural killer cells have Fc receptors, but they are not antigen-presenting cells.

62 During heterosexual intercourse, seminal fluid containing HIV contacts vaginal squamous mucosa. The virions are captured by cells, which then transport the virus via lymphatics to the regional lymph nodes. Within the germinal centers of these lymph nodes, the virions infect CD4+ lymphocytes and proliferate, causing CD4 lysis with release of more virions, which are taken up on the surface of cells having Fc receptors, allowing continued infection by HIV of more CD4+ cells passing through the nodes. Which of the following types of cells is most likely to capture HIV onto its surface via Fc receptors? □ (A) B cell □ (B) CD8+ cell □ (C) Follicular dendritic cell □ (D) Langhans giant cell □ (E) Macrophage □ (F) Mast cell □ (G) Natural killer cell

7 (A) The features described are those of classic Turner syndrome. Individuals who reach adulthood may have mosaic cell lines, with some 45,X cells and some 46,XX cells. A female carrier of the fragile X syndrome, X(fra), is less likely to manifest the disease than a male, but the number of triple repeat sequences (CGG) increases in her male offspring. The 47,XXY karyotype occurs in Klinefelter syndrome; affected individuals appear as phenotypic males. The "superfemale" karyotype (XXX) leads to mild mental retardation. Trisomy 16 is a cause of fetal loss early in pregnancy.

7 A 25-year-old woman with amenorrhea has never had menarche. On physical examination, she is 145 cm (4 ft 9 in) tall. She has a webbed neck, a broad chest, and widely spaced nipples. Strong pulses are palpable in the upper extremities, but there are only weak pulses in the lower extremities. On abdominal MR imaging, the ovaries are small, elongated, and tubular. Which of the following karyotypes is most likely to be present in this patient? □ (A) 45,X/46,XX □ (B) 46,X,X(fra) □ (C) 47,XXY □ (D) 47,XXX □ (E) 47,XX,+16

7 (D) These findings suggest a granulomatous inflammation, and tuberculosis is a common cause. Bacteria such as Staphylococcus and Klebsiella are more likely to produce acute inflammation. Plasmodium produces malaria, a parasitic infection without a significant degree of lung involvement. Candida is often a commensal organism in the oropharyngeal region and rarely causes pneumonia in healthy (non-immunosuppressed) individuals. Viral infections tend to produce a mononuclear interstitial inflammatory cell response.

7 A 32-year-old woman has had a chronic cough with fever for the past month. On physical examination, she has a temperature of 37.5°C, and on auscultation of the chest, crackles are heard in all lung fields. A chest radiograph shows many small, ill-defined nodular opacities in all lung fields. A transbronchial biopsy specimen shows interstitial infiltrates with lymphocytes, plasma cells, and epithelioid macrophages. Which of the following infectious agents is the most likely cause of this appearance? □ (A) Staphylococcus aureus □ (B) Plasmodium falciparum □ (C) Candida albicans □ (D) Mycobacterium tuberculosis □ (E) Klebsiella pneumoniae □ (F) Cytomegalovirus

7 (C) This woman has Sjögren syndrome, which is characterized by immunologically mediated destruction of salivary and lacrimal glands and other exocrine glands lining the respiratory and gastrointestinal tracts. Dryness and crusting of the nose can lead to perforation of the nasal septum. In 25% of cases, extraglandular tissues, such as lung, skin, kidney, and muscles, may be involved. Renal failure is more likely to occur with systemic lupus erythematosus (SLE) from glomerulonephritis. Libman-Sacks endocarditis is most often a feature of SLE. Photosensitivity is a feature of SLE, with formation of an erythematous rash in sun-exposed areas; it also can be a drug reaction. Sclerodactyly is a feature of scleroderma. When not extensive, it typically indicates limited scleroderma (CREST syndrome); when extensive, it indicates diffuse scleroderma, which has a poorer prognosis. Subcutaneous nodules can occur in rheumatic fever as part of the immunological reaction after some group A beta-hemolytic streptococcal infections. Nongonococcal urethritis is seen in Reiter syndrome, along with conjunctivitis and arthritis.

7 A 43-year-old woman has been bothered by a chronic, dry cough for the past 5 years. She has had increasing difficulty with blurred vision for the past year. On physical examination, she has a perforated nasal septum, bilateral mild corneal scarring, and oral cavity fissuring of the tongue and corners of her mouth. Laboratory studies show antibodies to SS-A and SS-B. The serum creatinine is 2.5 mg/dL, and the urea nitrogen is 25 mg/dL. A renal biopsy specimen examined microscopically shows tubulointerstitial nephritis. Which of the following is the most serious condition likely to complicate the course of her disease? □ (A) Chronic renal failure □ (B) Endocarditis □ (C) Non-Hodgkin lymphoma □ (D) Photosensitivity □ (E) Sclerodactyly □ (F) Subcutaneous nodules □ (G) Urethritis

7 (A) Tissue plasminogen activator is a thrombolytic agent that causes the generation of plasmin, which cleaves fibrin to dissolve clots. Aspirin prevents formation of new thrombi by inhibiting platelet aggregation. Heparin prevents thrombosis by activating antithrombin III. Nitric oxide is a vasodilator. Vitamin K is required for synthesis of certain clotting factors.

7 A 66-year-old woman comes to the emergency department 30 minutes after the onset of chest pain that radiates to her neck and left arm. She is diaphoretic and hypotensive; the serum troponin I level is elevated. Thrombolytic therapy is begun. Which of the following drugs is most likely to be administered? □ (A) Tissue plasminogen activator □ (B) Aspirin □ (C) Heparin □ (D) Nitric oxide □ (E) Vitamin K

7 (C) Integrins interact with the extracellular matrix proteins (e.g., fibronectin). Engagement of integrins by extracellular matrix proteins leads to the formation of focal adhesions at which integrins link to intracellular cytoskeletal elements such as actin. These interactions lead to intracellular signals that modulate cell growth, differentiation, and migration during wound healing. Epidermal growth factor stimulates epithelial cell and fibroblast proliferation. Platelet-derived growth factor (PDGF) can be produced by endothelium, macrophages, smooth muscle cells, and platelets; PDGF mediates migration and proliferation of fibroblasts and smooth muscle cells and migration of monocytes. Type IV collagen is found in basement membranes on which cells are anchored. Vascular endothelial growth factor promotes angiogenesis (capillary proliferation) through endothelial cell proliferation and migration in a healing response.

7 In an experiment involving observations on wound healing, researchers noted that intracytoplasmic cytoskeletal elements, including actin, interact with the extracellular matrix to promote cell attachment and migration in wound healing. Which of the following substances is most likely responsible for such interaction between the cytoskeleton and the extracellular matrix? □ (A) Epidermal growth factor □ (B) Fibronectin □ (C) Integrin □ (D) Platelet-derived growth factor □ (E) Type IV collagen □ (F) Vascular endothelial growth factor

7 (B) Intracellular mechanisms exist that deal with free radical generation, as can occur with radiant injury from irradiation. Glutathione peroxidase reduces such injury by catalyzing the breakdown of hydrogen peroxide. Phospholipases decrease cellular phospholipids and promote cell membrane injury. Proteases can damage cell membranes and cytoskeletal proteins. Endonucleases damage nuclear chromatin. Lactate dehydrogenase is present in a variety of cells, and its elevation in the serum is an indicator of cell death.

7 In an experiment, cells are subjected to radiant energy in the form of x-rays. This results in cell injury caused by hydrolysis of water. Which of the following cellular enzymes protects the cells from this type of injury? □ (A) Phospholipase Robbins & Cotran Review of Pathology Pg. 2 □ (A) Phospholipase □ (B) Glutathione peroxidase □ (C) Endonuclease □ (D) Lactate dehydrogenase □ (E) Protease

8 (B) Acute left ventricular failure after a myocardial infarction causes venous congestion in the pulmonary capillary bed and increased hydrostatic pressure, which leads to pulmonary edema by transudation in the alveolar space. Neutrophils and fibrin would be found in cases of acute inflammation of the lung (i.e., pneumonia). Fibrosis and hemosiderin-filled macrophages (heart failure cells) would be found in long-standing, not acute, left ventricular failure. Subpleural hemorrhagic necrosis occurs if there are pulmonary thromboemboli. These thromboemboli can cause right-sided heart failure. Purulent exudate in the pleural space (empyema) or draining from bronchi results from bacterial infection, not heart failure.

8 A 49-year-old man is in stable condition after an infarction of the anterior left ventricular wall. He receives therapy with anti-arrhythmic and pressor agents. He develops severe breathlessness 3 days later, and an echocardiogram shows a markedly decreased ejection fraction. He dies 2 hours later. At autopsy, which of the following microscopic changes is most likely to be present in the lungs? □ (A) Congestion of alveolar capillaries with fibrin and neutrophils in alveoli □ (B) Congestion of alveolar capillaries with transudate in alveoli □ (C) Fibrosis of alveolar walls with hemosiderin-laden macrophages in alveoli □ (D) Multiple areas of subpleural hemorrhagic necrosis □ (E) Purulent exudate in the pleural space □ (F) Purulent exudate in the mainstem bronchi

8 (B) RAS proteins transduce signals from growth factor receptors, such as epidermal growth factor, that have intrinsic tyrosine kinase activity. G proteins perform a similar function for G protein-linked, seven-spanning receptors. Cyclic AMP is an effector in the G protein signaling pathway. Cyclins and cyclin-dependent kinases regulate the cell cycle in the nucleus.

8 In an experiment, release of epidermal growth factor into an area of denuded skin causes mitogenic stimulation of the skin epithelial cells. Which of the following proteins is most likely to be involved in transducing the mitogenic signal from the epidermal cell membrane to the nucleus? □ (A) G proteins □ (B) RAS proteins □ (C) Cyclin D □ (D) Cyclic AMP □ (E) Cyclin-dependent kinase

9 (A) This boy has Down syndrome (trisomy 21), one of the trisomies that can result in a live-born infant. Although children with Down syndrome can function well, they often have many associated congenital anomalies. Among the more common is congenital heart disease, including ventricular septal defect. There is also a 10-fold to 20-fold increased risk of acute leukemia. Virtually all individuals with Down syndrome who live to age 40 have evidence of Alzheimer disease. Hepatic cirrhosis is a feature of galactosemia. Chronic renal failure may be seen in genetic disorders that produce polycystic kidneys. Myocardial infarction at a young age suggests familial hypercholesterolemia. Aortic dissection is seen in individuals with Marfan syndrome.

9 A 10-year-old boy who is mentally retarded is able to carry out activities of daily living, including feeding and dressing himself. On physical examination, he has brachycephaly and oblique palpebral fissures with prominent epicanthal folds. A transverse crease is seen on the palm of each hand. On auscultation of the chest, there is a grade III/VI systolic murmur. Which of the following diseases is he most likely to develop by age 20? □ (A) Acute leukemia □ (B) Hepatic cirrhosis □ (C) Chronic renal failure □ (D) Acute myocardial infarction □ (E) Aortic dissection

9 (D) These findings are characteristic of decompression sickness (the "bends"). At high pressures, such as occur during a deep scuba dive, nitrogen is dissolved in blood and tissues in large amounts. Ascending too quickly does not allow for slow release of the gas, and formation of small gas bubbles causes symptoms from occlusion of small arteries and arterioles. Hemorrhage or thrombosis from disseminated intravascular coagulation is more likely to occur in underlying diseases such as sepsis, and symptoms do not abate so quickly. Systemic vasodilation is a feature of some forms of shock. Venous thrombosis is more typically a complication of stasis, which does not occur in a physically active individual. Fat globules in pulmonary arteries are a feature of fat embolism, which usually follows trauma.

9 A 27-year-old man is on a scuba diving trip to the Caribbean and descends to a depth of 50 m in the Blue Hole off the coast of Belize. After 30 minutes, he has a malfunction in his equipment and quickly returns to the boat on the surface. He soon experiences difficulty breathing, with dyspnea and substernal chest pain, followed by a severe headache and vertigo. About 1 hour later, he develops severe, painful myalgias and arthralgias. These symptoms abate within 24 hours. Which of the following mechanisms is the most likely cause of these symptoms? □ (A) Disseminated intravascular coagulation □ (B) Systemic vasodilation □ (C) Venous thrombosis □ (D) Tissue nitrogen emboli □ (E) Fat globules in arterioles

9 (D) This patient has AIDS, with Kaposi sarcoma of the skin. Kaposi sarcoma is associated with a herpesvirus agent that is sexually transmitted: human herpesvirus 8 (HHV-8), also called the Kaposi sarcoma herpesvirus. Other herpesviruses are not involved in the pathogenesis of Kaposi sarcoma, although infection with these viruses can occur frequently in individuals with AIDS. HIV, although present in the lymphocytes and monocytes, is not detected in the spindle cells that proliferate in Kaposi sarcoma. With the exception of varicella-zoster virus, which is associated with dermatomally distributed skin vesicles known as shingles, skin lesions are uncommon manifestations of herpesviruses, which include cytomegalovirus, Epstein-Barr virus, and adenovirus.

9 A 37-year-old man who is HIV-positive has noticed multiple 0.5- to 1.2-cm plaquelike, reddish purple skin lesions on his face, trunk, and extremities. Some of the larger lesions appear to be nodular. These lesions have appeared over the past 6 months and have slowly enlarged. Molecular analysis of the spindle cells found in these skin lesions is likely to reveal the genome of which of the following viruses? □ (A) Cytomegalovirus □ (B) Epstein-Barr virus □ (C) Adenovirus □ (D) Human herpesvirus-8 □ (E) HIV-1

1 (C) Irreversible cell injury is associated with loss of membrane integrity. This allows intracellular enzymes to leak into the serum. All other morphologic changes listed are associated with reversible cell injury, in which the cell membrane remains intact.

A 17-year-old boy infected with hepatitis A experiences mild nausea for about 1 week and develops very mild scleral icterus. On physical examination, he has minimal right upper quadrant tenderness. Laboratory findings include a serum AST of 68 U/L, ALT of 75 U/L, and total bilirubin of 5.1 mg/dL. The increase in this patient's serum enzyme levels most likely results from which of the following changes in the hepatocytes? □ (A) Autophagy by lysosomes □ (B) Clumping of nuclear chromatin □ (C) Defects in the cell membrane □ (D) Dispersion of ribosomes □ (E) Swelling of the mitochondria


संबंधित स्टडी सेट्स

Blueprint: Stoichiometry and Basic Chemistry Math

View Set

Encyclopedia of Counseling: Ch 3 - Human Growth and Development, Encyclopedia of Counseling: Ch 9 - Research and Program Evaluation, Encyclopedia of Counseling: Ch 11 - Counseling Families, Diagnosis, and Advanced Concepts, Encyclopedia of Counseling...

View Set

Chapter 9 Microbial Genetics Learn Smart

View Set

ARE 336 Exam 2 Part one, practice exams

View Set

Mr. Burns- Chorus/Ensemble Lines

View Set

Fundamentals Nursing Prep U Chapter 7 Legal Dimensions of Nursing Practice

View Set

Management Exam 1 ( ch. 1, 2, 3, 4, 12, 13)

View Set